prueba 1 - math.cubava.cumath.cubava.cu/files/2015/05/solucionario-libro-jacinto.pdf · g 9 2x 3 x...

135
1 Solucionario del Jacinto PRUEBA 1 1.-La ecuación dada puede escribirse en la forma x x 3 2 9 log 2 y ahora podemos convertir a potencia haciendo la transformación siguiente x x 3 2 2 9 que puede ponerse como x x 2 2 2 9 3 donde, para facilitar el trabajo, realizaremos un cambio de variable y = 2 x ahora, luego de eliminar el denominador, la ecuación toma la forma 9 y y 2 = 8 en la que ordenando convenientemente y haciendo la descomposición en factores ( y 8 )( y 1 )=0 llegamos a la conclusión parcial y = 8 ó y = 1 para los que x = 3 ó x = 0. Solución única x = 0, pues x = 3 no pertenece al dominio de la ecuación original. 2.-Una simple inspección nos conduce a que cm AC AB 6 . Podemos concluir entonces que cm CD 12 ya que tiene que ser el doble del cateto que se opone al ángulo de 30° y decidimos que cm AD 3 6 (se opone al ángulo de 60 0 ) AB AD BD = cm 38 , 4 73 , 0 . 6 ) 1 73 , 1 ( 6 ) 1 3 ( 6 6 3 6 . El arco EB nos define el sector DBE del que calculamos su área aplicando la fórmula correspondiente 2 2 2 1 , 5 12 4 , 19 . 14 , 3 12 4 , 4 . 14 , 3 12 cm r A El área del triángulo rectángulo ADC se calcula 2 1 , 31 73 , 1 . 18 2 3 6 . 6 cm A El área sombreada es entonces As=31,1 18 5,1 = 8,0cm 2 . 3.-Consideremos hermano mayor: x Se puede formar el sistema hermano mediano: y 10 z z y x hermano menor: z 672 2 2 2 z y x x y = 4 La primera ecuación se puede escribir como x + y 9z = 0 , y la última se puede expresar por x = 4 + y , donde 9 4 2 y z Si sustituimos todas estas expresiones en la ecuación cuadrática nos queda 672 9 4 2 4 2 2 2 y y y Esta ecuación, después de hacer las operaciones indicadas y eliminar el denominador 81 puede expresarse como 0 53120 664 166 2 y y : 166 0 320 4 2 y y que admite la factorización ( y + 20 )( y 16 )=0 donde y =16 despreciando la solución negativa. Hermano mediano: 16 años, hermano mayor: 20 años y hermano menor: 4 añitos.

Upload: trinhhanh

Post on 28-Oct-2018

216 views

Category:

Documents


0 download

TRANSCRIPT

Page 1: PRUEBA 1 - math.cubava.cumath.cubava.cu/files/2015/05/solucionario-libro-jacinto.pdf · g 9 2x 3 x 2 y ahora podemos ... llegamos a la conclusión parcial y = 8 ó y = 1 para los

1

Solucionario del Jacinto

PRUEBA 1

1.-La ecuación dada puede escribirse en la forma xx 329log 2 y ahora podemos

convertir a potencia haciendo la transformación siguiente xx 3229 que puede ponerse

como x

x

2

229

3

donde, para facilitar el trabajo, realizaremos un cambio de variable y = 2x

ahora, luego de eliminar el denominador, la ecuación toma la forma 9 y – y2 = 8 en la que

ordenando convenientemente y haciendo la descomposición en factores ( y – 8 )( y – 1 )=0

llegamos a la conclusión parcial y = 8 ó y = 1 para los que x = 3 ó x = 0. Solución única

x = 0, pues x = 3 no pertenece al dominio de la ecuación original.

2.-Una simple inspección nos conduce a que cmACAB 6 . Podemos concluir

entonces que cmCD 12 ya que tiene que ser el doble del cateto que se opone al

ángulo de 30° y decidimos que cmAD 36 (se opone al ángulo de 600)

ABADBD = cm38,473,0.6)173,1(6)13(6636 .

El arco EB nos define el sector DBE del que calculamos su área aplicando la fórmula

correspondiente

222

1,512

4,19.14,3

12

4,4.14,3

12cm

rA

El área del triángulo rectángulo ADC se calcula 21,3173,1.18

2

36.6cmA

El área sombreada es entonces As=31,1 – 18 – 5,1 = 8,0cm 2 .

3.-Consideremos hermano mayor: x Se puede formar el sistema

hermano mediano: y 10

z

zyx

hermano menor: z 672222 zyx

x – y = 4

La primera ecuación se puede escribir como x + y – 9z = 0 , y la última se puede expresar por

x = 4 + y , donde 9

42

yz Si sustituimos todas estas expresiones en la ecuación

cuadrática nos queda 6729

424

2

22

yyy

Esta ecuación, después de hacer las operaciones indicadas y eliminar el denominador

81 puede expresarse como 053120664166 2 yy : 166

032042 yy que admite la factorización ( y + 20 )( y – 16 )=0 donde y =16

despreciando la solución negativa. Hermano mediano: 16 años, hermano mayor: 20 años y

hermano menor: 4 añitos.

Page 2: PRUEBA 1 - math.cubava.cumath.cubava.cu/files/2015/05/solucionario-libro-jacinto.pdf · g 9 2x 3 x 2 y ahora podemos ... llegamos a la conclusión parcial y = 8 ó y = 1 para los

2

4.-La desigualdad se puede escribir tomando como base la fracción 1/4 =0,25 .

4

1

4

1 2

47

x

x

donde, como la base es menor que 1, para trabajar con los exponentes

tenemos que considerar 12

47

x

x ; comparando con cero 01

2

47

x

x y ahora, operando

y reduciendo términos semejantes, llegamos a 02

)1(6

x

x que nos lleva a los intervalos

+ – 2 – 1 +

//////////////////////O////////////////////

y entonces se cumple para x – 2 ó x 1 .

5.-En la base tenemos que 362 a por lo que a=6 y trabajando con el área lateral llegamos a

que a

aa hhha

.122

.6.4

2

..460 donde cmha 5 .

Podemos calcular la altura de la pirámide aplicando el teorema de Pitágoras en el triángulo de

apoyo de la cara lateral, que es rectángulo

cma

hh ap 4352

22

2

2

y ahora hallamos la altura del cilindro mediante el

cálculo h=(3/4) .4 = 3cm . Si aplicamos el teorema de las transversales nos queda

que r

1

3

4 donde r =3/4=0,75cm. (radio del cilindro)

Vr =Vp – Vc= 332 437,423,5483.75,0.14,33

4.36.

3

.cmcmhA

hAcBc

pB

PRUEBA 2

2522 yx

1.-El sistema puede escribirse como 11

log 5

x

xy donde ( y – x ).x = 5 , entonces

despejando la variable “y” obtenemos x

xy

25 , ahora sustituyendo en la primera ecuación

queda 255

22

2

x

xx ,operando y eliminando el denominador llegamos a la ecuación

025152 24 xx que admite la factorización 0)5)(52( 22 xx

52,1 x ; 2

104,3 x y donde podemos notar que los valores negativos no pertenecen

Page 3: PRUEBA 1 - math.cubava.cumath.cubava.cu/files/2015/05/solucionario-libro-jacinto.pdf · g 9 2x 3 x 2 y ahora podemos ... llegamos a la conclusión parcial y = 8 ó y = 1 para los

3

al dominio del sistema original, por lo que las soluciones son los pares ordenados

)52;5( ;

2

103;

2

10

2.- a) El triángulo ABC es rectángulo en A por el recíproco del teorema de Pitágoras pues 222 121620 . Resulta que los triángulos ADC y CDE son iguales pues ambos son

rectángulos con hipotenusa común y ACD=DCE debido a la bisectriz. Entonces los lados

AD y DE son iguales por elementos correspondientes en triángulos iguales.

Podemos calcular el área del triángulo ABC; 2962

16.12

2

.cm

ACABA y plantear

DEDEDEDEDE

AA ADCBDC .18.8.102

.16

2

.2096

Donde DE = 5,3cm

b) 16CEAC (lados correspondientes en triángulos iguales)

41620 CEBCBE

por lo que el área del triángulo 22 116,10

2

4.3,5

2

.cmcm

BEDEA BDE

3.-Consideremos la distancia entre las ciudades como x

Nos conduce a la ecuación x = (1/4) x + (1/5) x + 12,5%[x – (9/20) x ] + 55 que se facilita

mediante x = (1/4) x + (1/5) x + (1/8).(11/20) x + 55 y se llega a que

77 x = 8800 donde x = 114,29km . Podemos calcular (1/8).(11/20).114 = 7,83

La distancia entre las ciudades es 114km. En camión recorrió 7,8km .

4.-Haciendo las sustituciones pertinentes podemos llegar a la ecuación

3)2(22

xgxf donde 32cos7)2sen3( 2 xx

y llegamos a la ecuación 032cos72sen3 2 xx

que puede escribirse como 032cos7)2cos1(3 2 xx

032cos72cos33 2 xx

llegando a la ecuación final cuadrática 02cos72cos3 2 xx

cos2x .( 3cos2x –7 ) = 0 donde cos2x = 0 ó cos2x = 7/3 (imposible)

por lo que 2x = (2k+1)/2 o sea x = (2k+1)/4 ; k entero. Pero en el intervalo dado

únicamente son soluciones x = 3/4 ó x = 5/4.

Page 4: PRUEBA 1 - math.cubava.cumath.cubava.cu/files/2015/05/solucionario-libro-jacinto.pdf · g 9 2x 3 x 2 y ahora podemos ... llegamos a la conclusión parcial y = 8 ó y = 1 para los

4

5.-El triángulo EAC es rectángulo en A ya que el prisma es recto y EA es altura.

Como .32cmEC entonces cmAC 16 pues se opone al ángulo de 30° en el triángulo

rectángulo y por semejante motivo cmAE 316 (se opone al ángulo de 60°).

En la base se puede calcular fácilmente que cmAB 10 en los triángulos rectángulos que

forman las diagonales del rombo cuando se cortan en su punto medio.

Entonces AL = 4. A(cara lateral)= AEAB..4 =22 11110773,1.16.40316.10.4 dmcm

Y el volumen se puede calcular,

Vprisma = .7,2265773,1.15363.16.2

12.16.

2

.. 33 dmcmh

BDAChAB

PRUEBA 3

1.-La ecuación dada, acomodando las bases, puede expresarse de la siguiente forma

xxx 42 2.72.4121 donde sugerimos hacer el cambio

xy 2 para luego

elevar ambos miembros al cuadrado 422 741.21 yyyy ; simplificamos la

expresión y volvemos a elevar al cuadrado ambos miembros donde obtenemos 42234 7444 yyyyy para llegar a 048 34 yy que se puede factorizar y concluir

que 03 y ó y = 1/2 ; la primera es imposible y de la segunda obtenemos, haciendo

reversible el cambio de variable, x = –1 ,que se debe comprobar en la ecuación original.

2.- Erróneamente podemos pensar que los triángulos son iguales (las apariencias a veces nos

engañan) y únicamente podemos concluir que AED ~ CDM ya que ambos triángulos son

rectángulos y AED = CDM (alternos entre paralelas) , o sea, son semejantes porque

tienen dos ángulos respectivamente iguales.

El área del triángulo AED se calcula fácilmente 216

2

8.4

2

.cm

ADAEA

Podemos calcular, aplicando el teorema de Pitágoras, la longitud del segmento

cmDE 548048 22 y entonces la razón de semejanza está dada por

5

52

54

8

DE

CDk ya que esos segmentos son proporcionales. Calculamos el área del

triángulo CDM mediante 2

2

2 8,1225

16.5.416.

5

52. cmAkA AEDCDM

As = 64 – ( 16 + 12,8) = 35,2 concluimos que 235cmAs .

Page 5: PRUEBA 1 - math.cubava.cumath.cubava.cu/files/2015/05/solucionario-libro-jacinto.pdf · g 9 2x 3 x 2 y ahora podemos ... llegamos a la conclusión parcial y = 8 ó y = 1 para los

5

3.-En principio tenemos que considerar 2x+50 y 216 x 0 lo que nos conduce a que

necesariamente x – 5 /2 y – 4 x 4 , o sea, – 5 /2 x 4. ¡Ahora a trabajar! La

expresión original puede escribirse como 116

52log

25

x

x donde podemos expresar que

5

1

16

522

x

x y después de comparar con cero y restar las fracciones llegamos a la

factorización 0)4)(4(5

)1)(9(

xx

xx de la que obtenemos los intervalos

+ – 9 – – 4 + –1 – 4 +

O////////////////////OO/////////////////////O

y nos conduce a – 9 x – 4 ó – 1 x 4 y que al intersectar con el intervalo del

análisis inicial nos queda finalmente que la solución es – 1 < x < 4.

4.-Tenemos que a + b + c = 24 pero a + b = 14 donde c =10 . Podemos ahora obtener el

sistema de ecuaciones a + b = 14

a2 + b2 = 102 Teorema de Pitágoras

b = 14 – a por lo que sustituyendo en la segunda ecuación nos conduce a la ecuación

cuadrática en una variable a2 + (14 – a)2 = 100. Operando y reduciendo términos semejantes

podemos llegar a la ecuación a2 – 14a + 48 = 0 que se puede factorizar como

( a – 6 )( a – 8 ) = 0 lo que nos aporta soluciones simétricas a = 6 ; b = 8 ó a= 8 ; b = 6.

El área del triángulo se puede calcular fácilmente 2242

8.6mA .

5.-a)La pirámide es recta ya que todas sus caras laterales son iguales (triángulos equiláteros)

entonces las proyecciones de las aristas laterales sobre el plano base son todas iguales y hace

que el pie de la altura esté en el centro del cuadrado base.

b) 22

22 9828,9873,2.36)31(

4

3.4 cmaa

aAAA LBT

c) La diagonal del cuadrado base mide cm26 y su mitad es cm23 . Para calcular la altura

de la pirámide acudimos al teorema de Pitágoras en uno de los triángulos que forman las

aristas laterales con sus proyecciones sobre la base.

cmh 23181836)23(6 22 .

3518,5041,1.363

23.36

3

.cm

hAV B

d) 123

23

2

d

htan donde concluimos que = 45.

Page 6: PRUEBA 1 - math.cubava.cumath.cubava.cu/files/2015/05/solucionario-libro-jacinto.pdf · g 9 2x 3 x 2 y ahora podemos ... llegamos a la conclusión parcial y = 8 ó y = 1 para los

6

PRUEBA 4

1.-Acomodando las bases y operando en el miembro izquierdo podemos llegar a obtener

la ecuación xxx 22 22

donde nos queda, igualando ahora los exponentes ya

que las bases son iguales, xxx 22 . Al elevar al cuadrado ambos miembros

llegamos a xxxx 2422 que nos conduce a la ecuación mucho más sencilla

442 xx que después de volver a elevar al cuadrado, operar y reducir términos

semejantes nos queda x2 – 4x = 0 con posibles soluciones x = 0 ó x= 4 donde, comprobando,

llegamos a que S={4}.

2.-Podemos llegar a que ABC ADB pues ambos triángulos son rectángulos (el primero

en C por el teorema de Tales y el segundo en A por ser punto de tangencia con un diámetro de

la circunferencia) además, ABC = DBA debido a la bisectriz, por lo que ambos

triángulos tienen dos ángulos respectivamente iguales.

b) Se puede establecer la proporcionalidad entre los lados en los triángulos que hemos probado

que son semejantes AC

AD

AB

BD

BC

AB pero de la primera igualdad se puede obtener

rápidamente 95,2.6,3.2

BCBDAB donde cmAB 3 y ahora podemos calcular el área

del círculo mediante la fórmula

22

1,706,74

9.14,3

4

.cm

dAO

3.-Si operamos y reducimos términos semejantes en la función g(x) podemos llegar a que

g(x)=1 por lo que podemos escribir 14

3019log

2

3

4

x

xx donde aplicando la operación

inversa nos queda 44

30192

3

x

xx que comparando con cero y efectuando en el miembro

izquierdo obtenemos 04

141942

23

x

xxx y que al factorizar el numerador y denominador

nos da 0)2)(2(

)2)(7)(1(

xx

xxx donde analizamos

– – 2 – –1 + 2 – 7 +

OO//////////////////////OO//////////////////

( observar que – 2 es un cero doble y entonces a su alrededor no cambia el signo)

y obtenemos los intervalos –1< x <2 ó x > 7 con x – 2 pero también tenemos que

considerar al inicio que 04

30192

3

x

xx y factorizando obtenemos 0

)2)(2(

)2)(3)(5(

xx

xxx

Page 7: PRUEBA 1 - math.cubava.cumath.cubava.cu/files/2015/05/solucionario-libro-jacinto.pdf · g 9 2x 3 x 2 y ahora podemos ... llegamos a la conclusión parcial y = 8 ó y = 1 para los

7

para llegar a los siguientes intervalos (observar que – 2 es un cero doble, esto nos ha

ocurrido porque decidimos no simplificar en este caso)

– – 3 + – 2 + 2 – 5 +

O/////////////O///////////////////////////////////OO///////////////////////////

donde nos queda que –3 < x < 2 ó x > 5 con x – 2 . Al intersectar y tomar las que son no

negativas nos queda que 0 x < 2 ó x > 7. Valdría la pena preguntarse si es necesario

analizar esta última desigualdad.

4.- La figura puede descomponerse en tres triángulos rectángulos iguales al trazar los

segmentos AC y EC , por lo que cada uno tendrá un área de 210 m2. Considerando que

aAB y bBC entonces nos queda el sistema a . b = 420

a2 + b2 = 292 Teorema de Pitágoras

despejando b = 420/a y sustituyendo en la segunda ecuación obtenemos

841420

2

22

aa y al eliminar el denominador obtenemos la ecuación

0176400841 24 aa donde se puede llegar a la descomposición en factores

(a2 – 400 )(a2 – 441) =0 que nos conduce a los valores a = 20 ó a = 21 y entonces b = 21 ó

b = 20 pero b > a por lo que necesariamente b = 21m y a = 20 m. El perímetro es

de 112 m y la cantidad de alambre será 5.112 m = 560 m por lo que la cerca costará

entonces 0,15.560 = 84. La cerca cuesta $84,00.

5.- Si = 45° entonces el triángulo que se forma, con la arista lateral y su proyección sobre

la base, es rectángulo isósceles por lo que h = d /2 y como d = 4 2 entonces

h=2 cm8,282,241,1.22 . El volumen se calcula entonces mediante

³159,143

8,2.16

3

.cm

hAV B .

PRUEBA 5

1.- Al sustituir las funciones nos quedará la ecuación 22

1sen

2

1cos 22 xx y

haciendo la sustitución cos2 x = 1– sen2 x y elevando al cuadrado ambos miembros llegamos

a la ecuación 42

1sen

2

1sensen

2

32sen

2

3 2222

xxxx

que puede ser escrita como 1sen4

3sen 42 xx donde se puede obtener que

1sensen4

3 42 xx y concluimos que 4sen4x – 4sen2 x +1=0 que admite la factorización

Page 8: PRUEBA 1 - math.cubava.cumath.cubava.cu/files/2015/05/solucionario-libro-jacinto.pdf · g 9 2x 3 x 2 y ahora podemos ... llegamos a la conclusión parcial y = 8 ó y = 1 para los

8

(2sen2 x – 1)2 =0 donde 2sen2 x=1 y entonces sen2 x = 1/2 con lo que llegamos a

sen x = 2

2 donde x=(2k+1)./4 ; k entero.

2.- La recta r contiene también al punto (–2;0) , por dato pasa por (0;2) y su pendiente se

puede obtener por mr = tan 450 = 1 . Su ecuación se puede obtener mediante la expresión

y – 0 =1(x+2) o sea r : x – y = – 2 . Como la recta s es perpendicular a r entonces su

pendiente será ms = –1 y su ecuación se puede obtener por

y – 0 = – 1 (x – 6) donde ponemos s : x + y = 6 . Si intersectamos ahora las dos rectas

mediante el sistema x + y = 6

x – y = – 2 concluimos que 2x = 4 por lo que

x = 2 ; y = 4 o sea P(2;4) con r s ={P}

El área de todo el triángulo grande que se forma entre las rectas y el eje X se calcula mediante

2162

4.8uAT y podemos entonces restar la pequeña área del triángulo que se forma a la

izquierda entre la recta r y los ejes de coordenadas 222

2.2uAP

Por tanto AC=AT – AP =16 – 2 =14u2. Hay otras maneras de calcular esta área.

3.- En primer lugar se tiene que cumplir que 0114

642 2

x

xx para que esté definida la

función logarítmica y por otra parte 01114

642log

2

2

x

xx para que se pueda definir la

raíz cuadrada. Esta última expresión nos conduce a 1114

642log

2

2

x

xxque, como la base

es mayor que 1, podemos transformarla en 2114

642 2

x

xx que podemos apreciar con

claridad que prevalece con respecto a la primera que obtuvimos y con la que, por supuesto, no

vamos entonces a trabajar. Si en esta última hacemos la comparación con cero y realizamos

las operaciones, trabajando con el denominador común, llegamos a

0114

228642 2

x

xxx que admite 0

114

)2)(4(2

x

xx

y que al trabajar con los factores hallamos la representación siguiente:

– 2 + 11/4 – 4 +

///////////////////////O////////////////////

Dom h = {x reales : 2 x 11/4 ó x 4 }

Page 9: PRUEBA 1 - math.cubava.cumath.cubava.cu/files/2015/05/solucionario-libro-jacinto.pdf · g 9 2x 3 x 2 y ahora podemos ... llegamos a la conclusión parcial y = 8 ó y = 1 para los

9

4.-Definiremos A1 : área de la alfombra A : área del salón

A2 : área descubierta

Tenemos por dato que A1 = A 2 y se cumple que A = A1 + A2 de donde podemos obtener,

considerando el ancho del salón como x y el largo como x+6, al trabajar con el cálculo de

áreas mediante las expresiones con la variable x , la ecuación siguiente:

x ( x + 6 ) = x ( x – 6 ) + x ( x – 6 ) lo que nos conduce a la ecuación cuadrática

x2 + 6x = 2x2 – 12x que se transforma en x2 – 18x = 0 que nos brinda las posibles

soluciones x = 0 ó x = 18 . Por supuesto que la primera es imposible. Podemos entonces

calcular que el ancho del salón es 18 m y que el largo es 24 m.

a) 22

1 2,22162

24.18

2damm

AA

b) P = 2 ( 18 + 24 ) = 2 . 42 = 84 m

5.-Como el triángulo ABC es rectángulo en C y cmAB 0,8 entonces cmBC 0,4 pues se

opone al ángulo de 300 y CBA = 600 por complementario.

Entonces el DAB = 600 por ángulos base del trapecio isósceles, por lo que DAC=300 por

suma de ángulos consecutivos. Se cumple que CAB=DCA por alternos entre paralelas y

entonces el triángulo ACD es isósceles de base AC por lo que cmCBDCAD 0,4 .

Podemos calcular el área del triángulo base ADC utilizando el ADC=1200 por suma de los

ángulos interiores del triángulo base .

20 342

3.8120sen

2

4.4sen.

2

.cmADC

DCADAT (existen otras vías para el cálculo

del área de este triángulo)

Calculamos el volumen del prisma pues conocemos su altura,

34252,4173,1.246.34. cmhAV T

Page 10: PRUEBA 1 - math.cubava.cumath.cubava.cu/files/2015/05/solucionario-libro-jacinto.pdf · g 9 2x 3 x 2 y ahora podemos ... llegamos a la conclusión parcial y = 8 ó y = 1 para los

10

PRUEBA 6

1.-La ecuación dada puede expresarse en la misma base de la siguiente manera

)cos8log()sen4log(3senlog 22

33.3 xxx y luego

)cos8log()sen4log(3senlog 22

33 xxx .

Igualamos los exponentes )cos8log()sen4log(3senlog 22 xxx y utilizando las

propiedades de los logaritmos )cos8log(]sen4.3senlog[ 22 xxx que nos permite

igualar los argumentos xxx 22 cos83sen.sen4 donde el miembro derecho se puede

expresar por 9 – sen2x y entonces elevamos al cuadrado ambos miembros

16sen2x ( sen2x + 3 ) = 81 – 18sen2x + sen4x que puede escribirse más simplificada como

5sen4x + 22sen2x – 27 = 0 que al factorizarla queda así

( 5sen²x + 27 )( sen²x – 1)=0 y admite las posibles soluciones reales sen x = 1 pues el

primer factor conduce a resultados imposibles, por lo que, en el intervalo dado, solo podemos

obtener x = /2 ó x = 3/2 ; pero esta última no pertenece al dominio de la ecuación original.

2.-Trazamos los radios OC y OB y se forman dos triángulos rectángulos iguales (lo que

puede probarse fácilmente) para luego calcular el radio del círculo ya que conocemos su área

78,5 = .r2 donde 2514,3

5,785,782

r por lo que r = 5 cm y entonces, como OA es

bisectriz del CAB entonces concluimos que OAC=300 y luego COA=600 (ángulos

complementarios) donde cmCA 35 ya que se opone al ángulo de 600 en el triángulo

rectángulo. Podemos calcular el área de todo el cuadrilátero mediante

AC= 2.AT = 2.23,4373,1.2535.5

2

.cm

CAOC y ahora calculamos el área del sector

circular COB que, como su ángulo es de 1200 , es la tercera parte del área del círculo que ya

conocemos, entonces AS= 22,263

5,78cm donde

AS = AC – AS = 43,3 – 26,2 = 17,1cm2 .

3.-Evaluando la función f para la suma indicada nos conduce a

122

539

4

56142

2

53.3

2

53

2

532

f por otra parte tenemos

g(0) – 1 = 4 – 1 = 3 y nos queda entonces resolver la desigualdad

473

3

23

122

xxxx

donde 0)1)(43(

3

)1)(2(

1

xxxx que cuando

efectuamos, con un mínimo denominador común, queda 0)43)(1)(2(

2

xxx que nos

Page 11: PRUEBA 1 - math.cubava.cumath.cubava.cu/files/2015/05/solucionario-libro-jacinto.pdf · g 9 2x 3 x 2 y ahora podemos ... llegamos a la conclusión parcial y = 8 ó y = 1 para los

11

lleva al análisis de los siguientes intervalos

– 1 + 4/3 – 2 +

///////////////////OO/////////////////O

las soluciones son los números x reales con x < 1 ó 4/3 < x < 2 .

4.-Apreciamos que todas las frases se refieren al segundo tiro, por lo que vamos a escribir

todas las expresiones en función del mismo

1er tiro : x + 1

2do tiro : x tenemos la ecuación x + (x + 1) + (x – 2) = 26

3er tiro : x – 2 donde 3 x = 27

donde nos queda que x = 9. Los puntos por disparo fueron 10, 9 y 7 en este orden.

5.-Consideremos, en el rectángulo base de la pirámide recta, que el ancho es a y que su largo

es b ( a < b ) donde podemos entonces escribir el sistema siguiente a partir de los datos dados

a . b = 60 ; 2 a – b = 2 luego podemos despejar b = 2a – 2 y hacer la sustitución

a ( 2a – 2 ) = 60 que nos conduce a la ecuación a2 – a – 30 = 0 que se puede factorizar y

obtener los posibles valores a = 6 ó a = – 5 (imposible).

Donde a = 6 y b = 10 en la base de la pirámide. Para calcular ahora la altura de la

pirámide, pero necesitamos primero calcular la altura de la cara lateral (triángulo isósceles que

tiene área de 39 m2) en la fórmula 2

.39 aha

donde ma

ha 136

7839.2 y ahora, en el

triángulo rectángulo de apoyo de la cara lateral (la cara lateral es el triángulo isósceles de

base a ) podemos aplicar el teorema de Pitágoras para calcular la altura de la pirámide

mb

hh ap 121445132

22

2

2

y calculamos el volumen

33 24,02403

12.10.6

3

.damm

hAV

pB

PRUEBA 7

1.-a) Primero x2 – 9 0 lo que conduce a x – 3 ó x 3 y por otra parte hay que hallar

las x que hacen que el denominador sea igual a cero por eso resolveremos la ecuación

siguiente 0192 xx donde 91 2 xx y al elevar al cuadrado ambos miembros

x2 – 2x + 1 = x2 – 9 y llegamos a que x = 5. El dominio de f está dado por

Dom f={x reales : x – 3 ó x 3 ; x 5}.

El dominio de la función g es fácil de hallar Dom g={x reales : x – 3 ó x 3}

El dominio común es la intersección de ambos dominios, o sea, Dom f.

Page 12: PRUEBA 1 - math.cubava.cumath.cubava.cu/files/2015/05/solucionario-libro-jacinto.pdf · g 9 2x 3 x 2 y ahora podemos ... llegamos a la conclusión parcial y = 8 ó y = 1 para los

12

b) Trabajando directamente en f(x) utilizando la conjugada del denominador

222

2

2

2

2

2

)9()1(

]91)[5)(5(2

91

91.

91

502

xx

xxxx

xx

xx

xx

x

si trabajamos ahora con el denominador nos queda

x2 – 2x + 1 – x2 + 9 = – 2 x + 10 = – 2 (x – 5) que se puede simplificar con el numerador y

llegar a la expresión, sin denominador, siguiente

9)5(54]9)1).[(5( 222 xxxxxxx

que es idénticamente igual a la función g(x) . Queda probada la identidad por tanto f(x)=g(x)

para todos los elementos del dominio común.

2.- Hay otras maneras de resolver esta ecuación. Tomaremos la más sencilla. Preparando

27 = 33 ; 243 = 35 podemos escribir la ecuación como:

sen2x ( 3 log 3 ) + cos2x ( log 3 ) + cos x ( 5 log 3 ) = 0 y podemos entonces extraer factor

común log 3 quedando ( log 3 )( 3sen2x + cos2x + 5 cos x ) = 0 y como tenemos que

log 3 0 solo nos queda 3 sen2x + cos2x + 5cos x = 0 donde haremos ahora la

sustitución sen2x = 1 – cos2x y obtenemos la ecuación cuadrática más sencilla y ordenada

2cos2x – 5 cos x – 3 = 0 que se puede descomponer mediante (2cos x + 1)( cos x – 3 ) = 0

donde cos x = – 1/2 ó cos x = 3 (imposible) por lo que únicamente se puede hallar la

solución x = 2/3 debido al intervalo dado al inicio.

3.-Es muy fácil probar que el triángulo AGD es rectángulo en G y ahora los tres triángulos

dados en la figura son rectángulos. Como ABC y EFB tienen un ángulo común entonces

son triángulos semejantes y lo mismo ocurre con AGD y ABC por lo que podemos, por

transitividad, expresar que ABCEFBAGD.

Podemos poner que DG

FB

AG

EF (lados proporcionales) y

calcular cmEF

FBDGDG 14

10

7.20. y también

BC

FB

AC

EF para

calcular cmEF

FBACBC 21

10

7.30. y ahora podemos plantear la resta de

áreas y calcular

24,1140)35140(3152

10.7

2

14.20

2

21.30)( dmAAAAp BEFAGDABC

Page 13: PRUEBA 1 - math.cubava.cumath.cubava.cu/files/2015/05/solucionario-libro-jacinto.pdf · g 9 2x 3 x 2 y ahora podemos ... llegamos a la conclusión parcial y = 8 ó y = 1 para los

13

4.-El área de la sala expresada en metros está dada por AS = x ( x + 4 ) ; el área de cada

mosaico pequeño, expresada en centímetros, está dada por AM = y2 y de cada mosaico

grande será AG = ( y + 5 )2 , pero el problema nos lleva a proponer la primera ecuación entre

las cantidades de mosaicos AS = 1 500 y2 = 960 ( y + 5 )2 de donde podemos, haciendo

operaciones y transformaciones, obtener la ecuación 9y2 – 160y – 40 = 0 que se descompone

en los factores ( 9y + 2 )( y – 20 ) = 0 con única posibilidad y = 20 cm.

Podemos ahora calcular el área de la sala pues conocemos el área de cada mosaico pequeño

AM = 202 = 400 cm2 de los que se necesitan 1 500 . 400 cm2 = 600 000 cm2 , o sea, la sala

tiene AS = 60 m2 y entonces vamos a la primera ecuación planteada que nos lleva a

x ( x + 4 ) = 60 y que se transforma en x2 + 4x – 60 = 0 donde x = 6 ó x = – 10

(esta última imposible) . El perímetro de la sala P = 2 ( 6 + 10 ) = 32 m .

5.-Observemos primero que DB es la proyección de EB sobre (considerando al plano base

de la pirámide como ). La arista lateral mayor es necesariamente EB pues tiene por

proyección a DB que es la diagonal mayor del trapecio rectángulo base. Podemos decidir que

cmDE 4,14 pues se opone al EBD=300 en el triángulo rectángulo EBD ( DBED en D )

y también podemos calcular

cmDEDB 2573,1.45,143. pues DB se opone al DEB=600 . Como el triángulo

ABD es rectángulo en A podemos calcular cmADDBAB 204001525 2222

por el teorema de Pitágoras.

Por la información inicial ahora decidimos que cmDC 10 y esto nos permite calcular el

volumen de la pirámide

33 1,1108445,14.15.2

1020.

3

1..

3

1dmcmhAV B

En el triángulo ADE (rectángulo en D pues ED es la altura) podemos calcular, aplicando el

teorema de Pitágoras,

cmAEADAE 2186,2025,43525,2102255,1415 2222

El triángulo ABE es también rectángulo en A pues en este punto se cumple el teorema de las

tres perpendiculares ya que AD es la proyección de AE y ABAD entonces el área del

triángulo ABE (cara lateral) es 22 1,2210

2

20.21

2

.dmcm

ABAEA

PRUEBA 8

1.- a) Está claro que la expresión M no puede estar definida para x = – 1 ni para x = 0 pero

hay que analizar también que cuando 01

1

x

x

x

x tampoco está definida lo que nos

conduce, luego de eliminar los denominadores, a la ecuación 2x2 – 1 =0 que nos entrega

soluciones x = 2

2 para los que no puede estar definida.

Page 14: PRUEBA 1 - math.cubava.cumath.cubava.cu/files/2015/05/solucionario-libro-jacinto.pdf · g 9 2x 3 x 2 y ahora podemos ... llegamos a la conclusión parcial y = 8 ó y = 1 para los

14

b) Al efectuar en el numerador y en el denominador llegamos a la expresión

siguiente12

1

)1(

12

)1(

1

22

x

xx

x

xxM que al evaluar para x = 1/2 nos queda M = – 2.

2.-Se puede llevar el miembro izquierdo a la misma base y luego de sumar los exponentes,

pues tenemos un producto con igual base, llegar a la desigualdad

3x

3

2x

2

1x

1

55

y como la base es mayor que 1 aparece la desigualdad

3

3

2

2

1

1

xxx con los respectivos exponentes, que comparando con cero, nos

queda 03

3

2

2

1

1

xxx y operando con el mínimo común denominador obtenemos

0)3)(2)(1(

)2)(1(3)3)(1(2)3)(2(

xxx

xxxxxx y que al final conduce a

0)3)(2)(1(

64

xxx

x donde analizaremos cuáles son los intervalos

+ – 3 – – 2 + – 3/2 – – 1 +

///////////////////OO/////////////////OO///////////////////

donde tomamos los intervalos x < – 3 ó – 2 < x < – 3/2 ó x > – 1 .

3.- Llegamos a la primera conclusión: El triángulo ABC es rectángulo en A pues se cumple el

recíproco del teorema de Pitágoras ya que 152 =122 +92 . El triángulo pequeño también es

rectángulo, pero en E, y como ambos tienen un ángulo común en el vértice C podemos

concluir que ABC DEC por tener dos ángulos respectivamente iguales.

Podemos hallar la razón de proporcionalidad 3

1

9

3

AB

DEk . El área del triángulo ABC se

puede calcular fácilmente 2542

12.9cmAT y el área del triángulo DEC se puede calcular

aplicando At=k2.AT . Entonces el área sombreada se calcula mediante

2

2

4854.9

8.

9

11.

3

1cmAAAAAA TTTtTS

Page 15: PRUEBA 1 - math.cubava.cumath.cubava.cu/files/2015/05/solucionario-libro-jacinto.pdf · g 9 2x 3 x 2 y ahora podemos ... llegamos a la conclusión parcial y = 8 ó y = 1 para los

15

4.-Consideramos que la longitud del frente es x ; la del lado está dada por y .

Llegaremos entonces a escribir el sistema 2,40 x + 2,10 (x + 2y) = 589,50

2 (x + y).2,40 = 648,00

Es muy bueno operar y eliminar todas las comas, o sea, llevarlo todo a números enteros.

Nos queda el sistema 450 x + 420 y = 58950

480 x + 480 y = 64800

que al simplificarlo convenientemente nos lleva a 15 x + 14 y = 1965

x + y = 135

que se puede resolver multiplicando la segunda ecuación por – 15 y sumarla con la primera

donde obtenemos y = 60 para luego calcular x = 75.

Lo que nos queda es solamente calcular:

$2,40 . 2 . 60 = $288,00

$2,10 . 2 . 75 = $315,00 Costará cercarlo $603,00

5.-Si consideramos que la altura de la pirámide está dada por x , entonces la arista de la base

se podrá escribir como x + 4 (trabajando en cm). En la fórmula del volumen se pueden

hacer las sustituciones pertinentes y aparece la ecuación que necesitamos. ( 0,2dm3=200cm3)

3

.)4(

3

. 2 xxhAV B

, o sea, 200 . 3 = ( x + 4 )2 . x

donde 600 = (x² + 8x + 16) . x que nos conduce a una ecuación de tercer grado dada por

x³ + 8x² + 16x – 600 = 0 y que admite la descomposición en factores, aplicando la división

sintética, ( x – 6 ) ( x2 + 14x + 100 ) = 0 . Llegamos a que únicamente x = 6 pues

x2 + 14x + 100 = 0 no tiene soluciones reales ya que D=b2 – 4ac=196 – 400

donde D = – 204 < 0 . Ya podemos decidir que h = 6 cm y que a = 10 cm.

El perímetro de su base es entonces P = 4 .10 = 40 cm .

PRUEBA 9

1.-La ecuación dada, después de efectuar la operación indicada que está en el argumento de la

raíz, toma la forma 665.215 xx donde se puede hacer el cambio de variable

y = 5x y proceder a elevar al cuadrado ambos miembros de la ecuación

3662)62).(1(21 yyyy para llegar a yyy 331)62)(1(2 que

elevamos de nuevo al cuadrado 4(2y² + 4y – 6 ) =961 – 186y +9y² para obtener la ecuación

reducida y ordenada y2 – 202y + 985 = 0 con D=b2 – 4ac=( – 202 )2 – 4 . 985

donde D = 40804 – 3940 =36864 >0 (tiene dos soluciones reales) para calcular

19236864 D ; las soluciones para y son 2

192202

2

192)202(2,1

y

donde y1 = 197 ó y2 = 5 ; el caso 5x = 197 no nos ofrece soluciones naturales pero el caso

5x = 5 nos aporta la posible solución x=1 que al comprobar verificamos que es la solución

pedida.

2.-a) Se puede probar que MED=CDN=ABN=FAM, todos los casos por tener

respectivamente iguales dos lados y el ángulo comprendido; por elementos correspondientes

en triángulos iguales se cumple que AMNADNMD por lo que AMDN es un rombo

(tiene todos sus lados iguales).

Page 16: PRUEBA 1 - math.cubava.cumath.cubava.cu/files/2015/05/solucionario-libro-jacinto.pdf · g 9 2x 3 x 2 y ahora podemos ... llegamos a la conclusión parcial y = 8 ó y = 1 para los

16

b)Se puede probar que no es un cuadrado utilizando varios caminos pero el más sencillo es

trazar AD y MN (diagonales del rombo) y verificar fácilmente que no son iguales al trazar la

circunferencia circunscrita al hexágono regular pues AD es el diámetro de la circunferencia y

MN no lo es (es menor). AMDN no es un cuadrado.

Otra vía para el inciso b :

MOD = DON = NOA = AOM (rectángulos y tienen iguales h y c)

Luego a R, por apotema y radio. Por tanto los triángulos no son isósceles y por ello AMDN

no es un cuadrado.

c) Conociendo el área del hexágono regular se puede calcular la medida de su lado ¨a¨

trabajando directamente en la fórmula

166,5 = 6 . ADOE ( O es el centro de la circunferencia y DOE equilátero)

4

3..65,166

2a donde 01,6473,1.37

3.6

3.5,166.4

3.6

5,166.42 a y podemos afirmar

que a = 8 cm

AAMDN = AABCDEF – 4 . AMED=166,5 – 4 . (1/2) .4.8.sen 1200

AAMDN = 166,5 – 64.2

3 = 166,5 – 32 . 1,73 = 111,14= 111 cm2.

3.-En principio se tiene que cumplir x3 – 64x 0 que nos conduce a la factorización

x ( x + 8 )( x – 8 ) 0 y nos aporta el siguiente análisis de intervalos:

– – 8 + 0 – 8 +

////////////////////////////////////////////////////////

al resolver nos da – 8 x 0 ó x 8 .

Por otra parte 02

76

x

x para que esté definida la función logarítmica

+ – 2 – –7/6 +

//////////////////////////OO////////////////////////////////////////////////

que al resolverla nos queda x < – 2 ó x > – 7 /6 .

Al intersectar ambos resultados llegamos a – 8 x < – 2 ó – 7 /6 < x 0 ó x 8.

Pero no podemos olvidarnos que el denominador no puede ser igual a cero por lo que debemos

buscar, si existe, cualquier valor real que lo convierta en cero

012

76log 3

x

x y 1

2

76log 3

x

x de donde obtenemos 3

2

76

x

x ;

ecuación que, al eliminar el denominador y resolverla, nos aporta x = – 1/3 por tanto del

dominio hay que excluirlo escribiendo x – 1/3 .

Page 17: PRUEBA 1 - math.cubava.cumath.cubava.cu/files/2015/05/solucionario-libro-jacinto.pdf · g 9 2x 3 x 2 y ahora podemos ... llegamos a la conclusión parcial y = 8 ó y = 1 para los

17

4.- Es un típico problema de cálculo. 18 . $14,00 = $252,00 (costo)

Como se pierde el 10% del costo hubo que vender esos 10 libros deteriorados al 90% del

precio de costo, veamos, 90%.$14,00 = $12,60 ; y como son 10 libros entonces por ellos se

pudo obtener solamente $126,00 .

Como el negocio aportó $32,00 de ganancia, al final se recaudó lo invertido más la ganancia

$252,00 + $32,00 = $284,00 , pero ya habíamos vendido los libros deteriorados, entonces

$284,00 – $126,00 = $158,00 que es lo que aportaron los 8 libros restantes, y entonces,

$158,00 : 8 = $19,75 . Cada uno de estos 8 libros se vendió a un precio de $19,75 .

5.- Consideremos que el vértice está dado por S y que el diámetro es AC ,vemos que se

forma un triángulo rectángulo pequeño OBC y que cmAC 10 . Vemos ahora

que cmOC 5 , entonces, como cmOB 4 tenemos que cmBC 3 (trío de números

pitagóricos) . Se puede calcular SB aplicando el teorema de la altura en el triángulo

rectángulo SOC ( ACSO ) ; SBBCOB .2

donde 3

16

3

422

BC

OBSB y ahora ya

podemos calcular la generatriz del cono sumando los segmentos en la hipotenusa del triángulo

rectángulo SOC.

.33,83

25

3

9163

3

16cmBCSBSCg

También tenemos que calcular la altura h del cono, que se puede hacer aplicando el teorema

de Pitágoras en el triángulo rectángulo SOC

cmOCSCSOh 66,63

20

9

40025

9

6255

3

25 2

222

El volumen es 3322

17,03,1743

66,6.5.14,3

3

..

3

.dmcm

hrhAV B

.

El área lateral es AL = .r.g = 3,14 . 5 . 8,33 = 130,78cm² = 1,3 dm

PRUEBA 10

1.- El miembro derecho tal vez nos pueda impresionar, pero, aplicando potencias y

propiedades de los logaritmos podemos descubrir su verdadero valor:

03

2

3

2

3

2

2log3

2log2

3

2

2log

2log

3

2

8log

4log3

2

Por lo que la ecuación puede escribirse más simplificada y con un solo logaritmo

log [ tan x . cos x . ( sen x + 1,5 ) ] = 0 . Podemos sustituir tan x y luego simplificar

cos x para llegar a log [ sen x . ( sen x + 1,5 ) ] = 0 donde necesariamente se cumple

sen x ( sen x + 1,5 ) = 1 que nos lleva a la ecuación cuadrática sen2 x + 1,5senx –1 =0

y que la podemos escribir como 2sen2x + 3sen x – 2 = 0 para no trabajar con valores

fraccionarios y admite la descomposición ( 2sen x – 1 )( sen x + 2 ) = 0 donde obtenemos

sen x = 1/2 ó sen x = – 2 (imposible).

Page 18: PRUEBA 1 - math.cubava.cumath.cubava.cu/files/2015/05/solucionario-libro-jacinto.pdf · g 9 2x 3 x 2 y ahora podemos ... llegamos a la conclusión parcial y = 8 ó y = 1 para los

18

En el intervalo que nos dan son posibles las soluciones x = /6 ó x = 5/6 pero podemos

apreciar rápidamente que x = 5/6 no está en el dominio de la ecuación original .

2.- Hay que hacer construcciones auxiliares para poder descubrir relaciones que se cumplen en

este ejercicio. Trazaremos el radio ADOM que corta a AD en P y a

BC en el punto N , trazaremos también el radio OD .

Surge un resultado de gran interés, P y N son puntos medios de las cuerdas AD y

BC respectivamente, por lo que OM es mediatriz de BC y bisectriz del BOC.

El triángulo ONC es rectángulo en N y como NOC=600 (bisectriz) entonces NCO=300

(ángulos complementarios), donde resulta que cmON 3 (se opone al ángulo de 300

y cmOC 6 ) y entonces cmNC 33 (se opone al ángulo de 600) por lo que cmBC 36

(base mayor del trapecio).

En el triángulo rectángulo OPD tenemos que cmNPONOP 5 y podemos calcular

cmOPODPD 11253656 2222

y cmAD 112 .

2.2

11236

3

6.14,3.

23

. 22

NP

ADBCOCAAA TSECS

266,2004,177,37)64,64,10(7,37)32,3.273,1.6(14,3.12 cmAS

AS= 21 cm2.

3.- Una simple evaluación del límite para x = 1 nos conduce a una forma indeterminada, por

lo que debemos trabajar con la expresión fraccionaria realizando transformaciones.

)1)(1(

)2)(1(

)1)(1(

31

)1)(1(

3

1

1

1

3

1

122

2

23

xxx

xx

xxx

xx

xxxxxx

Simplificamos (x –1) y entonces podemos calcular el límite,

13

3

111

21

1

221

xx

xlimx

4.- (Edgar Allan Poe, escritor romántico norteamericano, siglo XIX, padre del género

policiaco, célebre por sus narraciones de horror y misterio. “El corazón acusador”, “La

máscara de la muerte roja”, “Entierro prematuro”, entre otras)

Podemos designar el precio de las copias del tomo I por x

el precio de las copias del tomo II por y

Como los descuentos son de 15% y 10% respectivamente, ahora los precios disminuyen a

85% y 90% respectivamente y

obtenemos el sistema de ecuaciones 60 x + 75 y = 405

60.85%.x + 75.90%.y = 355,50

que se simplifica 4 x + 5 y = 27

51 x + 67,5 y = 355,5 y que al resolverlo nos lleva a las soluciones x=3 ;

y=3 de lo que se deduce que ambas copias tienen igual precio.

Se puede comprar la misma cantidad de cualquiera de las dos copias con el precio original.

Page 19: PRUEBA 1 - math.cubava.cumath.cubava.cu/files/2015/05/solucionario-libro-jacinto.pdf · g 9 2x 3 x 2 y ahora podemos ... llegamos a la conclusión parcial y = 8 ó y = 1 para los

19

5.- Si designamos la arista de la base de la pirámide por a y la altura por h podemos

escribir la diagonal de la base como 2ad y plantear un sistema de ecuaciones muy

interesante a partir del área de la sección transversal dada y el teorema de Pitágoras en el

triángulo de apoyo de la arista lateral L = 5,0 m.

2

.212

ha ; 2

2

2 52

2

ah

Se puede hacer en la primera una pequeña transformación para sustituirla directamente en la

segunda ecuación

2

212 a

h y entonces 25

122

2

hh para llegar a h4 – 25h2 + 144 = 0 que se

puede descomponer en factores ( h2 – 9 )( h2 – 16 ) = 0 y nos entrega cuatro posibilidades

(pero debemos despreciar las negativas) h = 4 ó h = 3 de donde hallamos, sustituyendo

en la primera ecuación, a= 23 ó a = 24 respectivamente, lo que nos lleva a deducir que

existen dos pirámides diferentes que cumplen las condiciones establecidas en el enunciado del

ejercicio.

32

1 243

4.2.9

3

4.)23(

3

.m

hAV B

32

2 323

3.2.16

3

3.)24(mV .

PRUEBA 11

1.- La ecuación parece inicialmente muy complicada pero no es así.

El primer detalle es percatarnos de que la base es la misma, x > 0 ; x 1 .

Aplicando las propiedades de los logaritmos, en este caso para convertir a un solo logaritmo el

miembro izquierdo, xxx x

x 2)]).1.2[(log 42 que ahora lo llevamos a potencia xx xxx 242 ).1.2( pero podemos efectuar en el primer miembro y obtenemos

xx xxx 242.2 y que podemos trasponer y ordenar adecuadamente 0.2 422 xxx xx

para darnos cuenta que es un trinomio cuadrado perfecto que se descompone fácilmente en

0)( 22 xx x (puede usted comprobarlo, si lo desea) y que admite únicamente xx – x2 = 0

donde xx = x2 por lo que x = 2 .

Otra vía :

La base es la misma, x > 0; x 1. La ecuación puede escribirse )2.(212log 2 xxx

x ,

pues 4log 4 xx. Luego 2xx – 2 – 1 = x2.(x – 2) , y de aquí organizando la ecuación de segundo

grado observamos que el trinomio es cuadrado perfecto, por lo que tenemos que xx – 2 – 1 = 0 ,

por lo que x = 2.

Page 20: PRUEBA 1 - math.cubava.cumath.cubava.cu/files/2015/05/solucionario-libro-jacinto.pdf · g 9 2x 3 x 2 y ahora podemos ... llegamos a la conclusión parcial y = 8 ó y = 1 para los

20

2.- Si hacemos un buen dibujo de la situación que se nos plantea podemos percatarnos, al

unir los centros de las circunferencias C(A; r =3) , C(B; r = 2) y C(C; r = 1), de que se nos

forma un triángulo donde sus lados son precisamente la suma de los respectivos radios de las

circunferencias tomadas dos a dos. Además, el triángulo formado es rectángulo porque cumple

el recíproco del teorema de Pitágoras ya que 5² = 4² + 3², o sea que ACB = 900 . La longitud

de la cuerda que se forma, cuando unimos los puntos de tangencia en la circunferencia de

centro C con r = 1, se calcula fácilmente porque forma con los radios un triángulo rectángulo

isósceles, por lo que esa cuerda mide cmPQ 2 , habiendo considerado a P como el punto

de tangencia entre las circunferencias de centro en C y en B. Falta ahora calcular las

longitudes de QR y PR , que son segmentos (cuerdas de las circunferencias) que pertenecen a

triángulos isósceles pero no son triángulos rectángulos.

Aplicaremos la ley de los cosenos (o generalización del teorema de Pitágoras) en cada

triángulo, pero antes debemos tener presente que cosCBA = 3/5 y que por otra parte

cosCAB= 4/5 por lo que podemos plantear los siguientes cálculos:

PR 2 = 22 + 22 – 2 . 2 . 2 . cosCBA = 4 + 4 – 8 . (3/5) = 8 ( 1 – 3/5) = 8.(2/5) = 16/5

QR 2 = 32 + 32 – 2 . 3 . 3 . cosCAB = 9 + 9 – 18 . (4/5) = 18 (1 – 4 /5)= 18.(1/5) =18/5

PR = cm5

54

5

16 ; QR = cm

5

103

5

18

calculamos el perímetro PPQR=5

24,2.41,1.3

5

24,2.441,1

5

5.23

5

542

donde PPQR=1,41 + 1,79 + 1,9 = 5,1 cm

3.-Para que el conjunto imagen de la función f esté contenido en el conjunto imagen de la

función g tiene que cumplirse que – 1 f(x) 1 por lo que, sustituyendo la expresión de

f(x) nos 1²

16²21....

xx

xxqueda lo que nos conduce a tener que resolver dos

desigualdades por separado y luego intersectar sus resultados.

Por la izquierda Por la derecha

01²

16²2

xx

xx 01

²

16²2

xx

xx

²16²2

xx

xxxx 0

²

²16²2

xx

xxxx

162²3

xx

xx 0

²

16²

xx

x

0)1(

)2)(83(

xx

xx 0

)1(

)4)(4(

xx

xx

+ –8/3 – –1 + 0 – 2 +

/////////////////////////O//////////////O/////////////////////////

+ – 4 – –1 + 0 – 4 +

////////////////////////////////OO/////////////////////////////

Page 21: PRUEBA 1 - math.cubava.cumath.cubava.cu/files/2015/05/solucionario-libro-jacinto.pdf · g 9 2x 3 x 2 y ahora podemos ... llegamos a la conclusión parcial y = 8 ó y = 1 para los

21

Podemos apreciar que la intersección de los intervalos que debemos seleccionar en

cada recta numérica nos conduce finalmente al resultado siguiente:

– 4 x – 8/3 ó 2 x 4

4.- Se puede probar que ACDE es un trapecio (el punto B es común a los paralelogramos)

luego, 4,2.2

18DEAC

; o sea, 15 DEAC y como el perímetro es 22 , entonces

7 DCAE

2522

DCAE por dato

Despejando DCAE 7 en la primera ecuación y sustituyendo en la segunda nos queda

( 7 – DC )2+ DC 2 =25, ecuación que luego de operar, reducir y ordenar nos queda de la

siguiente forma DC 2 – 7 DC + 12 = 0 y se descompone

( DC – 4 )( DC – 3 ) = 0 donde aparecen las respuestas DC = 3 cm ó DC = 4 cm para

AE = 4 cm ó AE = 3 cm respectivamente.

5.- Conocemos que V = 120 m3 pero que aumentará o disminuirá según ocurran

deformaciones en los lados de la base del ortoedro. Considerando que el ancho de la base sea

a y el largo, b entonces el volumen es V = b .a .h.

Podemos plantear el sistema de ecuaciones ( b +2 ). a .h = 160

b.( a – 2 ). h= 20

Que se transforma b.a.h + 2.a.h = 160 donde 2.a.h = 40

b.a.h – 2.b.h = 20 donde – 2.b.h = – 100

y obtenemos que a.h = 20 y b.h = 50 .

Si sustituimos en la fórmula del volumen podemos calcular V = 120 = b.20 de la que

resulta b = 6 y podemos calcular también h = 3

25

6

5050

b .

Calculamos a = 12/5 en la misma fórmula del volumen.

P1 = 2.( 6 + 25/3 ) = 86/3 = 28,7 = 29 cm

P2 = 2.( 12/5 + 25/3 ) = 21,4 = 21 cm.

PRUEBA 12

1.- Podemos aislar la raíz para que se elimine cuando elevemos al cuadrado ambos miembros

de la ecuación xx 2sen212cos2

xxx 2sen2sen4412cos 22 y hacemos la sustitución cos22x = 1 – sen22x de manera

que al reducir y ordenar llegamos a la ecuación cuadrática mucho más sencilla

sen22x – 2 sen 2x + 1 = 0 que se factoriza ( sen 2x – 1 )2 = 0 donde resulta que

sen 2x = 1 lo que nos aporta soluciones parciales 2x = /2 + 2k ; k entero y para hallar x

tenemos que dividir ambos miembros por 2.

Nos queda x = /4 + k ; k entero.

Page 22: PRUEBA 1 - math.cubava.cumath.cubava.cu/files/2015/05/solucionario-libro-jacinto.pdf · g 9 2x 3 x 2 y ahora podemos ... llegamos a la conclusión parcial y = 8 ó y = 1 para los

22

2.-Inicialmente podemos asegurar que la expresión M no está definida para los siguientes

valores x = 1 ; x = 0 ; x = – 2 ; x = – 3 pero para que pueda operarse con la raíz cuadrada

tiene que ocurrir M 0 .

Pero la expresión M puede tal vez darse más simplificada ya que tiene posibilidades de

realizar en ella factorizaciones y operaciones;

M =

)2)(1(

)3)(2)(1(.

)1)(1(.

31 xx

xxx

x

xx

x

x

x

x

= )2)(1(22

)3)(2(.

)1)(1(.

)3)(1(

)2(2

xx

x

xx

x

xx

xx

xx tenemos que resolver la

desigualdad 2 ( x – 1 )( x – 2 ) 0 que se analiza con los intervalos

+ 1 – 2 +

//////////////////////////////////

Está definida para x reales con x < 1 ó x 2 con x – 1 ; x – 2 ; x – 3 ; x 0

(no olvidar que desde el comienzo x 1)

3.- De inicio trazamos la sugerente construcción auxiliar TO' . Como se conoce la longitud de

la circunferencia mayor, podemos calcular su diámetro a partir de la fórmula L = d donde

ABcmL

d 1014,3

4,31

; ocurre que BCTO ' (radio en el punto de tangencia) y

BCAC (teorema de Tales, ACB inscrito sobre el diámetro)

Podemos decidir que ABC O TB pues ambos triángulos son rectángulos y tienen el

ABC común (tienen dos ángulos respectivamente iguales).

cmBC 86436100610 22 (teorema de Pitágoras)

cmCTBCBT 538 donde TO

AC

BT

BC

' lados proporcionales en los triángulos

semejantes para obtener 4

15

8

6.5.'

BC

ACBTTO y ahora podemos calcular el área del

círculo menor.

2

2

2 2,4415,4416

225.14,3

4

15.14,3)'.( cmTOAO

4.- Consideremos por ''x'' al número de días

''y'' jornal diario

Tendremos que x y = 84 ; ( x + 2 ) ( y – 1 ) = 84

x y – x + 2y – 2 = 84

2 y – x = 2

y despejando 2

2

xy

Page 23: PRUEBA 1 - math.cubava.cumath.cubava.cu/files/2015/05/solucionario-libro-jacinto.pdf · g 9 2x 3 x 2 y ahora podemos ... llegamos a la conclusión parcial y = 8 ó y = 1 para los

23

Sustituimos en la primera ecuación 842

2.

xx donde x2 + 2x = 168 ,

o sea x2 + 2x – 168 = 0 que se descompone ( x + 14 )( x – 12 ) = 0

x = 12 ó x = – 14 (imposible) Y entonces el jornal diario es de $7,00. Ahora dividimos

$210,00 : 3 = $70,00 (cada hombre) y al calcular $70,00 : $7,00 = 10 nos da la cantidad de

días . Deberán trabajar 10 días.

5.- Intentemos inicialmente dar al volumen de esta pirámide recta de base hexagonal regular

una expresión bien simplificada 33

2.4

3.6

3

. 2

2

a

a

hAV B

lo que nos indica que

debemos únicamente calcular la medida de a (arista de la base, que en este caso es igual a

cualquiera de los lados de los 6 triángulos equiláteros que se forman en la misma).

En cualquiera de los triángulos de apoyo de las aristas laterales, que son triángulos rectángulos

con un ángulo de 600 , podemos escribir la relación siguiente: aa

htan

2600 y entonces

calcular 3

32

3

2

60

20

tan

a .

Hay que trabajar en los triángulos rectángulos de apoyo de las caras laterales; estos triángulos

tienen, en la base de la pirámide, un cateto que es la altura de los triángulos equiláteros y que

llamaremos ap y que se puede expresar como: 12

3.3

32

2

3.

a

a p . Ahora podemos

aplicar el teorema de Pitágoras en el triángulo de apoyo para calcular la altura de la cara lateral

que es un triángulo isósceles 512 2222 pT ahh

Podemos calcular el volumen 3

2

2 3,273,1.3

43.

9

3.43.

3

323 cmaV

Y el área lateral AL= 28,775,724,2.73,1.25.3

32.3

2

..6 cm

ha T

PRUEBA 13

1.- La ecuación original puede escribirse, haciendo transformaciones con las propiedades de

las operaciones con potencias, de la manera siguiente:3

333

9

9 sensen

sen xx

x

pero que

puede llevarse todo a la misma base mediante 3

333

9

3 sensen

sen2 xx

x

Page 24: PRUEBA 1 - math.cubava.cumath.cubava.cu/files/2015/05/solucionario-libro-jacinto.pdf · g 9 2x 3 x 2 y ahora podemos ... llegamos a la conclusión parcial y = 8 ó y = 1 para los

24

y se pueden eliminar los denominadores multiplicando toda la expresión por 9,

de manera que

32senx + 27 = 9.3senx + 3. 3senx ; o sea, la ecuación toma la forma 32senx – 12 . 3senx + 27 = 0

que se descompone en

( 3senx – 9 )(3senx – 3 ) = 0 y nos entrega posibilidades 3senx = 9 ó 3senx = 3 donde la primera

nos da senx = 2 (imposible) y la segunda senx = 1 que llegamos a obtener x = /2 en el

intervalo que se propone al inicio.

2.- Vemos de inicio que ABC es rectángulo en C por el recíproco del teorema de Pitágoras

pues 302 = 242 + 182 . Trazamos la construcción auxiliar rOT por lo que ETO es

rectángulo en T (radio en el punto de tangencia). Podemos decidir además que

ABC ETO pues ambos son rectángulos y ABC =TEO (alternos entre las paralelas

BC y EO ) tienen dos ángulos respectivamente iguales.

Por lados proporcionales podemos calcular 5

2

30

12

AB

OEk y a partir de este resultado

calcular 6,924.5

2. ACkOT y también 2,718.

5

2. BCkTE y ahora ya tenemos

datos para trabajar en el trapecio (que se puede probar que es un trapecio rectángulo debido a

los radios perpendiculares en los puntos de tangencia, igual que se puede probar que TODB es

un cuadrado)

OTTEBTTEBE = 7,2 + 9,6 = 16,8 cm y entonces llegamos a:

22 3,17,1266,9.

2

4,266,9.

2

6,98,16.

2dmcmBD

ODBEAT

3.- La primera desigualdad que aparece es 04

72

x

x pues es el argumento del logaritmo, que

se factoriza mediante la expresión siguiente 0)2)(2(

7:

xx

x y luego, como la base es

mayor que 1 podemos escribir 24

72

x

x

por lo que tenemos que considerar en este caso las dos desigualdades y después hacer la

intersección de sus resultados. En la última desigualdad comparamos con cero

024

72

x

x y al operar con el mínimo denominador común obtenemos

04

8272

2

x

xx que al reducir, ordenar y multiplicar ambos miembros por – 1 para

facilitar el trabajo de descomposición en factores (cambiamos el sentido de la desigualdad)

nos ofrece la expresión 04

122

2

x

xx que se factoriza completamente 0

)2)(2(

)1)(12(

xx

xx

Page 25: PRUEBA 1 - math.cubava.cumath.cubava.cu/files/2015/05/solucionario-libro-jacinto.pdf · g 9 2x 3 x 2 y ahora podemos ... llegamos a la conclusión parcial y = 8 ó y = 1 para los

25

– –2 + 2 – 7 +

De la primera O/////////////////////////////////////OO////////////////

+ –2 – –½ + 1 – 2 +

De la segunda ///////////////////O/////////////O/////////////////////////////////////

Se cumple para las x reales con – ½ x 1 ó x 7.

4.- Como el triángulo ABC es equilátero y M es punto medio de AB entonces se cumple que

BCBM ; además BM es la proyección de NB sobre el plano de la base

del prisma (lo consideramos ) y también NM ya que N es punto medio de ED .

En el triángulo rectángulo MBN el MBN=600 (ángulos complementarios) y entonces

MB = 6 = hE (se opone al ángulo de 300) y MN = 36 (se opone al ángulo de 600).

Como AC PR y ACMB entonces PRMB y como MB es proyección de NB

tenemos que en B se cumple el teorema de las tres perpendiculares y por ello tendremos que

PRNB . El triángulo, que se forma al trazar NR , o sea, NBR es rectángulo en B y

podemos calcular fácilmente que NR = 13cm pues forma un trío pitagórico con NB = 12cm

y BR = 5cm . Para calcular el volumen aplicaremos, para calcular el área de la base, la

fórmula del área del triángulo equilátero en función de su altura (que ya la hemos calculado y

representado por hE). 33

22

22,021636.3

36.

3

3. dmcmh

hhAV E

B ; NR = 13 cm .

5.- Consideremos las variables, cantidad de libros : x

precio por libro : y

por lo que podemos plantear las ecuaciones siguientes x . y = 24

( x + 3 )( y – 1,80) = 24

y podemos escribir x y = x y – 1,8 y + 3 y – 5,4

donde 1,8 x – 3 y = – 5,4

3 x – 5 y = – 9 podemos despejar en la primera y = x

24

y sustituir 3 x – 5 ( x

24) = – 9 ahora eliminamos el denominador y ordenamos

3 x2 + 9x – 120 = 0 que se simplifica x2 + 3 x – 40 = 0 ; se puede descomponer

en factores ( x + 8 )( x – 5 ) = 0 y se obtienen x = 5 ó x = – 8 que es imposible. Ahora hay

que responder con mucho cuidado! La hermana compró 8 libros, a un precio de

$24,00 : 8 = $3,00 cada uno, entonces los 5 libros, al precio que los pagó la hermana, le

habrían costado 5. $3,00 = $15,00.

Page 26: PRUEBA 1 - math.cubava.cumath.cubava.cu/files/2015/05/solucionario-libro-jacinto.pdf · g 9 2x 3 x 2 y ahora podemos ... llegamos a la conclusión parcial y = 8 ó y = 1 para los

26

PRUEBA 14

1.- Calcularemos el valor numérico de log40,25=log4( 1/4 ) =log4 4 – 1= (– 1) log44 = – 1 .

Entonces la desigualdad toma la forma más sencilla 13011

2092

2

xx

xx que primero se

puede dividir en ambos miembros por – 1 para mejorar la factorización en el denominador

(hay que cambiar el sentido de la desigualdad) y luego al comparar con cero tendremos

013011

2092

2

xx

xx que, después de realizar la operación indicada, trabajando con el

mínimo denominador común, reducir y ordenar, obtenemos la expresión factorizada

0)5)(6(

)52(10

xx

x en la que se analizan los intervalos

– 5/2 + 5 – 6 +

////////////////OO////////////////////////

Se cumple para los x reales con 5/2 x < 5 ó x > 6 .

2.- En el triángulo ABC podemos aplicar el teorema de la altura DBADCD .2

y calcular

49

622

DB

CDAD y ahora podemos calcular AC en el triángulo rectángulo ACD,

5246 22 AC y para calcular el área del rombo podemos aplicar la fórmula

2022

4598,4473,1.262

3.5260sen.)52(sen. cmEACACAR

3.- Podemos escribir la ecuación como 4 sen2x + (2 sen x.cos x)2 = 3 que podemos llevar

a 4 sen2x + 4 sen2x .cos2x – 3 = 0 y sustituyendo cos2x nos queda con una sola función

trigonométrica 4 sen2x + 4 sen2x ( 1 – sen2x ) – 3 = 0 . Si multiplicamos, reducimos y

ordenamos la ecuación convenientemente obtenemos una ecuación de cuarto grado

4 sen4x – 8 sen2x + 3 = 0 que se descompone ( 2sen2x – 1 )( 2sen2x – 3 )=0 donde sen2x = 2

1

ó sen2x = 2

3 (imposible porque la raíz cuadrada de

2

3 es mayor que 1) y finalmente

sen x = 2

2 para hallar x = /4 ó x = 3/4 en el intervalo dado.

4.-Tomaremos como variable para cada una de las chicas la primera letra de sus respectivos

nombres, lo que nos llevará a plantear un sistema de tres ecuaciones con tres incógnitas

x + y + z = 42 que se puede simplificar (I) x + y + z = 42

y + x = z – x (II) 2x + y – z = 0

x + y = 4

3 z (III) 4x + 4y – 3z = 0

Page 27: PRUEBA 1 - math.cubava.cumath.cubava.cu/files/2015/05/solucionario-libro-jacinto.pdf · g 9 2x 3 x 2 y ahora podemos ... llegamos a la conclusión parcial y = 8 ó y = 1 para los

27

Fácilmente, sumando ordenadamente las ecuaciones I y II, podemos llegar a la ecuación sin

la variable z : (IV) 3x + 2y = 42 . Si multiplicamos la ecuación I por 3 y la sumamos

ordenadamente con la ecuación III obtenemos otra ecuación, también sin la variable

z : (V) 7x + 7y = 126. Si resolvemos este último sistema de dos ecuaciones llegamos a que

x = 6 ; y = 12 para luego hallar z = 24. Las horas realizadas por Xiomara, Yaquelín y Zoe

fueron 6 , 12 y 24 en este orden .

5.- Con los datos que nos ofrece el ejercicio podemos aplicar el teorema de Pitágoras en el

triángulo rectángulo que forman la altura h = 2r con el radio y la generatriz

g2 = h2 + r2 donde 554)2(20 22222 rrrrrr y calcular el valor

numérico de 545

20r por tanto 58h para ahora calcular el volumen

3322

5,115003

5,4501

3

24,2.8.5.16.14,3

3

58.)54.(14,3

3

.

3

.dmcm

hrhAV B

PRUEBA 15

1.- a) La ecuación nos queda como 1)1(log 3 xx que puede expresarse como

31 xx y aislando convenientemente la raíz 13 xx para elevar al

cuadrado ambos miembros llegar a: x2 – 6x + 9 = x – 1 que al reducir y ordenar tenemos la

ecuación x2 – 7x + 10 = 0 que se descompone ( x – 5 )( x – 2 ) = 0 donde las posibles raíces

son x = 5 ó x = 2 . Comprobamos en la ecuación original y verificamos que x = 5 es la

única solución , x = 2 es una raíz falsa.

b) Atendiendo a que 4log104 podemos escribir 4sen

sen2cos2

x

xx donde eliminamos el

denominador a la vez que sustituimos cos 2x = 1 – 2 sen2x y llegamos a la ecuación

reducida y ordenada 6 sen2x – sen x –1 = 0 que admite la factorización siguiente

( 2sen x – 1 )( 3 senx + 1 ) = 0 donde sen x =2

1 ó sen x = –

3

1 (con esta última igualdad no

vamos a trabajar porque sus soluciones estarán fuera del intervalo dado al comienzo del

ejercicio) entonces x = /6 .

2.- a) OCrAC ( AC se opone al ángulo inscrito de 300) . Los triángulos ABC y OCN son

rectángulos en A y C respectivamente (teorema de Tales, ángulo inscrito sobre un diámetro, y

rOC perpendicular en el punto de tangencia C) además ACB=NOC (alternos entre

paralelas) por lo que ABC = NOC por tener respectivamente iguales dos ángulos y el lado

comprendido.

b)ONC= ABC = 300 (en un mismo triángulo o en triángulos iguales, a lados iguales

corresponden ángulos iguales).

Page 28: PRUEBA 1 - math.cubava.cumath.cubava.cu/files/2015/05/solucionario-libro-jacinto.pdf · g 9 2x 3 x 2 y ahora podemos ... llegamos a la conclusión parcial y = 8 ó y = 1 para los

28

c) 33AB se opone al ángulo de 600 y CNAB (elementos correspondientes)

también

ONBC 6 (hipotenusas de triángulos iguales). Podemos calcular el perímetro del

pentágono que nos piden.

P= OBONCNACAB 4,2273,1.61236123633333

P=22 cm

3.-Para que la función esté definida tiene que cumplirse que 01

45 24

x

xx

que se puede expresar como 01

)1)(4( 22

x

xx y factorizada completamente

01

)1)(1)(2)(2(

x

xxxx donde podemos simplificar (x – 1) considerando x 1

( x + 2 )( x – 2 )( x + 1 ) 0 que nos lleva a analizar los intervalos siguientes

– – 2 + – 1 – 2 +

///////////////////////////////////////////////////////////////////////

El dominio es Dom f = { x reales : x – 2 ó – 1 x 2 con x 1 }

4.- Podemos referirnos al ancho como la variable x, entonces el largo lo denotaremos como

x + 8. Podemos formar ecuaciones A = x ( x + 8 )

( x + 3 ) ( x + 8 + 3 ) = A + 57

Si sustituimos directamente la primera en la segunda nos queda una ecuación cuadrática en la

variable x : ( x + 3 ) ( x + 11 ) = x ( x + 8 ) + 57

que se transforma en una ecuación lineal 6 x = 24

y calculamos x = 4

Calculamos entonces el perímetro P = 2 ( 4 + 12 ) = 2 . 16 = 32 cm.

Page 29: PRUEBA 1 - math.cubava.cumath.cubava.cu/files/2015/05/solucionario-libro-jacinto.pdf · g 9 2x 3 x 2 y ahora podemos ... llegamos a la conclusión parcial y = 8 ó y = 1 para los

29

5.- Si el radio está dado por r, entonces la generatriz la podemos escribir como r + 2

En la fórmula para el cálculo del área lateral AL = r g podemos entonces llegar a la

ecuación 47,1 = 3,14 r ( r + 2 ) donde r ( r + 2 ) = 15

r2 + 2r – 15 = 0

( r + 5 )( r – 3 ) = 0

Tomaremos solamente r = 3 y en el triángulo rectángulo formado por la altura, el radio y la

generatriz podemos encontrar ahora que g = 5 donde necesariamente, por trío de números

pitagóricos, h = 4.

Calculamos el volumen 3322

3868,3712.14,33

4.3.14,3

3

..

3

.dmdm

hrhAV B

PRUEBA 16

1.- La ecuación dada puede transformarse en 2

1)2(log42log 7

2

7 xx , si aplicamos

de nuevo las propiedades de los logaritmos 2

1

2

42log

2

7

x

x y entonces

772

422

12

x

x donde podemos eliminar el denominador 7).2(42 2 xx

y elevamos al cuadrado ambos miembros 2 x2 – 4 = 7 ( x2 – 4 x + 4 )

para llegar a la ecuación 5 x2 – 28 x + 32 = 0 que se factoriza ( 5 x – 8 )( x – 4 ) = 0 y nos

ofrece posibles soluciones x = 4 ó x = 8/5 (no pertenece al dominio de la ecuación original).

Comprobamos y nos queda que la única solución es x = 4 .

2.- Trazamos la construcción auxiliar sugerente rOC donde CBOC (por ser el radio

perpendicular a la tangente en el punto de tangencia) cmrOAOC 30 .

El triángulo OAC es isósceles de base AC por lo que OAC=ACO (ángulos base) y

entonces AOC = 1200 (suma de ángulos interiores de un triángulo). El COB=600

(adyacente con el de 1200) y entonces CBO=300 (complementarios en el triángulo

rectángulo OCB). Podemos calcular que 330CB (se opone al ángulo de 600 ) y

entonces calcular el área rayada restando del área del triángulo rectángulo el área del sector

circular de 1200.

14,3.15073,1.4506

30.14,3

2

330.30

3

.

2

. 22

OCCBOC

AAA ST

22 1,35,3074715,778 dmcmA

3.-Propondremos una variable para designar la cantidad de metros que había inicialmente en el

Page 30: PRUEBA 1 - math.cubava.cumath.cubava.cu/files/2015/05/solucionario-libro-jacinto.pdf · g 9 2x 3 x 2 y ahora podemos ... llegamos a la conclusión parcial y = 8 ó y = 1 para los

30

rollo de alambre: x

Del texto del problema deducimos la ecuación en la variable x

75% . x + 30 + (1/6) x = x

(3/4) x + 30 + (1/6) x = x

9 x + 360 + 2 x = 12 x

donde obtenemos x = 360

La cantidad de alambre vendido fue (3/4). 360 + 30 = 270 + 30 = 300

Entonces 300 . $0,65 = $195,00. Importó el alambre vendido : $195,00.

4.-La expresión N puede calcularse numéricamente pues por partes tenemos:

tan 1350 = – tan 450 = – 1

sen 4500 = sen ( 3600 + 900 ) = sen 900 = 1

cos ( – 600 ) = cos 600 = ½

Entonces 14

4

5,0.8

1.51

N

a) La expresión M no admite los ángulos que indefinen a las funciones tangente y

cotangente, que son todos los ángulos axiales y casualmente, los ángulos que anulan el

denominador están contenidos en los que ya expresamos anteriormente, por lo que los valores

inadmisibles son x = k (/2) ; k entero.

Debemos probar que M = 1. Partiremos de la expresión M haciendo sustituciones.

xx

xxxxx

x

x

x

x

xx

xM

cossen2

sen2cos2cos42cossen2

cos

sen

sen

cos

cossen2

cos421

2222

xx

xxxx

xx

xxxx

cossen2

)sen(cos22cossen2

cossen2

sen2cos22cossen2 2222

1cossen2

cossen2

cossen2

22cossen2

xx

xx

xx

xx queda probado que M = N.

5.-Podemos comenzar calculando la arista a de la base de la pirámide en la fórmula para el

área lateral aaha

A a

L .2613..22

..4260 por lo que a = 10 cm.

En el triángulo rectángulo de apoyo de cualquiera de las caras laterales (triángulo rectángulo

pues la altura h es perpendicular a la base de la pirámide) el cateto en la base mide la mitad

de la longitud de a y la hipotenusa es 13ah , entonces h = 12 por ser un trío de números

pitagóricos.

Podemos calcular el volumen 33

22

4,04003

12.10

3

.

3

.dmcm

hahAV B

La altura es 12 cm.

Page 31: PRUEBA 1 - math.cubava.cumath.cubava.cu/files/2015/05/solucionario-libro-jacinto.pdf · g 9 2x 3 x 2 y ahora podemos ... llegamos a la conclusión parcial y = 8 ó y = 1 para los

31

PRUEBA 17

1.- Podemos, ya que conocemos el área del cuadrado, calcular la longitud del lado del mismo

A = 16 = a2 por lo que a = 4 cm. Se puede calcular rápidamente la suma de las áreas de los

dos triángulos pequeños ADG y EFB restando el área del cuadrado GFED del área del

trapecio ABED.

2

2 41620164.2

46cmAAA cTRAPT

Trazaremos el segmento CM (altura relativa a la base en el triángulo isósceles, entonces M es

el punto medio de AB y de GF ) que intersecta a DE en N (N también es punto medio pero

de DE ). Por el teorema de las transversales podemos plantear la proporción MB

NE

CM

CN si

consideramos que CN = x entonces 3

2

4

x

x donde 3 x = 2 (x+4) y hallamos que x = 8.

Ahora podemos calcular el área del triángulo isósceles que se forma sobre el cuadrado

2162

8.4

2

.cm

CNDEAI . El área de la región sombreada es entonces

AS = A2T + AI = 4 + 16 = 20 cm2. Hay otras vías si consideramos otros triángulos!

2.-Si conocemos el área lateral y tenemos la relación g = 2 r podemos fácilmente calcular el

valor de r en la fórmula AL = 157 = r g = 2 r2 = 2.3,14.r2 = 6,28.r2 donde despejamos

2528,6

1572 r y nos queda que solamente r = 5 cm

En el triángulo rectángulo que forman la altura del cono, la generatriz y el radio de la base,

como la generatriz es el doble del radio, podemos asegurar que el ángulo que forman la

generatriz y la altura es de 300, y el ángulo en la base del cono es de 600.

Podemos entonces decidir que la altura del cono mide cmh 35 porque se opone al ángulo

de 600 .

Calculamos el volumen 3

22

34,2263

73,1.125.14,3

3

35.5.14,3

3

..

3

.cm

hrhAV B

V = 226 cm3. El ángulo de inclinación de la generatriz con el círculo base es de 600.

3.- Convertiremos a la misma unidad V = 0,36 m3 = 360 dm3 . La base no es un cuadrado.

Como todas las frases en este ejercicio están referidas al ancho, vamos a tomar como única

variable al ancho del rectángulo base y declararla como a . Entonces en la fórmula del

volumen V = a.b.h = a ( a + 12 )( 3 a – 1 ) = 360.

Y nos conduce a la ecuación 3 a3 + 35 a2 – 12 a – 360 = 0.

Si aplicamos el método de la división sintética podemos llegar a la factorización

( a – 3 )( 3 a2 + 44 a + 120 ) = 0. El discriminante de la ecuación cuadrática está dado por

D = 442 – 4.3.120 = 1936 – 1440 = 496>0 que no tiene raíz cuadrada entera, por lo que la

única solución entera que tendremos es a = 3. Obtenemos b = 15 y h = 8.

AL = PB . h = 2 .( 15 + 3 ). 8 = 16 . 18 = 288 dm2 = 2,9 m2 .

Page 32: PRUEBA 1 - math.cubava.cumath.cubava.cu/files/2015/05/solucionario-libro-jacinto.pdf · g 9 2x 3 x 2 y ahora podemos ... llegamos a la conclusión parcial y = 8 ó y = 1 para los

32

4.-El sistema propuesto se puede inicialmente transformar en

log ( 2 y – x ) = 0

x2 – y2 = 208 donde podemos escribir 2 y – x = 1

x2 – y2 = 208

y despejamos x = 2 y – 1 para sustituir en la segunda ecuación ( 2 y – 1 )2 – y2 = 208

de la que podemos obtener 3 y2 – 4 y – 207 = 0 con D = b2 – 4 a c

D = (– 4 )2 – 4.3.( –207 ) = 16 + 2484 = 2500>0 (tiene dos soluciones reales)

Calculamos las posibles soluciones

6

504

3.2

2500)4(

22;1

a

Dby donde 9

6

541 y ó

3

23

6

462

y

No tomaremos la segunda solución porque en el enunciado nos piden solamente las soluciones

enteras. Para y = 9 obtenemos x = 17 que son las soluciones pedidas.

5.-Cuando se hace referencia a los números reales no negativos se está considerando que

x 0. En la desigualdad dada hacemos la comparación con cero y nos queda

0289

418222

2

xx

xx efectuamos la diferencia 0

89

16182418222

22

xx

xxxx

reducimos términos semejantes y obtenemos 089

20202

2

xx

x que se puede descomponer en

factores 0)1)(8(

)1)(1(20

xx

xx y al analizar los intervalos nos queda

+ – 8 – – 1 – 1 +

O/////////////////////////////O///////////////////O

pero de aquí solo tomaremos los que tienen la condición inicial del ejercicio, por lo que la

solución es x reales con 0 x < 1.

PRUEBA 18

1.- Comenzaremos con un primer análisis 2 – log4 x > 0 donde obtenemos que

log4 x < 2 y como la base es mayor que 1 entonces x < 42 por lo que x < 16.

Trabajando con la expresión original, como la base es mayor que 1 entonces podemos escribir

2 – log4 x 22 donde llegamos a log4 x –2 y finalmente, como la base es mayor que 1

entonces x 4–2. La respuesta final es 1616

1 x .

Page 33: PRUEBA 1 - math.cubava.cumath.cubava.cu/files/2015/05/solucionario-libro-jacinto.pdf · g 9 2x 3 x 2 y ahora podemos ... llegamos a la conclusión parcial y = 8 ó y = 1 para los

33

2.- En principio ADE = EDC (por la bisectriz) y además EDC = AED (alternos

entre las paralelas) por tanto ADE = AED (propiedad transitiva de la igualdad).

Entonces el triángulo ADE es isósceles de base DE y AEAD (lados que se oponen a

ángulos iguales en un mismo triángulo).

ADBEABAE 5,188,33,22 y ahora podemos calcular el perímetro

).(2 ADABP 2.( 22,3 + 18,5 ) = 2.40,8 = 81,6 cm.

3.- Si elevamos al cuadrado ambos miembros de la ecuación obtenemos

x

xx

sen

2cos1sen.2 2 donde

x

xx

sen

2cos1sen2 2 que eliminando el denominador

podemos llegar a – cos2x . senx = cos2x y que al final escribiremos como

cos2x + cos2x . senx = 0 y descomponemos mediante factor común

cos2x .( 1 + senx ) = 0 donde cos2x = 0 ó senx = –1

2x = (2k +1)./2 ó x = 3/2 (raíz extraña)

x = (2k +1)./4

Las soluciones, en el intervalo dado, son /4 ó 3/4, porque 5/4 y 7/4 también son

raíces extrañas.

4.- Consideremos la base del triángulo isósceles como b y los lados como a.

El perímetro se puede escribir como la ecuación 2a + b = 50.

Si trazamos la altura correspondiente a la base se forman dos triángulos rectángulos y en uno

cualquiera de ellos aplicaremos el teorema de Pitágoras, pues sabemos que h = 15

2

2

2 152

ba . Despejaremos b = 50 – 2 a en la primera ecuación ; se sustituye en la

segunda y nos queda 2252

2502

2

aa donde, realizando la operación indicada y

eliminando el denominador nos queda la ecuación 900)250(4 22 aa

que nos conduce a 2000 a = 34000 con solución a = 17 y entonces b = 16.

Calculamos el área 22 2,11202

15.16dmcmA .

5.- La diagonal interior forma con la diagonal de la base y la altura un triángulo rectángulo. El

ángulo de inclinación de la diagonal (hipotenusa) es de 600 y como la misma mide 10 cm

entonces la diagonal del cuadrado base mide 5 cm (se opone al ángulo de 300) y la altura

medirá cm35 (se opone al ángulo de 600).

Calculamos su volumen 3322

11,01,1082

73,1.12535.

2

5.

2. dmcmh

dhAV B .

La arista de la base mide 2

25a y entonces podemos calcular el área total

45,2.50256502535.2

25.4

2

5.2..4

2.2.2

22

had

AAA LBT

22 5,15,1475,12225 dmcmAT .

Page 34: PRUEBA 1 - math.cubava.cumath.cubava.cu/files/2015/05/solucionario-libro-jacinto.pdf · g 9 2x 3 x 2 y ahora podemos ... llegamos a la conclusión parcial y = 8 ó y = 1 para los

34

PRUEBA 19

1.- Si en el segundo miembro de la ecuación sustituimos sen 2x = 2 sen x.cos x podemos

simplificar considerando que cos x 0 , entonces la ecuación nos queda

xx sen14cos2 donde podemos elevar al cuadrado ambos miembros para eliminar la

raíz cos2x + 4 = 1 + 2 sen x + sen2x . Sustituimos cos2x = 1 – sen2x y después de reducir y

ordenar llegamos a la ecuación sen2x + sen x – 2 = 0 que admite la descomposición en

factores ( sen x + 2 )( sen x – 1 ) = 0 donde sen x = – 2 (no es posible) ó sen x = 1

entonces x = /2 (que no puede ser solución porque en la ecuación original convierte el

denominador en cero). S = { } .

2.- Propondremos para el ángulo menor la variable x , entonces se puede plantear una

ecuación referida completamente a esta variable x + ( x + 150 ) + ( x + 300 ) = 1800 (suma de

los ángulos interiores de un triángulo). Donde 3 x = 1450 por lo que x = 450.

Calculamos el ángulo mediano 450 + 150 = 600 y como lo que nos piden es la longitud del

lado mediano, podemos aplicar la ley de los senos sin necesidad de figura de análisis pues en

un triángulo, al lado menor se opone el ángulo menor.

00 45sen

4

60sen

L donde podemos despejar L y calcular:

cmcmL 9,48786,441,1.73,1.22

234232

2

34

2

2

2

3.4

45sen

60sen.40

0

3.- a) El gráfico que nos piden es muy fácil construirlo.

b) Con un simple análisis podemos decidir que mr1 = –1 , aunque también se puede hallar

mediante el cálculo; entonces mr2 = 1 pues r1 r2 . Podemos ahora escribir una ecuación no

paramétrica de la recta r2 ya que de ella conocemos un punto y su pendiente.

y – 2 = 1 ( x – 0 ) donde podemos escribir r2 : x – y + 2 = 0

c) Hallaremos el punto de intersección M de ambas rectas. Primero hay que tener la

ecuación de la recta r1 ; y – 0 = –1 ( x – 4 ) , o sea, y = – x + 4 que se puede sustituir de

inmediato en r2 y obtener x = 1 para luego hallar y = 3.

El área pedida se calcula restando del área del triángulo que forman las dos rectas con el eje X

el área del triángulo pequeño que forma la recta r2 con los ejes de coordenadas; M ( 1 ; 3 ) y

consideramos que D es el punto de intersección de la recta r2 con el eje X.

20,7292

2.2

2

3.6cmAAA CODAMDAMCO

Page 35: PRUEBA 1 - math.cubava.cumath.cubava.cu/files/2015/05/solucionario-libro-jacinto.pdf · g 9 2x 3 x 2 y ahora podemos ... llegamos a la conclusión parcial y = 8 ó y = 1 para los

35

4.- Analizamos que el triángulo rectángulo que se forma entre la generatriz del cono, el

radio y la altura del cono es un triángulo isósceles, ya que tiene un ángulo de 450, por lo que r

= hco

La altura del cilindro es hCI = 3 . 3,2 = 9,6 cm .

3

2,36,9.

33

.. 2r

hhA

hAhAVVVr CO

CIB

COB

CIBCOCI

332 27,03,27453,8.24,10.14,3)07,16,9(2,3.14,3 dmcmVr

5.- De inicio hay que expresar que el argumento tiene que ser mayor que cero

019168

2

23

xx

xxx que al efectuar y reducir los términos semejantes nos conduce a

09157

2

23

xx

xxx que se factoriza 0

)1(

)3)(1( 2

xx

xx y considerando que x 1

podemos simplificar y llegar a la expresión 0)3( 2

x

x

que nos conduce al siguiente análisis de intervalos

– 0 + 3 +

O////////////////////O////////////////////////////

y podemos escribir Dom f = { x reales : x > 0 ; x 1 ; x 3 }.

PRUEBA 20

1.- Convertimos la expresión a potencia 12

cos

cos2senxtan

x

xx

; eliminamos denominador

y a la vez sustituimos la tangente x

xxxx

2

2

cos

sen.coscos2sen con

cos x 0 que al simplificar en el miembro derecho y de nuevo eliminar denominador

llegamos a la ecuación 2 cos2x + sen x . cos x – sen2x = 0 que se descompone en factores

( 2cos x – sen x )( cos x + sen x ) = 0 donde 2cos x = sen x ó cos x = –sen x

y podemos escribir 2cos

sen

x

x ó 1

cos

sen

x

x , lo que es lo mismo que

tan x = 2 ó tan x = –1 . De la primera obtenemos, utilizando la tabla x = 63,40 ó

x = 243,40 (utilizando la fórmula de reducción para el tercer cuadrante). Por otra parte

tan x = –1 ; con posible soluciones x = 1350 (segundo cuadrante) ó x = 3150

(fórmula de reducción en el cuarto cuadrante).

La primera y la tercera no están en el intervalo inicial de la ecuación y la última no pertenece

al dominio de la ecuación original pues conduce a que la base sea 1.

Única solución x = 243,40.

Page 36: PRUEBA 1 - math.cubava.cumath.cubava.cu/files/2015/05/solucionario-libro-jacinto.pdf · g 9 2x 3 x 2 y ahora podemos ... llegamos a la conclusión parcial y = 8 ó y = 1 para los

36

Otra vía :

Para los valores admisibles se tiene que xtanx

xx 2

cos

cos2sen

, es decir,

tan x + 2 = tan2 x donde podemos llegar a que tan2x – tan x – 2 = 0 y obtener tan x = 2 ó

tan x = –1 .

2.- En la figura trazamos las construcciones auxiliares rOBAO y la altura OM , relativa

a la base AB del triángulo isósceles AOB.

Podemos calcular el radio del círculo pues conocemos su área 31,4 = .r2

Donde r2 = 10 , o sea, 10r . Aplicamos el teorema de Pitágoras en el triángulo

rectángulo AOM y calculamos 191022

OMAOOM .

Se cumple que triángulo ADC AOM pues ambos triángulos son rectángulos y

ACB = AOM (el ACB es la mitad del AOB, inscrito y central respectivamente sobre

el arco AB, pero el AOM también es la mitad del AOB en el AOB) entonces los

triángulos tienen dos ángulos respectivamente iguales.

Podemos plantear la proporcionalidad entre los lados homólogos OM

CD

AM

AD

AO

AC que

sustituyendo 1310

4 CDAD despejando

5

10.6

10

10.12

10

3.4AD y

5

10.2

10

1.4CD . Podemos finalmente calcular el área del ACD

24,25

12

5

2.6

25.2

10.12

2

25

10.12

2

5

106.

5

10.2

2

.cm

ADCDA .

3.- Consideramos las siguientes variables

matrícula de preuniversitario : x

matrícula de secundaria básica : y entonces x + y = 3710

45%.( 95%.x + 90%.y ) = 95%.x

y nos queda la segunda ecuación más simplificada escribiendo

xyx .100

95.

100

90.

100

95.

100

45

que conduce a xyx .

20

19.

10

9.

20

19.

20

9

si efectuamos las operaciones, reducimos términos semejantes y despejamos xy .162

209 .

Page 37: PRUEBA 1 - math.cubava.cumath.cubava.cu/files/2015/05/solucionario-libro-jacinto.pdf · g 9 2x 3 x 2 y ahora podemos ... llegamos a la conclusión parcial y = 8 ó y = 1 para los

37

Sustituimos en la primera ecuación 3710.162

209 xx

3710.162

371x donde 1620

371

601020

371

162.3710x

Calculamos entonces 3710 – 1620 = 2090 y 90%.2090 = 1881

Concursaron 1881 alumnos de secundaria básica.

4.- Comenzaremos escribiendo 0259 2

23

x

xxx con x < –1,6 que se puede descomponer en

factores 0)53)(53(

)1.( 2

xx

xxx pero en el trinomio x2 + x + 1 tenemos que D < 0 y entonces

no se puede descomponer en factores reales, por lo que x2 + x + 1 > 0 para todas las x reales

ya que el coeficiente de x2 es positivo. Analizamos los intervalos

– –5/3 + 0 – 5/3 +

O//////////////////////O///////////////////////

–1,6

/////////////////////////////O

Intersectando se obtiene la repuesta –5/3 < x < –1,6 [ 5/3 = 1,666… ]

5.- Como el perímetro del hexágono base es de 36dm entonces la arista de la base se puede

obtener a = 6dm = R (radio de la circunferencia circunscrita).

Como el ángulo de inclinación de la arista lateral L es de 600 podemos decidir que

L = 12dm (se opone al ángulo de 300) y que la altura de la pirámide está dada por

36h . Debemos calcular el radio de la circunferencia inscrita en el hexágono base, que

coincide con la altura de los triángulos equiláteros de la base 3.32

36

2

3.

ahAP .

Ya podemos calcular el volumen residual

2

2

.4

3..6.

3

3.6.

33

.

3

.APBCBH

BCBH hL

AAhhAhA

VcVpVr

2

2

)3.3.(14,32

3.6.3.3.2Vr

33129336.927.14,3.3.22

3.36.3.3.2dmVR

Hay que rebajar 31dm3.

Page 38: PRUEBA 1 - math.cubava.cumath.cubava.cu/files/2015/05/solucionario-libro-jacinto.pdf · g 9 2x 3 x 2 y ahora podemos ... llegamos a la conclusión parcial y = 8 ó y = 1 para los

38

PRUEBA 21

1.- Si elevamos los dos miembros al cuadrado nos queda la igualdad

84224424 232324 xxxxxxxx y cuando trasponemos, reducimos y

ordenamos queda x4 + 3x2 – 4 = 0 que se puede descomponer en factores de la siguiente

manera ( x2 + 4 )( x2 – 1) = 0 donde x2 = – 4 (imposible en los reales)

ó x2 = 1 y se obtienen x = 1. Si comprobamos vemos que ambas satisfacen la igualdad

inicial.

2.- En la figura, 16 EHAD (datos), y 8 EDAE (E es punto medio).

Podemos probar que AFE EHD pues ambos triángulos son rectángulos y además

EHD = AEF (ambos son agudos y tienen sus lados respectivamente perpendiculares)

entonces tienen dos ángulos respectivamente iguales.

En el triángulo EDH, como la hipotenusa mide el doble de un cateto, entonces el otro cateto

mide 38DH . Podemos escribir la relación entre los lados proporcionales de los

triángulos semejantes DH

AE

ED

AF

EH

EF sustituimos

38

8

816

AFEF y entonces podemos

calcular a voluntad cualquiera de los segmentos que necesitamos para responder la pregunta

que nos hace este ejercicio. Si calculamos EF obtenemos 3

316

38

8.16EF y hallamos el

área del rectángulo pequeño 22 5,163,147

3

73,1.256

3

316.16. dmcmEFEHAEFGH

Podemos utilizar la vía de calcular AF y hallar el área del rectángulo mayor que nos lleva a

la otra respuesta del ejercicio, 20,3 dmAABCD

3.-Nos percatamos de que 2x + 2 > 0 para todo valor real de x ; establecemos la desigualdad

propuesta 3

2

22

4

3

32 x

x

x

y podemos eliminar los denominadores multiplicando en

ambos miembros por 3.( 2x + 2 ) > 0 entonces obtenemos la desigualdad

( 2x + 3 )( 2x + 2 ) – 12 < 2x ( 2x + 2 ) donde, si multiplicamos y trasponemos llegamos a

22x + 5.2x + 6 – 12 – 22x – 2.2x < 0 donde 3 . 2x < 6

y finalmente 2x < 2 quedando como única posibilidad x < 1 (la base es mayor que 1, se

mantiene el sentido de la desigualdad).

4.-Tomamos por variables Precio del cuadro : c

Precio del tenedor : t

Podemos formar las siguientes ecuaciones c + t = 66

t = 36%.(c + 2)

simplificamos en la expresión del tanto por ciento )2.(25

9 ct , que la podemos sustituir

directamente en la primera ecuación 66)2.(25

9 cc y eliminamos el denominador

25 c + 9 c + 18 = 1650 y así 34 c = 1632 , donde c = 48 y entonces t = 18 .

Y calculamos 48 – 18. La diferencia es de $30,00

Page 39: PRUEBA 1 - math.cubava.cumath.cubava.cu/files/2015/05/solucionario-libro-jacinto.pdf · g 9 2x 3 x 2 y ahora podemos ... llegamos a la conclusión parcial y = 8 ó y = 1 para los

39

5.-Como conocemos el volumen del cono inferior y el radio de su base, podemos calcular su

altura en I

IIIIB hhhhrhA

.42,93

.9.14,3

3

.3.14,3

3

..

3

.1,47

22

y entonces cmhI 542,9

1,47 . En el triángulo ABC, rectángulo en B, podemos aplicar el

teorema de la altura para calcular la altura del cono superior SI hhr .2 , o sea,

mh

rh

I

S 8,15

9

5

322

. Calculamos el volumen del cono superior

33 1796,163

87,50

3

8,1.9.14,3

3

.mm

hAV SB

CS

VDIF= VCI – VCS = 47,1 – 17 = 30,1m3=30m3 .

PRUEBA 22

1.-Si, directamente, elevamos al cuadrado ambos miembros para eliminar las raíces cuadradas

debemos tener cuidado con el miembro derecho pues, al aplicar potencia de potencia, hay que

multiplicar correctamente los exponentes:

211 232)32)(12(212 xxxxx . En este ejercicio, para agilizar el trabajo,

vale la pena aplicar un cambio de variable, considerando y = 2x ,

yyyyy 43)3)(12(212 , preparamos la ecuación para volver a elevar al

cuadrado 23522 2 yyy entonces 4.(2y2 –5y –3) = y2 + 4y + 4 de la que haciendo

las operaciones necesarias podemos llegar a la ecuación cuadrática ordenada

7y2 – 24y – 16 = 0 que se puede descomponer en factores mediante

(7y + 4).(y – 4) = 0 , o sea, y = – 4/7 ó y = 4 ,ahora haremos reversible el cambio de

variable 2x = – 4/7 (imposible) ó 2x = 4 donde podemos calcular la posible solución

x = 2 .

Se comprueba en la ecuación original y se verifica con certeza que es la única solución .

2.- Como M es punto medio de CD entonces se puede asegurar que CDAB en el punto M

(propiedad del diámetro perpendicular a la cuerda en su punto medio) . Podemos calcular que

MD = 2,8 cm. Aplicamos el teorema de Pitágoras en el triángulo rectángulo AMD para

calcular cmMDADAM 5,425,208,23,5 2222

y ahora, trazamos el

segmento auxiliar DB y se forma el triángulo ADB, rectángulo en D (por el teorema de

Tales, ángulo inscrito sobre el diámetro). En este triángulo podemos aplicar el teorema de los

catetos para hallar AB (diámetro), pues

ABAMAD .2

donde podemos despejar cmAM

ADAB 24,6

5,4

09,28

5,4

3,5 22

Page 40: PRUEBA 1 - math.cubava.cumath.cubava.cu/files/2015/05/solucionario-libro-jacinto.pdf · g 9 2x 3 x 2 y ahora podemos ... llegamos a la conclusión parcial y = 8 ó y = 1 para los

40

El área que nos piden se puede calcular mediante la diferencia de áreas entre la mitad del

área del círculo y el área del triángulo ADM.

)6,122,30(5,0)8,2.5,41,3.14,3(2

1

2

.

2

. 22

MDAMr

AAA TSEMICIRS

28,86,17.5,0 cmAS Otra vía: Se puede utilizar la ecuación 2

22

)OAAM(MDOD

donde r2 =2,82 +(4,5-r)2

3.-Designamos para el largo del rectángulo la variable : b A = a.b

para el ancho : a

Como la diagonal del rectángulo mide 30 m entonces podemos aplicar el teorema de

Pitágoras y obtener una primera de ecuación b2 + a2 = 302

Otra ecuación puede hallarse mediante a ( b + 6 ) = A + 108 .

Si operamos en esta última y sustituimos A=a.b nos queda una ecuación donde podemos

simplificar a.b : a.b + 6.a = a.b + 108 donde 6 .a = 108 y calculamos directamente

que a = 18 y ahora podemos calcular la longitud b en la primera ecuación mediante

2457632490018900900 22 ab

Calculamos el perímetro P = 2 ( b + a ) = 2 ( 24 + 18 ) = 2 . 42 = 84 m.

4.- a)En principio no puede estar definida para ningún ángulo axial, pues indefinen a las

funciones cotangente y tangente según el caso. Pero el denominador se hace cero cuando

cot x = tan x y esto ocurre en todos los ángulos que su lado terminal sea bisectriz de algún

cuadrante, por tanto, la expresión no estará definida para los siguientes ángulos :

x = k/2 ; x = /4 + k/2 ; k entero

b) Partiremos del segundo miembro para intentar llegar al primero, haciendo transformaciones

x

xxxx

xx

x

x

x

x

s

x

x

tanxx

x22

222

22

222

sencos

cos.sen.sencos4

cos.sen

sencos

2cos4

cos

sen

sen

cos

2cos4

cot

2cos4

xxxxxxxxxx 2sen.2cos.2cos.sen.2.sencos2cos.sen.sencos4 2222

x4sen . Queda demostrada la identidad para los valores admisibles de la variable.

5.- Con los datos que nos dan podemos calcular inicialmente el área del paralelogramo base

mediante 20 36

2

3.12120sen.3.4 cmAB .

También en el paralelogramo base podemos calcular la longitud de la diagonal mayor que

llamaremos D , aplicando la ley de los cosenos

cmD 3712252

1.24916120cos.3.4.234 022

y de igual manera procederemos para calcular la diagonal menor d que se opone al ángulo de

600.

Page 41: PRUEBA 1 - math.cubava.cumath.cubava.cu/files/2015/05/solucionario-libro-jacinto.pdf · g 9 2x 3 x 2 y ahora podemos ... llegamos a la conclusión parcial y = 8 ó y = 1 para los

41

cmd 1312252

1.2491660cos.3.4.234 022

Podemos calcular la altura h del paralelepípedo recto trabajando en el triángulo rectángulo

que tiene como hipotenusa D (ahora diagonal interior menor del paralelepípedo) y como

cateto la diagonal menor d en el paralelogramo base.

cmh 626.4241337133722

Calculamos ya, finalmente, el volumen, 33 5176,5041,1.362.3.123.2.3.1262.3.6. cmcmhAV B .

PRUEBA 23

1.- Exploramos inicialmente las bases y podemos escribir 5

24,0 y por otra parte

2

2

5

4

2525,6

y entonces podemos sustituir en la desigualdad original a la vez que

aplicaremos la propiedad de potencia de potencia en el miembro derecho

32 log241log

2

5

5

2xx

y como

1

5

2

2

5

podemos trabajar en el miembro derecho

de la ecuación y escribir

4log21log 32

5

2

5

2

xx

; como las bases ya son iguales

entonces podemos igualar los exponentes log2x + 1 = 2 log x3 – 4 donde

log2x + 1 = 6 log x – 4 que se puede escribir log2x – 6 log x + 5 = 0 y que se descompone

en factores mediante ( log x – 5 )( log x – 1 ) = 0 ; se obtienen los resultados log x = 5 ó

log x = 1 , o sea, x = 105 ó x = 10 .

2.-Si hacemos una buena figura de análisis podemos observar que para poder modelar el

ejercicio tenemos que prolongar los dos lados y entonces, fuera del trapecio, se forma el

ángulo recto entre los mismos.

Tenemos entonces un triángulo rectángulo con ángulos de 300 y 600 (ángulos

complementarios) .

En la fórmula del área del trapecio podemos trabajar y calcular la altura del mismo,

pues sabemos que la paralela media mide 2

10ba

entonces hba

ATRA .2

donde 40 = 10 . h y calculamos h = 4 cm .

Si en la figura de análisis trazamos dos sugerentes alturas en los extremos de la base menor

¨ a¨ entonces se forma un rectángulo de lados h y a sobre la base b .

Se forman, a ambos lados de este rectángulo, dos triángulos rectángulos con catetos h

y ángulos de 300 y 600 que podemos en los mismos calcular la longitud de los catetos que se

forman sobre la base b y que llamaremos m y n.

Page 42: PRUEBA 1 - math.cubava.cumath.cubava.cu/files/2015/05/solucionario-libro-jacinto.pdf · g 9 2x 3 x 2 y ahora podemos ... llegamos a la conclusión parcial y = 8 ó y = 1 para los

42

Podemos plantear, trabajando en la base, que b = a + m + n .

m

tan4

3

330 0 y además,

ntan

43600

343

3.12

3

3.4m y

3

34

3

4n

Ahora podemos formar un sistema de ecuaciones 20 ba

3

3434 ab

Si sustituimos directamente la segunda ecuación en la primera podemos calcular la longitud

del segmento a ; 203

3434 aa donde

3

316202 a y calculamos el

valor de a para concluir nuestra prueba.

cma 38,562,4103

73,1.810

3

3810 . Queda probado que a > h .

3.-Se tiene que cumplir, necesariamente, en el intervalo dado que:

cos 2x – 5 cos x – 2 0 y si sustituimos adecuadamente cos 2x = 2 cos2x – 1

entonces nos queda que 2 cos2x – 5 cos x – 3 0 que podemos descomponer en factores

( 2 cos x + 1 )( cos x – 3 ) 0 y al resolverla nos lleva a intervalos

+ –1/2 – 3 +

/////////////////////////////////////////////////

Vemos que el intervalo de la derecha no tiene sentido para cos x pues se cumple siempre que

–1 cos x 1 .

Por lo que nos queda solo por analizar –1 cos x –1/2 para x en [ 0 ; 2 ]

Pero cos x = –1/2 cuando x = 2/3 ó x = 4/3

cos x = –1 cuando x =

entonces se satisface para el intervalo completo 3

4

3

2 x que contiene a x = .

4.-Es un problema de aplicación en la Física. Recordemos que v

st

Tiempo en el viaje de ida : v

t I

300 Tiempo en el viaje de regreso :

10

300

vtR

Pero tenemos que 11 RI tt , y sustituyendo en esta ecuación llegamos a la ecuación

fraccionaria 1110

300300

vv donde eliminamos los denominadores

300 ( v – 10 ) + 300 v = 11 v ( v – 10 ) y podemos formar la nueva ecuación cuadrática

ordenada y comparada con cero 11 v2 – 710 v + 3000 = 0

que se descompone en factores ( 11v – 50 )( v – 60 ) = 0

v = 50/11=4,55 km/h (imposible) ó v = 60 km/h .

Velocidad en el viaje de regreso 50 km/h entonces s = v . t = 50 . 2 = 100 km

Recorrió 100 km en las dos primeras horas en el viaje de regreso.

Page 43: PRUEBA 1 - math.cubava.cumath.cubava.cu/files/2015/05/solucionario-libro-jacinto.pdf · g 9 2x 3 x 2 y ahora podemos ... llegamos a la conclusión parcial y = 8 ó y = 1 para los

43

5.- Nos dan el volumen 5,12 dm3 = 5120 cm3 y se cumple que, trabajando en cm, podemos

plantear PIRAMIDEPRISMA VVV

hhhbahba

hba .3

1280.32.10.

3

4...

3

4

3

....5120 donde cmh 12

1280

3.5120

En el triángulo rectángulo de apoyo de la cara lateral delantera en la pirámide podemos

observar que el cateto en la base de la misma mide 5 cm (mitad de la arista de la base que

mide 10 cm) y como la altura mide 12 cm , entonces, la hipotenusa mide 13 cm (trío de

números de Pitágoras).Esta hipotenusa es la altura del triángulo lateral (cara) isósceles.

De igual manera podemos calcular la altura del triángulo lateral (cara) isósceles por el costado

de la figura, catetos de 12 (altura de la pirámide) y 16 (mitad de la arista de la base) entonces

la hipotenusa mide 20 (trío de números pitagóricos) .

Podemos calcular el área lateral de todo el cuerpo

162420041642.242

20.10.2

2

13.32.212).1032.(2.. PIRAMLPRISLL AAA

El área lateral es de 16,2 dm2.

PRUEBA 24

1.- Se pueden elevar ambos miembros al cuadrado para tener posibilidades de llevar toda la

ecuación a la misma función trigonométrica:

cos2x – 2 cos x . sen x + sen2x = sen22x + 2 sen 2x + 1 pero sen2x + cos2x = 1

queda entonces la ecuación en la única variable sen 2x ;

sen22x + 3 sen 2x = 0 que se descompone en sen 2x ( sen 2x + 3 ) = 0

donde sen 2x = 0 ó sen 2x = – 3 (imposible)

y podemos plantear que se cumple para 2 x = k ; con k entero

x = k/2

Calcularemos ahora cuáles son las posibles soluciones en el intervalo que se nos indica al

inicio para k = 0 ; x = 0

para k = 1 ; x = /2 (se comprueba que no es solución)

para k = 2 ; x = (se comprueba que no es solución)

para k = 3 ; x = 3/2

para k = 4 ; x = 2

Únicas soluciones x = 0 ó x = 3/2 ó x = 2.

2.- a)Como M( ½ ; ½ ) es punto medio del segmento AB y conocemos las coordenadas de

A( –2 ; 3 ) podemos calcular las coordenadas B( xB ; yB ) .

2

2

2

1 Bx

y 2

3

2

1 By

donde xB = 3 ; yB = – 2 por tanto podemos escribir

que B( 3 ; –2 ) .

Page 44: PRUEBA 1 - math.cubava.cumath.cubava.cu/files/2015/05/solucionario-libro-jacinto.pdf · g 9 2x 3 x 2 y ahora podemos ... llegamos a la conclusión parcial y = 8 ó y = 1 para los

44

b)Comprobaremos si el triángulo tiene alguna característica especial que nos pueda ayudar

en el proceso de cálculo para el área, de lo contrario tendremos que utilizar la fórmula de

Herón. Analizaremos las pendientes, 23

6

03

42

CB

CB

BCxx

yym

2

1

)2(0

34

ACm y con este cálculo de las pendientes podemos afirmar que los

segmentos BC y AC son perpendiculares pues se cumple que mAC . mBC = –1 .

uyyxxBC CBCB 5345)6(3)42()03( 222222

uAC 514)1()2()43()02( 2222 .

Calculamos el área 25,7

2

15

2

5.3

2

5.53

2

.u

ACBCAT .

3.- Cuando nos referimos a, único intervalo, lo hemos hecho con el propósito de hallar el

intervalo máximo; sabemos que existen infinitos intervalos, subconjuntos propios del

intervalo solución, que también pueden ser solución de este ejercicio.

Necesariamente, antes de comenzar el trabajo algebraico, debemos analizar que x > 0 para

todos los x reales que aparezcan como posibles respuestas de la desigualdad.

Para trabajar con mayor comodidad consideremos y = log2x .

La desigualdad puede quedar ahora como 11

11

yy que inmediatamente vamos a

comparar con cero, 011

11

yy para realizar las operaciones en el miembro izquierdo

de la desigualdad trabajando con el mínimo común denominador y ( y – 1 )

0)1(

)1(1

yy

yyyy ; y que al operar y reducir en el numerador, arribamos a la

desigualdad 0)1(

12

yy

yy que podemos escribir como 0

)1(

12

yy

yy donde, al intentar

descomponer en factores el numerador, encontramos que su discriminante

D=b2 –4ac= (–1)2 – 4.1.1 = 1 – 4 = –3 < 0 por lo que no tiene descomposición factorial en los

reales. El trinomio y2 – y + 1 va a tomar siempre valores positivos para todo valor real que

asignemos a su variable pues el coeficiente de y2 es positivo. El análisis de intervalos para la

desigualdad toma entonces el siguiente esquema de resolución:

+ 0 – 1 +

O//////////////////////////O

donde arribamos a la conclusión parcial que 0 < y < 1 para luego hacer reversible nuestro

cambio de variable log2 1 < log2 x < log2 2 concluyendo que, como la base es

mayor que 1 ,se cumple solo si 1 < x < 2 . Este es el intervalo máximo que nos pide el

ejercicio.

4.- Tomaremos las variables con su letra de comienzo, o sea, A , L y C. Podemos formar

Page 45: PRUEBA 1 - math.cubava.cumath.cubava.cu/files/2015/05/solucionario-libro-jacinto.pdf · g 9 2x 3 x 2 y ahora podemos ... llegamos a la conclusión parcial y = 8 ó y = 1 para los

45

entonces dos ecuaciones leyendo directamente en el texto del ejercicio,

2 L + 3 A = 26 C

(*) L + 13 C = 5 A quedando esta última como 5 A – L = 13 C y el sistema

aparece ahora más cómodo para su resolución mediante el método de reducción:

3 A + 2 L = 26 C 3 A + 2 L = 26 C

5 A – L = 13 C (2) 10 A – 2 L = 26 C 13 A = 52 C

A = 4 C

con este resultado podemos, en la ecuación (*) , calcular L = 7 C .

Podemos ahora responder 5 L + 7 A = 5 .(7C) + 7 .(4C) = 35 C + 28 C = 63 C .

Se pueden obtener 63 cocos.

5.- Trabajaremos calculando el volumen para verificar que el alumno tiene razón.

Debemos calcular algunos valores para poder aplicar la fórmula y calcular el volumen de la

pirámide. El segmento BC = 5 cm (se opone al ángulo de 300 en el triángulo rectángulo

ABC donde la hipotenusa mide 10 cm) y el segmento AB = 5 3 cm (se opone al ángulo de

600). Como CD es la altura de la pirámide entonces BD es la proyección de CB sobre el

plano base y debido a que el triángulo ABC es rectángulo en B entonces en el punto B se

cumple el teorema de las tres perpendiculares por lo que el triángulo ABD (base de la

pirámide) es también rectángulo en B .

Podemos, de manera rápida, calcular BDCD = cm2

25 (lado del triángulo rectángulo

isósceles BDC que su hipotenusa mide 5 cm).

Calculamos el volumen de la pirámide:

12

25.

12

25.35

1

62

25

.356

2

25.

2

25.35

3

.2

.

3

.AB

CDBDAB

hAV B

Queda probada la razón que tenía el alumno en su planteamiento.

33 1802,1812

73,1.125

12

25.35 cmcmV

PRUEBA 25

1.- En la ecuación original podemos, de inmediato realizar la transformación a ecuación

exponencial de base 2 escribiéndola como 2sencos1 22

2102 xx y se puede llevar

a la forma 0102.42.222 sencos x

; y haciendo la sustitución

cos2x = 1– sen2x, para convertir a la misma función trigonométrica obtenemos,

0102.42.222 sensen1 x

que se puede expresar, haciendo transformaciones en

su primer término, como 0102.42

4 2

2

sen

sen x

x

Page 46: PRUEBA 1 - math.cubava.cumath.cubava.cu/files/2015/05/solucionario-libro-jacinto.pdf · g 9 2x 3 x 2 y ahora podemos ... llegamos a la conclusión parcial y = 8 ó y = 1 para los

46

Podemos hacer un cambio de variable para simplificar el trabajo: xw

2sen2

010.44

ww

; eliminamos el denominador y obtenemos la ecuación cuadrática ya

ordenada 4 w2 – 10 w + 4 = 0 que se simplifica y se descompone en factores

( 2 w – 1)( w – 2 ) = 0 por lo que w =1/2 ó w = 2 (soluciones parciales).

Entonces 2

12

2sen x ó .22

2sen x La primera es imposible pero la segunda nos lleva

a sen2x = 1 por lo que sen x = 1 y entonces los ángulos son los siguientes

x = /2 + k ; k entero.

2.-Hacemos el dibujo adecuado de las circunferencias (una es mayor que la otra) y apreciamos

que se intersectan en dos puntos que llamaremos P y Q . Estos dos puntos forman la cuerda

PQ de la cual debemos calcular su longitud.

Denotaremos las circunferencias C’(O’; r=6) y C(O; r=8) ; podemos trazar el romboide

simétrico de vértices O’POQ y señalar al punto medio de PQ por M (sobre el

segmento OO' ).

En el triángulo O’PO se cumple que 102 = 82 + 62 por lo que dicho triángulo es rectángulo en

P (recíproco del teorema de Pitágoras) y podemos entonces calcular su área

2242

6.8cmA , resultado que nos servirá para calcular la longitud del segmento MP , ya

que hay otra manera de calcular el área trabajando con la hipotenusa y su altura relativa, que

precisamente es MP .

2

.10

2

.'24

MPMPOOA donde podemos despejar cmMP 8,4

10

24.2 y ahora,

como el romboide simétrico es una figura axialmente simétrica respecto a su diagonal O´O

al multiplicar por 2 obtenemos la longitud de la cuerda común PQ = 9,6 cm.

3.- Preparemos el trabajo con la variable

ha de boniatos : x

ha de papas : x + 480

Si se recolecta el 80% de las hectáreas de papas, entonces queda en el campo el 20%.

Si se recolecta el 25% de las hectáreas de boniatos, queda el 75% en el campo.

Podemos formar la desigualdad 20%( x + 480 ) < 75%x donde hay que restar 300 a

las hectáreas que quedan de boniato para poder igualar y obtener la ecuación

20%( x + 480 ) = 75%x – 300 que la simplificamos 3004

3)480.(

5

1 xx y nos

conduce a la ecuación más sencilla 4 ( x + 480 ) = 15 x – 6000 donde 11 x = 7920

para hallar x = 720 . Estaban sembradas 720 ha de boniatos y 1200 ha de papas.

Page 47: PRUEBA 1 - math.cubava.cumath.cubava.cu/files/2015/05/solucionario-libro-jacinto.pdf · g 9 2x 3 x 2 y ahora podemos ... llegamos a la conclusión parcial y = 8 ó y = 1 para los

47

4.- Primero hay que resolver la ecuación dada. Si eliminamos los denominadores llegamos

a la ecuación 3 k x – 2 x = 5 x + 10 k donde 3 k x – 7 x = 10 k por lo que

x ( 3 k – 7 ) = 10 k , y despejando, 73

10

k

kx .

Planteamos la condición que nos propone el ejercicio 173

10

k

k que debemos comparar con

cero 0173

10

k

k y efectuar en el miembro izquierdo manteniendo, claro está, el

denominador 073

77

k

k que resolvemos con los intervalos

+ –1 – 7/3 +

/////////////////////////OO//////////////////////////////

y obtenemos que k < –1 ó k > 7/3 para que x > 1.

5.- Con el volumen de la esfera podemos calcular su radio.

3

.56,12

3

.14,3.4..

3

4523

33

3 EE

EE

rrrV despejamos 125

56,12

1569

56,12

523.33 Er

donde rE = 5 cm.

En la figura trazamos la construcción auxiliar (con mucho cuidado) OP = rE desde el centro

de la esfera hasta la generatriz del cono y, si le asignamos al vértice del cono la letra S ,

entonces el triángulo OPS es rectángulo en P (radio en el punto de tangencia).

Como la altura del cono es h = SM = 18 cm entonces por resta de segmentos

OS = 13 cm. También podemos hallar SP = 12 cm (trío de números pitagóricos) en el

triángulo rectángulo OPS. Consideremos que la generatriz g = SR con P SR .

Se cumple que OPS SMR ,ambos son rectángulos y tienen al MSR por ángulo

común. (tienen dos ángulos respectivamente iguales).

Podemos plantear las igualdades SM

SP

MR

OP

SR

OS por ser lados proporcionales, se oponen

a los ángulos respectivamente iguales en los triángulos semejantes.

Sustituimos los valores de las longitudes que conocemos, 18

12513

MRSR y ahora podemos

despejar convenientemente cmSR 5,1912

234

12

18.13

cmMR 5,712

90

12

18.5

Page 48: PRUEBA 1 - math.cubava.cumath.cubava.cu/files/2015/05/solucionario-libro-jacinto.pdf · g 9 2x 3 x 2 y ahora podemos ... llegamos a la conclusión parcial y = 8 ó y = 1 para los

48

Calculamos el volumen del cono y su área lateral.

3322

B dm1,1cm75,10593

18.25,56.14,3

3

18.5,7.14,3

3

h.r.

3

h.AV

22 6,423,4595,19.5,7.14,3.. dmcmgrAL .

PRUEBA 26

1.-No siempre hay que elevar al cuadrado ambos miembros, en muchas ocasiones es necesario

elevar al cubo para eliminar la o las raíces.

x3 + 6x + 20 = x3 + 6x2 + 12x + 8 ; reduciendo y ordenando 6x2 + 6x – 12 = 0

se simplifica y se llega a x2 + x – 2 = 0 que se descompone en factores mediante

( x + 2 )( x – 1 ) = 0 para obtener x = –2 ó x = 1.

w = log3 ( –2 + 2 ) imposible ; w = log3 ( 1 + 2 ) = log3 3 = 1.

Único valor posible w = 1.

2.- Designaremos por el total de votos : x

Votos de Olga : xx .5

1%.20

Votos de René : xxx .15

8.

5

4.

3

2%.80.

3

2

Votos de Dora : 940

Podemos plantear la siguiente ecuación 940.15

8.

5

1 xxx

Eliminamos denominadores 15 x = 3 x + 8 x + 14100

Entonces nos queda 11 x = 14100 donde x = 3525 y ahora podemos calcular la

cantidad de electores E de la circunscripción en 99,80%.E = 3525 .

06,35328,99

352500

80,99

100.3525E . Aclaración importante: Cuando el delegado calculó el

tanto por ciento de los votantes emitió una aproximación al obtener 99,80% y entonces ahora

debemos buscar cuál es la cantidad entera de electores, por ello vamos a comprobar bien antes

de responder, 8018,993532

100.3525 . Son 3532 electores y no hay otra respuesta posible!

3.- En el intervalo que nos dan debemos hacer una primera aclaración:

x k/2 ; k entero pues con los mismos se anulan indistintamente los denominadores

de la desigualdad.

Comparamos con cero 03

8

cos

1

sen

122

xx

operamos en el miembro izquierdo con el

mínimo común denominador 0cos.sen3

cos.sen8sen3cos322

2222

xx

xxxx

Page 49: PRUEBA 1 - math.cubava.cumath.cubava.cu/files/2015/05/solucionario-libro-jacinto.pdf · g 9 2x 3 x 2 y ahora podemos ... llegamos a la conclusión parcial y = 8 ó y = 1 para los

49

Sustituimos sen2x = 1 – cos2x en el numerador de la desigualdad y realizamos las

operaciones indicadas 0cos.sen3

cos8cos8cos33cos322

4222

xx

xxxx nos queda

0cos.sen3

3cos2cos822

24

xx

xx que se factoriza 0

cos.sen3

)1cos2)(3cos4(22

22

xx

xx

Analizamos 3sen2x.cos2x > 0 (no olvidar el análisis inicial)

2cos2x + 1 > 0

Por lo que la única posibilidad es que 4cos2x – 3 0 que se descompone en factores

0)3cos2)(3cos2( xx y analizamos los intervalos

+ – 2

3 –

2

3 +

//////////////////////////////////////////////////////////////

–1 cos x 1

/////////////////////////////////////////////////////////////

Intervalo de la izquierda: –1 cos x –2

3 (imposible en el intervalo que nos señalan al

inicio del ejercicio, pues en el mismo la función cos x siempre es positiva o nula)

Intervalo de la derecha:2

3 cos x 1 . Aquí sí podemos hallar soluciones.

Se cumple para – /6 x /6 ; x 0 en el intervalo dado.

4.- Podemos decidir de inmediato que si tan SBR = –1 entonces SBR = 1350 por lo que

calculamos SBA = 450 (ángulos adyacentes, son suplementarios, suman 1800).

El triángulo SOB es rectángulo e isósceles (O es el centro del círculo base) entonces hay una

relación muy importante: h = r . En el triángulo SOB podemos aplicar el teorema de

Pitágoras g2 = h2 + r2 y llegar a g2 = 2r2 .

En el triángulo rectángulo SOR también podemos aplicar el teorema de Pitágoras y calcular la

longitud de r, r2 + ( r + 7 )2 = 17 2 ; cuando efectuamos, reducimos y ordenamos llegamos

a la ecuación r2 + 7r – 120 = 0 que la descomponemos en factores mediante

( r + 15 )( r – 8 ) = 0 , donde solo admitiremos que r = 8 cm .

Calculamos el volumen del cono mediante:

3332

54,089,5383

512.14,3

3

8.14,3

3

..

3

.dmcm

hrhAV B

Calculamos el área del triángulo SBR:

20 28

2

7.8

2

2.7.2.8.

2

1135sen...

2

1cmBRSBAT

Page 50: PRUEBA 1 - math.cubava.cumath.cubava.cu/files/2015/05/solucionario-libro-jacinto.pdf · g 9 2x 3 x 2 y ahora podemos ... llegamos a la conclusión parcial y = 8 ó y = 1 para los

50

5.- Consideremos las coordenadas del punto A( a ; b ). Podemos en la ecuación de la recta

r : 3x + y + 4 = 0 sustituir sus coordenadas ya que pertenece a dicha recta, por lo que

podemos escribir 3a + b + 4 = 0 . Por otra parte, como el punto equidista de B y de C

podemos igualar las distancias desde A hasta los mismos,

CABA dbabad ,

2222

, )2()3()6()5(

a2 + 10a +25 + b2 – 12b + 36 = a2 – 6a + 9 + b2 – 4b + 4

16a – 8 b + 48 = 0

2a – b + 6 = 0 y ahora trabajamos en el sistema 2a – b + 6 =0

( s : 2x–y+6=0 , recta mediatriz del segmento CB ) 3a + b + 4 =0

que al resolverlo podemos hallar a = –2 ; b = 2 . Las coordenadas de A( –2 ; 2 ) .

PRUEBA 27

1.- Tomaremos la vía de sustituir 1 = log2 2 en la ecuación original y a la vez aplicaremos las

propiedades de los logaritmos en el miembro izquierdo.

xx 1

2

1log2loglog 22

2

2 ahora podemos escribir el segundo miembro con

un solo logaritmo utilizando las propiedades de los mismos,

xx 1

2

1.2loglog 2

2

2 y entonces procedemos a igualar los argumentos ya

que la función log es inyectiva en todo su dominio.

xx 1.212 donde xx 1212

que elevaremos al cuadrado para

eliminar la raíz del segundo miembro y nos conduce a la ecuación,

x4 – 2x2 +1 = 4 ( 1 – x ) , o sea, x4 –2x2 + 4x – 3 = 0 en la que, mediante la división

sintética, hacemos la descomposición en factores,

( x – 1 )( x3 + x2 – x + 3 ) = 0 . Obtenemos x = 1 . La ecuación cúbica , de tener

soluciones reales no serán enteras, no la analizaremos.

Comprobamos en la ecuación original y con certeza x = 1 es la solución pedida.

2.-Un breve análisis inicial nos lleva a pensar que es un problema geométrico donde debemos

aplicar necesariamente el trabajo con variables.

Propondremos : AE = x y entonces BC = x +1

Podemos plantear la ecuación en la fórmula para el área 2

)1(300

xxA

donde 600 = x2 + x que podemos ordenar x2 + x – 600 = 0 y descomponer en

factores fácilmente ( x + 25 )( x – 24 ) = 0 y tomar únicamente x = 24cm .

Entonces podemos decidir que cmBCAB 25 . Ahora el trabajo consiste en calcular la

longitud AC ( base del triángulo isósceles ). En el triángulo rectángulo ABE podemos

aplicar el teorema de Pitágoras y calcular BE ,

cmAEABBE 7495766252425 2222

, y calcularemos la longitud

del segmento EC mediante la resta de segmentos sobre el lado BC ,

Page 51: PRUEBA 1 - math.cubava.cumath.cubava.cu/files/2015/05/solucionario-libro-jacinto.pdf · g 9 2x 3 x 2 y ahora podemos ... llegamos a la conclusión parcial y = 8 ó y = 1 para los

51

BEBCEC = 25 – 7 = 18cm .

Si aplicamos ahora el teorema de Pitágoras en el triángulo rectángulo AEC podemos calcular

la longitud de la base AC del triángulo isósceles ABC,

cmAEECAC 309005763242418 2222

Podemos calcular el perímetro del triángulo ABC; P = 25 + 25 + 30 = 80 cm.

3.- La cantidad de mangos se puede denotar por x .

Haciendo una lectura detallada del problema podemos hallar la sugerente ecuación relativa a

la cantidad total de mangos con las operaciones que se fueron realizando.

Niña Señor Joven

12332

.4

13

2

xx

xx

Esta ecuación puede escribirse de manera más sencilla

15)6.(8

13

2 x

xx y sin denominadores 8x = 4x + 24 + x – 6 + 120

de donde obtenemos x = 46 . En el puesto de frutas había al inicio 46 mangos .

4.-Es importante hacer un buen gráfico de la situación que nos plantean. Podemos hallar la

ecuación cartesiana de la recta r1 . Calculamos su pendiente pues tenemos su ángulo de

inclinación con respecto al eje X . m1 = tan 350 = 0,7 = 7/10 .

La ecuación queda r1 : )3.(10

77 xy , o sea, 7x – 10y + 49 = 0 .

El punto de intersección de la recta r1 con el eje X tiene la forma P( xo ; 0 ) y la abscisa xo

se calcula 7xo = – 49 , donde , xo = – 7 entonces P( – 7 ; 0 ).

La pendiente de la recta r2 se puede calcular fácilmente, pues es perpendicular a la recta r1 y

entonces se cumple que m1 . m2 = – 1. Sustituyendo y despejando adecuadamente

calculamos 7

10

107

11

1

2

m

m y podemos formar la ecuación de la recta

r2 : )7.(7

100 xy donde llegamos a 10x + 7y +70 = 0 .

El punto de intersección de esta última recta con el eje Y es Q( 0 ; yo ) quiere esto decir que

7yo = – 70 para calcular yo = – 10. Q( 0 ; – 10 ) .

5.- Es un ejercicio de Geometría del Espacio que carece totalmente de valores numéricos pues

solamente nos dan relaciones para trabajar.

De la primera relación podemos establecer que ll .42 donde podemos escribir que

0.42 ll y calcular 4l u . Podemos decidir que la diagonal del cuadrado base mide

AC = 24 u y que el área del cuadrado base es ACB = 16 u2 .

Page 52: PRUEBA 1 - math.cubava.cumath.cubava.cu/files/2015/05/solucionario-libro-jacinto.pdf · g 9 2x 3 x 2 y ahora podemos ... llegamos a la conclusión parcial y = 8 ó y = 1 para los

52

El área del triángulo AFC es, según el dato que nos dan al inicio, también 16 u2 .

AAFC= 16 = 2

.2.4

2

. AFAFAC y podemos calcular la altura del prisma despejando

uAF 2.42

2.8

24

16.2 . Para calcular el área lateral de la pirámide primero hay que

hallar la longitud de FB aplicando el teorema de Pitágoras en el triángulo rectángulo ABF.

uFB 34)12(16162.164)2.4( 22

Podemos, aplicando el teorema de las tres perpendiculares, demostrar que el triángulo FBC es

rectángulo en B pues AB es la proyección de FB sobre el plano base y

BCAB . Ya podemos calcular el área lateral de la pirámide ABCF .

3.82.8162

34.4

2

24.416 FBCABFAFC AAAA

21,418,133,111673,1.842,1.816 uA

PRUEBA 28

1.- Podemos escribir la ecuación original con una pequeña transformación,

log3 3 + log3 ( 1 + 2sen x ) = log3 ( 9 + sen 2x . cos x ) aplicando propiedades de los

logaritmos podemos llevar a un solo logaritmo el miembro izquierdo y a la vez hacemos una

sustitución en el miembro derecho en la expresión sen 2x =2senx . cos x ,

log3 [ 3 . ( 1 + 2sen x ) ] = log3 ( 9 + 2sen x . cos2x )

Podemos igualar los argumentos pues la función logarítmica es inyectiva en todo su dominio

3 + 6 sen x = 9 + 2sen x (1 – sen2x ) si realizamos las operaciones indicadas, reducimos y

ordenamos convenientemente llegamos a la ecuación cúbica,

2 sen3x + 4 sen x – 6 = 0 , simplificando aún más, sen3x + 2 sen x – 3 = 0

se puede descomponer en factores utilizando la división sintética o regla de Ruffini y obtener

la expresión siguiente:

( sen x – 1 )( sen2x + sen x + 3 ) = 0 donde la primera ecuación nos conduce a que

sen x = 1 pero la ecuación cuadrática no se puede descomponer en factores con

soluciones reales porque D = b2 – 4.a.c = 12 – 4.1.3 = 1 – 12 = – 11 < 0 .

Entonces, en el intervalo que nos proponen, la única posible solución es x = /2 .

Hacemos la comprobación en la ecuación original y verificamos que es la solución.

2.-Se debe hacer la sugerente construcción auxiliar .ABCT (radio en el punto de tangencia).

Como M es punto medio de AC , entonces rCTNCMCAM .

En el triángulo ATC, rectángulo en T, como CT es la mitad de AC entonces se cumple que

TAC = 300 . En el triángulo ABC, BCAB .2 porque BC se opone al ángulo de 300 ,

entonces AB = 2 ( r + 2 ).

Page 53: PRUEBA 1 - math.cubava.cumath.cubava.cu/files/2015/05/solucionario-libro-jacinto.pdf · g 9 2x 3 x 2 y ahora podemos ... llegamos a la conclusión parcial y = 8 ó y = 1 para los

53

Apliquemos el teorema de Pitágoras en el triángulo rectángulo ABC .

AB 2 = AC 2 + BC 2 , o sea , [2 ( r + 2 ) ]2 = ( 2 r )2 + ( r + 2 )2

4 ( r2 + 4r + 4 ) = 4r2 + r2 + 4r + 4

4r2 + 16r + 16 = 4r2 + r2 + 4r + 4

Al final nos queda la ecuación r2 – 12r – 12 = 0 donde D = 144 + 48 = 192 > 0 .

D = 192 = 26. 3 ; entonces podemos aplicar:

3462

3.812

2

3212

1.2

3.2)12(

.2

36

2,1

a

Dbr

Y se puede apreciar fácilmente que solo podemos tomar 13928,12346 r ,

porque si restamos, el resultado es negativo.

Calculamos el área sombreada aplicando diferencia de áreas entre el triángulo ABC y el sector

circular MCN.

rrr

rrrrr

AAA STS .2.4

14

.2

4

.

2

)2.(.2 22

22

22 6234,6226169.215,026169).785,01(13.213.4

14,31 cmAS

3.- De inicio las x tienen una primera condición, x 0 , que nos impone el texto del

ejercicio y que debemos considerar en nuestra respuesta final.

La expresión subradical tiene que ser mayor o igual que cero.

04

1

1615

88624

23

xxx

xxx , buscando el mínimo común denominador haremos la

descomposición en factores 04

1

)1)(4)(4(

8862

23

xxxx

xxx y se realiza la operación

indicada 0)1)(4)(4(

448862

2323

xxx

xxxxxx donde el numerador se puede expresar

por 492 2 xx y entonces 0)1)(4)(4(

)4)(12(2

xxx

xx.

Consideramos que x 4 ; simplificamos 0)1)(4(

)12(2

xx

x y hacemos el siguiente

análisis de los signos por intervalos:

+ –4 – ½ +

/////////////////////////////////////////////////////

Pero, según la condición inicial, x 0 , entonces la respuesta final está dada por las x reales

con las condiciones 2

1x ; x 4 .

Page 54: PRUEBA 1 - math.cubava.cumath.cubava.cu/files/2015/05/solucionario-libro-jacinto.pdf · g 9 2x 3 x 2 y ahora podemos ... llegamos a la conclusión parcial y = 8 ó y = 1 para los

54

4.-Designemos el referido número de dos cifras por n = 10.x + y

x : cifra de las decenas

y : cifra de las unidades

El nuevo número será entonces m = 10.( x – 4 ) + ( y + 4 )

Al restar nos queda n – m = 10.x + y – [ 10.( x – 4 ) + ( y – 4 ) ] = 10.y + x

Si realizamos correctamente todas las operaciones indicadas y transformaciones necesarias

llegaremos a la expresión 36 = 10.y + x lo que nos indica evidentemente que y = 3

y que x = 6.

Podemos decidir que el número original es 63. Prueba : 63 – 27 = 36 .

5.- Como DBGE es un cuadrado podemos calcular, pues conocemos su área, la longitud de

su lado ¨ a¨ que es la diagonal del rectángulo base del ortoedro.

a2 = 32 donde cma 242.1632 y no nos podemos olvidar que esta

longitud es también la altura h del ortoedro, por lo que cmh 24 .

a) Calculamos la diagonal del ortoedro cmaEB 82.42.2.42. .

b) Se puede comprobar que el triángulo AEB es rectángulo en A pues AD es la proyección

de AE sobre el plano base y ABAD (teorema de las tres perpendiculares en el punto A) .

Procederemos a calcular las longitudes de AE y de AB para poder calcular fácilmente su

área. En la base, el triángulo AOD es equilátero ya que, en principio, es isósceles pues

OADO (mitades de las diagonales iguales del rectángulo base) y tiene además un ángulo

interior de 600. Entonces podemos calcular la longitud de .222

24

22cm

aDBAD

Aplicamos el teorema de Pitágoras en el triángulo rectángulo ADE para calcular

2.162.424222222

EDADAE

cmAE 10240 .

Aplicamos el teorema de Pitágoras en el triángulo rectángulo ADB para calcular la longitud

de cmADDBAB 622483222242222

.

Calculamos el área del AEB (rectángulo en A)

2165,1524,2.73,1.45.3.45.2.3.2.22

.10.2.6.2

2

.cm

AEABAT

c)Calculamos ahora el volumen del ortoedro

37806,7873,1.41,1.323.2.3224.6.2.2.2... cmhABADhAV B

PRUEBA 29

1.-Es un ejercicio donde vale la pena hacer un cambio de variable y = 4x . Acomodaremos la

ecuación para hacer el cambio:

xxxx 474.4744.4.2 2

xxx 474.1674.9 , donde nos queda yyy 71679

Page 55: PRUEBA 1 - math.cubava.cumath.cubava.cu/files/2015/05/solucionario-libro-jacinto.pdf · g 9 2x 3 x 2 y ahora podemos ... llegamos a la conclusión parcial y = 8 ó y = 1 para los

55

y que ahora elevamos al cuadrado en ambos miembros para intentar eliminar las raíces

yyyyy 716)716)(79(.279 aislamos la raíz para volver a elevar al

cuadrado en ambos miembros 4949144224 2 yyy (pero antes dividiremos por 2

ambos miembros para obtener números más pequeños)

144 y2 = 144 y2 + 49 y – 49 donde nos queda 49 y = 49 , o sea, y = 1 .

Hacemos ahora reversible el cambio de variable 4x = 1 que se cumple solo si x = 0.

Comprobamos en la ecuación original y verificamos que es la solución pedida.

2.- Pocas veces nos encontramos con un ejercicio de geometría plana que nos conduzca a tener

que resolver una desigualdad, esto se puede apreciar rápidamente en el contexto mismo del

ejercicio ya que nos están acotando el área del rombo.

Diagonal mayor : D

Diagonal menor : d pero D = 2.d + 5 entonces en la fórmula para calcular el área del

rombo RA

dD

2

. podemos escribir 5,31

2

).52(

dd que se puede multiplicar por 2

en ambos miembros pues 2 > 0 y nos conduce a una desigualdad más simplificada

2d2 + 5d 63 que comparamos con cero y después podemos descomponer en factores

( 2d – 9 )( d + 7 ) 0 para analizar los intervalos

+ –7 – 9/2 +

//////////////////////////

donde podemos llegar a resultados con respecto a la diagonal menor que no puede ser

negativa pues es la longitud de un segmento y no puede ser nula porque no existiría tal rombo

0 < d 9/2 aquí podemos multiplicar por 2 todos los términos

0 < 2d 9 podemos sumar 5 a todos los términos

5 < 2d + 5 14 y sustituimos 2d + 5 = D

5 < D 14 que son los únicos valores que puede tomar D para que se

cumpla que AR 31,4 cm2.

Otra vía: Se puede trabajar con ecuaciones en los extremos del rango del área.

3.- Se prueba fácilmente que AED CEB ya que D = B (inscritos sobre el mismo

arco AC y además AED = CEB (opuestos por el vértice). Tienen entonces dos ángulos

respectivamente iguales. Podemos calcular la razón de semejanza en dos segmentos

homólogos (se oponen a ángulos iguales en triángulos semejantes) 24

8

AD

CBk

podemos calcular la longitud de EB = k . ED = 2.3,7 = 7,4cm

y ahora podemos calcular la longitud de EBABAE = 8,4 – 7,4 = 1 cm.

Necesariamente EC = 2 cm (es el lado homólogo de AE ) por lo que

EDECDC = 2 + 3,7 = 5,7 cm.

Page 56: PRUEBA 1 - math.cubava.cumath.cubava.cu/files/2015/05/solucionario-libro-jacinto.pdf · g 9 2x 3 x 2 y ahora podemos ... llegamos a la conclusión parcial y = 8 ó y = 1 para los

56

4.-Vamos a calcular, específicamente, dónde la expresión es igual a cero para luego

responder con el complemento según el intervalo universo, en este caso es el intervalo dado al

inicio,

[ 0 ; 2 ]. Escribimos, sustituyendo ya en algunos términos,

0sen

cos

cos.sen2

)sen(cos1 22

x

x

xx

xx y eliminamos denominadores x k/2 ; k entero

0cos.2sencos1 222 xxx y al sustituir sen2x = 1 – cos2x llegamos a que

0cos2cos1cos1 222 xxx que nos sorprende con 2 = 0 lo que es evidentemente

imposible, por lo que esa expresión nunca toma valor cero.

La respuesta es: Se cumple para toda x del intervalo dado, con la única condición que estas

x no pueden ser los ángulos axiales, o sea, x k/2 con k = 0 ;1 ; 2 ; 3 ; 4 .

5.- a)Utilizando la fórmula para el volumen del cilindro podemos calcular el radio del

recipiente pues conocemos el volumen de las granallas de Zn que hacen, al ser tapadas por el

agua, que la altura del agua se desplace 2 dm . En la fórmula V = AB . h sustituyendo nos

queda 157 = .r2.2 donde el único valor desconocido es r .

2528,6

157

14,3.2

157

.2

1572

r el radio del recipiente mide r = 5 dm.

b)Podemos escribir AT = 2.AL ,o sea, 2.AB + AL = 2.AL y podemos llegar a la relación

más sencilla 2.AB = AL

2..r2 = 2..r.h que nos conduce a que h = r 0

VC = .r2.h = 3,14 . r3 = 3,14 . 53 = 3,14 . 125 = 392,5 dm3 = 0,39 m3.

PRUEBA 30

1.- Hay ecuaciones que pueden ser resueltas mediante un simple análisis de expresiones que

aparecen en la misma ecuación, esto no quiere decir que no se puedan emplear métodos

algorítmicos que también nos llevan a obtener, o tal vez no, las posibles soluciones. En el

miembro derecho de esta ecuación, la cantidad subradical se puede escribir como

– cos2x 0 , para que la raíz esté definida solo puede ocurrir que cos2x = 0 , expresión

donde necesariamente cos x = 0 y esto se cumple solo si los valores de x = k/2 ; k

entero . En el intervalo que propone el ejercicio aparece el valor x = /2 como único que

puede tomar la variable. Nos queda ahora verificar si ese único valor de x satisface la

ecuación dada. Realizamos la prueba y verificamos que la solución es x = /2 . Si usted

quiere, puede entrenar sus habilidades algorítmicas y llegará a la ecuación final

cos4x + cos2x = 0 que lo conducirá directamente a obtener la misma respuesta.

2.- Se puede demostrar que ABD DEC ya que ambos triángulos son rectángulos y se

cumple también que ABD = CDE (alternos entre las paralelas DC y AB ) entonces los

triángulos tienen dos ángulos respectivamente iguales.

Para poder calcular el área del triángulo DEC necesitamos obtener más informaciones del

mismo o, por lo menos, conocer las longitudes de sus catetos EC y DE .

Trazamos la construcción auxiliar ABCF y se forma el rectángulo AFCD.

Page 57: PRUEBA 1 - math.cubava.cumath.cubava.cu/files/2015/05/solucionario-libro-jacinto.pdf · g 9 2x 3 x 2 y ahora podemos ... llegamos a la conclusión parcial y = 8 ó y = 1 para los

57

Por resta de segmentos podemos calcular que FB = 3cm. En el triángulo rectángulo CFB,

su hipotenusa CB = 5 cm, entonces, el otro cateto CF = 4 cm (trío de números pitagóricos y

no es necesario calcular).

Para poder establecer la razón de proporcionalidad entre los lados de los triángulos semejantes

necesitamos calcular la hipotenusa del triángulo ABD, pues del otro triángulo únicamente

conocemos su hipotenusa.

Aplicamos el teorema de Pitágoras y calculamos su longitud,

.104)19(443.4412 22222222

cmADABDB

Podemos escribir DE

AB

EC

AD

DC

DB y sustituimos

DEEC

124

9

104 donde haremos los

despejes adecuados y obtenemos que:

10

109

104

9.4EC ;

10

1027

104

12.9DE

Ya podemos calcular el área del triángulo CDE.

21215,1220

27.9

2

10

1027.

10

109

2

.cm

DEECAT .

(Otra forma para calcular el área: Calcular el área del triángulo ABD , dividirla por la razón

de proporcionalidad k2 > 1 entre las áreas y obtenemos el área del DEC)

3.- Primeramente resolveremos la desigualdad en su variable.

Comparamos con cero 01832

2

2

yy

yy efectuamos en el miembro izquierdo

0832

2

22

yy

yyyy reducimos los términos semejantes y obtenemos

082

2

2

yy

yy que se puede descomponer en factores 0

)1(

)2)(4(

yy

yy .

Analizamos los signos de los intervalos a partir de sus ceros y de los valores que la indefinen.

+ –2 – 0 + 1 – 4 +

///////////////////OO///////////////////////////

De donde se obtienen los intervalos –2 y < 0 ó 1 < y 4 . Ahora buscamos las

respuestas para la variable x en: – 2 2x < 0 ó 1 < 2x 4 .

( Imposible) 0 < x 2 solamente.

4.-Las variables por las cantidades de billetes.

Cantidad de billetes de $ 3 : x x + y + z = 22

Cantidad de billetes de $ 5 : y 3x +5y +10z = 157

Cantidad de billetes de $10 : z z – x = y

Pero podemos apreciar rápidamente que z = x + y sustituimos en la primera ecuación y

calculamos de inicio que 2z = 22 por lo que z = 11.

Page 58: PRUEBA 1 - math.cubava.cumath.cubava.cu/files/2015/05/solucionario-libro-jacinto.pdf · g 9 2x 3 x 2 y ahora podemos ... llegamos a la conclusión parcial y = 8 ó y = 1 para los

58

Si sustituimos su valor en la primera y en la segunda ecuación tendremos un pequeño

sistema :

x + y = 11 .(–3) ; si adicionamos obtenemos 2y = 14 donde y = 7.

3x + 5y = 47

Podemos calcular que en billetes de $ 5 hay 7. $ 5 = $ 35,00.

5.- Con el volumen de la pirámide y el área de su base podemos calcular su altura.

3

.144

3

.384

hhAV B

P donde cmh 8144

3.384 . En la figura h = OP y

consideraremos que M es el punto medio de la arista de la base a = BC .

Podemos calcular la longitud de BC pues a2 = 144 entonces a = 12cm.

Trabajaremos en el triángulo rectángulo OPM de apoyo de la cara lateral de la pirámide

(todas son caras laterales iguales, triángulos isósceles que tienen por base a = 12cm)

Calculamos cma

OM 62

12

2 porque la pirámide es recta.

Calculamos la altura de la cara lateral aplicando el teorema de Pitágoras o la propiedad de los

tríos de números pitagóricos. Catetos de 6 y 8 entonces la hipotenusa es 10.

cmhC 10

El área total del ortoedro:

26723842888.12.4144.2.4.2 cmAAA CLBO

El área total de la pirámide:

23842401442

10.12.4144.4 cmAAA TLBP

La razón: 7

4

672

384

O

P

A

AR

Page 59: PRUEBA 1 - math.cubava.cumath.cubava.cu/files/2015/05/solucionario-libro-jacinto.pdf · g 9 2x 3 x 2 y ahora podemos ... llegamos a la conclusión parcial y = 8 ó y = 1 para los

59

PRUEBA 31

1.- Aplicando las leyes de los logaritmos, la ecuación se puede expresar por:

log7 [( 2x – 1 )( 2x – 7 )] = log7 7 y, como la función logarítmica es inyectiva en

todo su dominio, podemos igualar los argumentos.

( 2x – 1 )( 2x – 7 ) = 7 ,donde , al efectuar nos queda 22x – 7.2x – 2x + 7 = 7

que se puede expresar como 22x – 8.2x = 0 y se puede factorizar 2x.( 2x – 8 ) = 0 .

2x = 0 (imposible) ó 2x = 8 que nos lleva a obtener x = 3 .

2.- No debemos olvidar la primera condición que nos impone el ejercicio, x < 0 .

El denominador de la fracción se puede descomponer en factores utilizando un conveniente

agrupamiento y luego trabajando con el factor común.

( x3 + 4x2 ) – ( x + 4 ) = x2 ( x + 4 ) – ( x + 4 ) = ( x + 4 ) ( x2 – 1 ) .

En el numerador se descompone la suma de cubos.

x3 + 1 = ( x + 1 )( x2 – x + 1 ) [este último trinomio no admite descomposición].

Podemos escribir entonces 0)1)(1)(4(

)1)(1( 2

xxx

xxx y simplificarla considerando que

x – 1 , 0)1)(4(

12

xx

xx y hacer el análisis de los signos por intervalos,

+ –4 – 1 +

O/////////////////////////////O

como solamente podemos considerar las que sean negativas entonces la respuesta está dada

por x reales – 4 < x < 0 con x – 1 .

3.- En la figura, todos los lados del cuadrado miden 6 cm debido a que el área del mismo es

36 cm2. Por resta de segmentos podemos calcular la longitud del segmento FC = 12 cm .

Aplicando el teorema de las transversales, pues EF AB , podemos plantear la proporción

EF

FC

AB

AC y sustituir

6

1218

AB para calcular la longitud del segmento

cmAB 912

18.6.

Aplicamos el teorema de Pitágoras en el triángulo rectángulo ABC y calculamos la longitud de

su hipotenusa,

cmABACBC 59)14.(992.9918 22222222

Podemos calcular el perímetro del triángulo ABC.

BCACABP = 9 + 18 + 9.2,24 = 27 + 20,16 = 47,16 cm = 47 cm.

Page 60: PRUEBA 1 - math.cubava.cumath.cubava.cu/files/2015/05/solucionario-libro-jacinto.pdf · g 9 2x 3 x 2 y ahora podemos ... llegamos a la conclusión parcial y = 8 ó y = 1 para los

60

4.- Designamos las variables

Precio que pagan los mayores : x Precio que pagan los niños : y

Podemos, con la información dada, plantear el sistema

Primera función 144x + 240y = 264 : 8

Segunda función 180x + 150y = 255 : 5

El sistema simplificado 18x +30y = 33

36x + 30y = 51 Restamos las ecuaciones y obtenemos que

18y = 18 , o sea, y = 1 , podemos calcular x = 0,5 . Ya tenemos los precios y ahora

podemos dar respuesta al problema para los que piensan ir a la tercera función:

6 . $ 1,00 + 123 . $0,50 = $ 6,00 + $ 61,50 = $ 67,50

Deben pagar en conjunto $ 67,50 .

5.- a)Lo primero que se aprecia es que el triángulo OPT (en la base) es rectángulo en T (radio

en el punto de tangencia) y hasta podemos decidir que OP = r = 5 cm (trío de números

pitagóricos con 12 y 13) .

Como conocemos el área lateral del cono podemos ahora calcular la altura

ggrAL .5.14,3..220 despejamos cmg 14012,145.15,3

220

ST = g = 14cm . Pero podemos probar que el triángulo PST , del cual nos piden calcular su

área, es rectángulo en T. OT es la proyección de ST sobre la base y se cumple además que

PTOT entonces PTST (teorema de las tres perpendiculares en el punto T). Calculamos

su área,

284

2

14.12

2

.cm

STPTAT

b)El triángulo SOT es rectángulo en O ( OS es la altura del cono) entonces podemos aplicar

el teorema de Pitágoras para calcular la altura h = OS del cono.

cmOTSTh 19319.917125196514 2222

Calculamos el volumen del cono,

33

22

34,03,3423

36,4.3.25.14,3

3

19.3.5.14,3

3

..

3

.dmcm

hrhAV B

C

PRUEBA 32

1.-Una aclaración importante: Atendiendo al rigor matemático y a la estética, el argumento

del logaritmo se debe escribir siempre en la misma línea, o sea, a la misma altura, que se

escribe la expresión log .

La ecuación original, trabajando con las propiedades o leyes de las operaciones con

logaritmos y las potencias, tratando de llevar todo a la misma base, se puede escribir como:

242.28log)2(log 33

xx , o sea, 22

8log)2(log 33 xx

Page 61: PRUEBA 1 - math.cubava.cumath.cubava.cu/files/2015/05/solucionario-libro-jacinto.pdf · g 9 2x 3 x 2 y ahora podemos ... llegamos a la conclusión parcial y = 8 ó y = 1 para los

61

Como la función exponencial es inyectiva en todo su dominio podemos igualar los

exponentes 18log)2(log 33 xx y se puede llevar a un solo logaritmo escribiendo

1]8).2[(log 3 xx donde aplicamos la operación inversa y podemos escribir

38).2( xx . Ahora podemos elevar al cuadrado ambos miembros para eliminar la raíz

9)8.()2( 2 xx realizamos la operación inmediata ( x2 – 4x + 4 )( x + 8 ) – 9 = 0 y

después de multiplicar y reducir los términos semejantes llegamos a la nueva ecuación

x3 + 4x2 – 28x + 23 =0 que se puede descomponer en factores utilizando la regla de Ruffini

mediante la expresión

( x – 1 )( x2 + 5x – 23 ) = 0 . Intentamos descomponer en factores el trinomio y al calcular

D = b2 – 4ac = 52 – 4.1.(–23) = 25 + 92 = 117 que no tiene raíz cuadrada exacta y entonces la

despreciamos porque el ejercicio solo nos pide las soluciones que sean enteras, por lo que

x = 1 es la única posible solución en el conjunto de los números enteros. Hacemos la

comprobación y vemos que x=1 es solución.

2.- En principio BTA = 900 (inscrito sobre el diámetro, teorema de Tales) . Como MTAT

y se cumple que MN AT entonces MTMN (si dos rectas son paralelas y una de ellas es

perpendicular a una tercera entonces la otra también es perpendicular a la tercera) lo que nos

indica que el triángulo MNT es rectángulo en M .

Se cumple entonces que MTN ABT , ambos son rectángulos y además tenemos que

BAT = MTN (son respectivamente inscrito y semiinscrito sobre el mismo arco en la

circunferencia) , los triángulos tienen dos ángulos respectivamente iguales.

En el triángulo MTN podemos calcular la longitud de MT aplicando el teorema de Pitágoras

o analizando directamente que mide 8 cm (trío de números pitagóricos con 6 y 10) y ahora

podemos calcular también que BT = 4 cm (porque M es punto medio del segmento

MT = 8 cm ). Escribimos las razones entre los segmentos homólogos

MT

AT

MN

BT

NT

AB y sustituimos

86

4

10

ATAB donde podemos ahora calcular las

longitudes cmAB3

20

6

4.10 ; cmAT

3

16

6

8.4 y r = cm

3

10

a)La longitud de la circunferencia : cmABL 2193,2014,3.3

20.

b)El área del triángulo BAT : 21166,10

2.3

4.16

2

4.3

16

2

.cm

BTATAT

3.-Consideremos por x el ancho de las carreteras. Podemos escribir la superficie de las

carreteras como 67500m2 . Obtenemos la ecuación de la superficie total de las carreteras

dentro del terreno rectangular,

67500 = 800.x + 600.x – x2

(hemos restado al final x2 , que es el área del cuadrado que nos queda en el centro de la figura;

si no lo hacemos lo tendríamos considerado dos veces en 800.x + 600.x ).

Page 62: PRUEBA 1 - math.cubava.cumath.cubava.cu/files/2015/05/solucionario-libro-jacinto.pdf · g 9 2x 3 x 2 y ahora podemos ... llegamos a la conclusión parcial y = 8 ó y = 1 para los

62

Podemos escribir la ecuación como x2 – 1400x + 67500 = 0 en la cual nos resulta

un poco difícil hallar a golpe de vista una descomposición en factores.

D = b2 – 4ac = (–1400)2–4.67500 = 1960000 – 270000 = 1690000 > 0 (dos soluciones reales) ;

[ 13001690000 D ]

2

13001400

2

1300)1400(

.22;1

a

Dbx ; x1 = 1350 m (imposible)

x2 = 50 m

El ancho de las carreteras es 50 m .

4.- a) Podemos apreciar que el denominador en la expresión nunca va a tomar el valor cero

porque tan22x 0 para todo valor x de su dominio, entonces podemos escribir que

1 + tan22x > 0 . Única posibilidad de valores inadmisibles 2x = /2 + k ; con k número

entero, y que al despejar x nos aporta x = /4 + k/2 ; k entero.

b)El miembro derecho de la expresión, debido a la fórmula del coseno para los ángulos

negativos, podemos escribirlo como cos (– 4x ) = cos 4x . Partimos del miembro izquierdo

xxxx

x

xx

x

xx

x

x

x

x

4cos)]2(2cos[2sen2cos

2cos

2sen2cos

2cos

2sen2cos

2cos

2sen1

2cos

2sen1

22

2

22

2

22

2

2

2

2

Queda probada la identidad trigonométrica.

5.-En principio podemos calcular la longitud de la portería pues conocemos el área del

rectángulo de entrada ABCD y la altura de los postes verticales.

AE = 3,6 = 5,1.. ABADAB despejamos mAB 4,25,1

6,3

Los triángulos laterales son triángulos rectángulos iguales. Trabajaremos con el triángulo

ADE , rectángulo en A . Debemos calcular las longitudes de los segmentos AE y DE .

Podemos plantear DE

ADAED sen y despejar para calcular

cmAED

ADDE 3

3

3

2

3

5,1

sen

y entonces .

2

3cmAE pues se opone

al ángulo de 300 .

a)La cantidad de malla que se necesita para forrar los laterales es Am= 2.AT + ADEFC

25,545,515,3.73,1)4,22

5,1.(73,173,1.4,2

2

73,1.5,13.4,2

2

2

3.5,1

.2 mAm

Page 63: PRUEBA 1 - math.cubava.cumath.cubava.cu/files/2015/05/solucionario-libro-jacinto.pdf · g 9 2x 3 x 2 y ahora podemos ... llegamos a la conclusión parcial y = 8 ó y = 1 para los

63

b)El volumen de toda la estructura es

36,1557,14

4,2.73,1.5,14,2.

4

3.5,1.. mhAhAV TB

PRUEBA 33

1.- Tenemos que igualar las funciones, o sea, f(x) = g(x) en el dominio dado.

4

1cos.sensen.cos 33 xxxx donde

4

1cos.sensen.cos 33 xxxx

podemos extraer factor común cos x . sen x en el miembro izquierdo

4

1)sen(cossen.cos 22 xxxx y expresarla

2

1)sen(coscos.sen.2 22 xxxx

para convertirla en 2

12cos.2sen xx y luego 2 sen2x.cos2x = 1 de donde podemos

llegar a que sen 4x = 1 por lo que 4x = /2 + 2k ; k entero , o sea ,

x = /8 + k/2 ; k entero . Trabajamos con la periodicidad k/2 y podemos calcular con

k = 0 hasta k = 1 pues k = 2 nó nos da valores dentro del intervalo inicial del ejercicio.

Soluciones /8 ó 5/8.

2.- Podemos expresar directamente el área sombreada que nos piden calcular, como la suma

del área del triángulo rectángulo y el área del sector circular.

ASOM = AT + ASEC pero para realizar el cálculo carecemos de algunos elementos.

Tenemos que ABD = 300 (complementario con ADB) y DBC = 600 ( ahora

complementario con ABD) . El CBE = 1200 (adyacente con DBC = 600).

Calculamos AD = 5 cm (se opone al ángulo de 300 en el triángulo rectángulo ABD entonces

es la mitad de la hipotenusa DB = 10 cm) y además AB = .35 cm (se opone al ángulo de

600) . Podemos escribir DCAB y BCAD (lados opuestos del rectángulo)

2,266,213

25.14,3

2

73,1.25

3

5.14,3

2

5.3.5

360

120..

2

. 2

0

02

BCBCDC

ASOM

.488,47 2cmASOM

3.-Mucho cuidado si usted no conoce las diferencias que existen entre estos tres tipos de

animales. Los camellos tienen dos gibas y el dromedario tiene solo una; el avestruz no tiene

giba y tiene solo dos patas.

Tomaremos las variables, para mayor comodidad, con la primera letra del animalito.

Ecuación de la cantidad de los animales : a + d + c = 20

Ecuación de las patas de los animales : 2a + 4d + 4c = 70

Ecuación de las gibas : d + 2c = 23 donde d = 23 – 2c .

Page 64: PRUEBA 1 - math.cubava.cumath.cubava.cu/files/2015/05/solucionario-libro-jacinto.pdf · g 9 2x 3 x 2 y ahora podemos ... llegamos a la conclusión parcial y = 8 ó y = 1 para los

64

Sustituimos en las dos primeras ecuaciones la variable d y entonces quedan las ecuaciones

a – c = – 3 las restamos ordenadamente y nos queda – c + 2c = – 3 + 11

a – 2c = –11 c = 8

entonces d = 23 – 2.8 = 23 – 16 = 7 ; a = 20 – 8 – 7 = 5

Hay 5 avestruces , 8 camellos y 7 dromedarios.

4.- La expresión está definida cuando 03

1222323

xx

x

xx

x . Estos denominadores se

pueden descomponer en factores para calcular el mínimo denominador común y poder calcular

la diferencia en el miembro izquierdo,

x3 + x2 = x2 ( x + 1 ) ; x3 – 3x2 = x2 ( x – 3 ) entonces podemos efectuar la diferencia

0)3)(1(

)12)(1()2)(3(2

xxx

xxxx y en el numerador, después de efectuar y reducir los

términos semejantes queda – x2 + 5x – 6 – 2x2 – x + 1 = – 3x2 + 4x – 5 que para

descomponer más fácilmente multiplicaremos la desigualdad en ambos miembros por ( – 1 ) y

entonces cambiaremos el sentido de la desigualdad,

0)3)(1(

5432

2

xxx

xx y al intentar descomponer el numerador, para nuestra sorpresa, ocurre

que D = b2 – 4ac = (– 4)2–4.3.5= 16 – 60 = – 44<0 lo que nos indica que ese trinomio, como

a > 0 , siempre toma valores positivos y no tenemos necesidad de analizarlo. La expresión

nos conduce al siguiente análisis de signos por intervalos:

+ –1 – 0 – 3 +

O/////////////O//////////////////////////O

Podemos dar la respuesta: Está definida para x reales con – 1 < x < 3 ; x 0.

5.-En el cilindro se cumple que AL = AB , entonces 2..r.h = .r2 , pero podemos hacer en

esta igualdad simplificaciones , por lo que 2.h = r .

Como h = 4 dm podemos concluir que r = 8 dm .

Calculamos su área lateral

AL = 2..r.h = .r2 = 3,14.82 = 3,14.64 = 200,1dm2 = 2,0m2

Calculamos el volumen

V = AB.h = 200,1.4 = 800,4dm3 = 0,8 m3.

PRUEBA 34

1.- Analizaremos primero si existen puntos de intersección entre las dos curvas planteando el

sistema de ecuaciones

– 15x2 + 9y2 = 9 : (– 3)

2x2 + 3y2 = 66

o sea, 5x2 – 3y2 = – 3

2x2 + 3y2 = 66 al sumar las ecuaciones nos queda 7x2 = 63 en la que podemos

hacer el despeje de la variable x2 = 9 para calcular los posible valores de esta variable

x = 3 ó x = – 3 que si los sustituimos en cualquiera de las ecuaciones cuadráticas anteriores

Page 65: PRUEBA 1 - math.cubava.cumath.cubava.cu/files/2015/05/solucionario-libro-jacinto.pdf · g 9 2x 3 x 2 y ahora podemos ... llegamos a la conclusión parcial y = 8 ó y = 1 para los

65

podemos encontrar cuatro puntos de intersección entre estas dos curvas

( 3 ; 4 ) ; ( 3 ; – 4 ) ; ( –3 ; 4 ) y ( –3 ; – 4 ) . Ahora tenemos que analizar cuál de estos

cuatro puntos pertenece a la recta de ecuación 5x – 3y = –3 . Haciendo las sustituciones

pertinentes con los correspondientes cálculos llegamos a la conclusión de que el único punto

que responde la pregunta del ejercicio es ( –3 ; – 4 ) , observemos;

5( –3) – 3 (– 4) = –15 + 12 = – 3 .

Otra vía: Se pudo trabajar al inicio con la recta y una de las dos curvas para luego comprobar

los puntos que se obtienen en la ecuación de la otra curva.

2.- AD es la altura relativa a la base en el triángulo isósceles por lo que D es el punto medio

de la base y entonces BD = 3 cm. Como el triángulo ABD es rectángulo en D podemos

calcular o decidir que AB = 5 cm (trío de números pitagóricos 3 ; 4 y 5) .

Se cumple que ABD BDE pues ambos triángulos son rectángulos y además tienen un

ángulo común, el B ; tienen entonces dos ángulos respectivamente iguales.

Calculamos la razón de proporcionalidad y lo hacemos entre un par de lados que sean

homólogos 5

3

AB

BDk y para hallar el perímetro del triángulo BDE aplicamos la

relación PBDE = k . PABD = cm2,75

12.3)543.(

5

3 .

Para calcular el área aplicamos la relación

ABDE = k2 . AABD = 2

2

2,216,250

108

2.25

12.9

2

4.3.

5

3cm

3.- Los sacos que recogieron los alumnos, por día durante toda la semana, se pueden expresar

todos referidos a la cantidad del primer día realizando ese importante trabajo.

lunes : x ; martes : x + 5 ; miércoles : ( x +5) + 5= x +10 ; jueves : x +15

viernes : x + 20 ; sábado : x + 25 y el domingo : x + 30 .

Según el texto del problema podemos escribir la ecuación xx .8

1130 ; que puede ser

transformada en xx 112408 , y al resolverla obtenemos x = 80.

Ya podemos dar respuesta a las preguntas que nos hace este problema, que como podemos

apreciar, no es el resultado que nos entrega la variable x de su ecuación .

a)Una simple suma por día nos da la cantidad de sacos que ese grupo de alumnos recogió en

toda la semana 80 + 85 +…+ 110 = 665.

b)Calculamos ahora el tanto por ciento que representó la cantidad de sacos que se recogieron

el domingo %5,16100.165,0100.665

110P

Se recogieron en total 665 sacos de papas. Lo recogido el domingo fue 16,5% del total.

Page 66: PRUEBA 1 - math.cubava.cumath.cubava.cu/files/2015/05/solucionario-libro-jacinto.pdf · g 9 2x 3 x 2 y ahora podemos ... llegamos a la conclusión parcial y = 8 ó y = 1 para los

66

4.- a)No está definida, únicamente, cuando el denominador sen x + cos x = 0 , o sea, cuando

se cumple que sen x = – cos x que para concluir podemos llegar a una ecuación más

directa, tan x = – 1 , donde la variable x solo permite que le asignen ángulos de los

cuadrantes II y IV. No está definida para x = 3/4 + k ; k entero.

b)Vamos a partir del miembro izquierdo de la igualdad y haciendo transformaciones,

intentaremos llegar al segundo,

xx

xx

xxxxxx

xx

xxcos.sen1

cossen

)coscos.sen)(sencos(sen

cossen

cossen 2233

2

2sen2

2

cos.sen.22

2

)cos.1.(2 xxxxsex

. Queda probada.

5.- En la fórmula para calcular el perímetro del círculo base podemos hacer sustituciones y

después despejar el radio del círculo, rrLO .14,3.2..22,94

cmr 1528,6

2,94

14,3.2

2,94 . Como el cono es recto, entonces, dos generatrices

diametralmente opuestas y el diámetro de la base forman entre sí un triángulo isósceles;

en este caso los ángulos en la base medirán cada uno 300 ( suma de los ángulos interiores de

un triángulo isósceles ). En el triángulo rectángulo que forman entre sí la altura h del cono, el

radio, y la generatriz g , podemos calcular esta última pues 2

31530cos 0

gg

r y

entonces cmg 3103

3.30

3

15.2 .

Calculamos también cmh 352

310 (se opone al ángulo de 300, es la mitad de la

hipotenusa g ).

Calculamos el área lateral del cono,

22 1,88,81473,1.471310.15.14,3.. dmcmgrAL .

Calculamos el volumen,

3322

04,220373

73,1.5.225.14,3

3

3.5.15.14,3

3

..

3

.dmcm

hrhAV B

.

Page 67: PRUEBA 1 - math.cubava.cumath.cubava.cu/files/2015/05/solucionario-libro-jacinto.pdf · g 9 2x 3 x 2 y ahora podemos ... llegamos a la conclusión parcial y = 8 ó y = 1 para los

67

PRUEBA 35

1.- El miembro derecho de la ecuación puede ser transformado y obtener una expresión más

simplificada, cos 2x + 2sen2x = cos2x – sen2x + 2sen2x = cos2x + sen2x = 1.

Elevamos al cuadrado en ambos miembros de la ecuación transformada,

1coscos1

cos.sen2

cos1

sen 22

xx

xx

x

x donde aislamos la raíz para volver a elevar al

cuadrado en ambos miembros x

xx

x

xx

cos1

cos1cos1

)cos1(

cos).cos1(2

22

pero antes

simplificamos de nuevo el miembro derecho para facilitar el trabajo, y entonces nos queda

1 – cos x – ( 1 – cos x ) = 1 – cos x – 1 + cos x = 0 . Nos percatamos de que podemos

escribir directamente que la cantidad subradical tiene que ser necesariamente nula, por lo que

la ecuación ahora nos quedará más simplificada,

( 1 – cos x ). cos x = 0 donde cos x = 1 ó cos x = 0 y hallar los posibles ángulos que la

satisfacen x = 2k ó x = /2 + k ; k entero . Al comprobar en la ecuación original

verificamos que todas son soluciones.

2.- En el triángulo rectángulo BCE podemos aplicar el teorema de Pitágoras y calcular la

longitud de cmEBCECB 9812,298,514,52,7 2222

Se cumple que BCE ABD ya que ambos triángulos son rectángulos y además

ECB=ABD (ángulos agudos con sus lados respectivamente perpendiculares).Tienen dos

de sus ángulos respectivamente iguales. Calculamos la razón de semejanza k entre un par

de lados homólogos 3

5

54

90

4,5

9

EB

ADk y podemos ahora calcular la longitud del

segmento cmECkAB 122,7.3

5. . Calculamos el área del rectángulo,

AR= CBAB. = 12 . 9 = 108 cm2 = 1,1 dm2.

3.- Vamos a considerar las variables Precio de cada libro : x

Precio de cada revista : y

Formamos el sistema de ecuaciones 8x + 5y = 13,60 . (– 2)

2x + 10y = 10,40

–16x – 10y = – 27,20

2x + 10y = 10,40 que al sumar ordenadamente las dos ecuaciones nos lleva a que

– 14x = – 16,80 , o sea, x = 1,20 , por lo que calculamos que y = 0,80.

Ya podemos responder nuestro problema, considerando que hay que comprar de los dos

artículos. 2. $ 1,20 + 2. $0,80 = $ 2,40 + $ 1,60 = $ 4,00.

Se pueden comprar 2 libros y 2 revistas gastando exactamente $ 4,00. (no hay otra

posibilidad).

Page 68: PRUEBA 1 - math.cubava.cumath.cubava.cu/files/2015/05/solucionario-libro-jacinto.pdf · g 9 2x 3 x 2 y ahora podemos ... llegamos a la conclusión parcial y = 8 ó y = 1 para los

68

4.- Podemos calcular, aplicando propiedades, el valor numérico del miembro derecho de

la

desigualdad 52

10

10

1010

2log

2log1 . Entonces comparamos con cero y llegamos a la

desigualdad 01162

2

2

xx

xx donde efectuamos la diferencia y obtenemos en el

numerador la expresión 2x2 + x – 16 – x2 – x por lo que podemos escribir la desigualdad

como 016

2

2

xx

x que descomponemos en factores en el numerador y denominador

mediante 0)1.(

)4).(4(

xx

xx .

Hacemos el siguiente análisis de los signos por intervalos,

+ –4 – –1 + 0 – 4 +

///////////////////OO///////////////////////////

No podemos olvidar que al comienzo del ejercicio nos imponen la condición x < 2.

Respondemos que se cumple para las x reales con – 4 x < –1 ó 0 < x < 2.

5.- Tenemos que utilizar, en el momento oportuno, la fórmula para el área total de la

pirámide. En este caso es una pirámide recta de base cuadrada. AT = AB + 4ALAT.

En cualquiera de las caras laterales (triángulos isósceles) podemos trazar la altura relativa a la

base, que la divide en dos partes iguales y en cualquiera de los triángulos rectángulos iguales

que se forman podemos aplicar el teorema de Pitágoras e intentar calcular la longitud de

44

5.

22

5

2

22222

2 aaaaaLha

aa

ha 4

.4 2

que es una buena relación.

Page 69: PRUEBA 1 - math.cubava.cumath.cubava.cu/files/2015/05/solucionario-libro-jacinto.pdf · g 9 2x 3 x 2 y ahora podemos ... llegamos a la conclusión parcial y = 8 ó y = 1 para los

69

En la fórmula para el área total sustituimos adecuadamente y llegamos a la ecuación

siguiente:

22222 32..22

..427 aaaaaa

haaA a

T donde podemos despejar y calcular

la longitud de dma 393

27 y entonces escribir dmha 3 .

En el triángulo rectángulo de apoyo de cualquiera de las caras laterales podemos aplicar el

teorema de Pitágoras y calcular la longitud de la altura de esta pirámide,

2

3.

4

.3

42)(

222

2

2 aaaa

ahh aP

.

Calculamos el volumen,

333

2

8,7785,76

73,1.27

6

73,1.3

6

3.

3

2

3..

3

.dm

a

aa

hAV PB

P

.

PRUEBA 36

1.- Para comenzar a trabajar haremos un reagrupamiento de los términos

x

xx

332log23log 2

22

2 , aplicando las propiedades de los logaritmos

podemos llegar a una ecuación con un solo logaritmo,

x

xx

323.23log 22

2 . Debemos observar que cuando efectuemos el producto

indicado en el argumento del logaritmo (conduce a una diferencia de cuadrados) llegaremos a

una ecuación que ya estuvimos trabajando en la pregunta # 1 de la prueba # 1 , pero en este

caso el punto de partida, o sea, la ecuación original, es distinta y esto se debe tener presente en

nuestra respuesta final, veamos,

Page 70: PRUEBA 1 - math.cubava.cumath.cubava.cu/files/2015/05/solucionario-libro-jacinto.pdf · g 9 2x 3 x 2 y ahora podemos ... llegamos a la conclusión parcial y = 8 ó y = 1 para los

70

xx 329log 2 y ahora podemos aplicar la operación inversa y convertirla a

exponencial xx 3229 , donde, x

x

2

829 , podemos eliminar el denominador

822.9 2 xx . Al ordenar convenientemente para descomponer luego en factores aparecen

los pasos siguientes, 082.922 xx ; 0)12).(82( xx , donde 2x = 8

ó 2x = 1 y podemos calcular que x = 3 ó x = 0. Se pueden comprobar las soluciones.

2.- Se debe hacer un pequeño esbozo de la situación. Calculamos las coordenadas del punto

medio M del segmento AB ,

2;

2

BABA yyxxM que con un cálculo rápido podemos

determinar que

3;

2

3M . Calcularemos la pendiente de la recta que contiene a los puntos

A y B, 23

6

30

061

AB

AB

rxx

yym . Pero las rectas r1 y r2 son perpendiculares,

(r2 es la mediatriz del segmento AB ) entonces se cumple que

mr1 . mr2 = – 1 y podemos calcular 2

1

2

12

rm .

Podemos hallar la ecuación de la recta r2 :

2

3.

2

13 xy que se puede transformar en

x – 2y + 9/2 = 0 para poder hallar más fácilmente los interceptos con los ejes de coordenadas

sustituyendo indistintamente.

Con el eje X : y = 0 entonces x = – 9/2. El punto es (–9/2 ; 0).

Con el eje Y : x = 0 entonces – 2y = – 9/2 . El punto es (0 ; 9/4).

3.- Al final del ejercicio no debemos olvidar que se nos impone x – 2 .

Para que esté definida la expresión tenemos que considerar que:

0213

823

2

xx

xx y descomponer completamente en factores utilizando para el

denominador la regla de Ruffini 0)3)(4)(1(

)4(2

xxx

xx que resolveremos, en esta

oportunidad, sin simplificar el factor (x – 4)

– –3 + –1 – 0 + 4 +

O///////////////O/////////////////////////////////O////////////////////

Como x –2 , la respuesta es: x reales con –2 x < –1 ó x 0 ; x 4 .

Page 71: PRUEBA 1 - math.cubava.cumath.cubava.cu/files/2015/05/solucionario-libro-jacinto.pdf · g 9 2x 3 x 2 y ahora podemos ... llegamos a la conclusión parcial y = 8 ó y = 1 para los

71

4.- Vamos a considerar que los tres números enteros consecutivos del ejercicio son:

x ; x + 1 ; x + 2 podemos plantear la ecuación xxxx

xxx3

)2()1(

)2).(1.(

que al efectuar toma la forma xx

xxx3

33

23 23

y ahora eliminamos el denominador

x3 + 3x2 +2x = 9x2 + 9x para llegar a la ecuación reducida y ordenada x3 – 6x2 – 7x = 0

que se descompone en factores x ( x – 7 )( x + 1 ) = 0 .

Los posibles valores de la variable son x = 0 ó x = 7 ó x = – 1.

Podemos obtener respuestas para el ejercicio solamente con las dos primeras, la última no se

desprecia porque sea negativa, sino porque no puede ser solución de la ecuación original,

compruébelo usted. Entonces los tres números son 0 ; 1 ; 2 ó 7 ; 8 ; 9 .

Otra vía: Pruebe usando las variables en la forma x-1 ; x ; x+1 y verá qué fácil pero .....

5.-Es importante destacar que esta pirámide no es regular, a pesar de tener por base un

cuadrado, pues no es recta, ya que el pie de su altura no está en el centro de la base.

El único dato numérico que poseemos es que la suma de las áreas de dos “importantes”

triángulos es 18 cm2 . Podemos probar que esos dos importantes triángulos son iguales, o sea,

SDA = SDC ya que ambos son triángulos rectángulos pues SD es altura, entonces es

perpendicular al plano base, por lo que es perpendicular a toda recta que pase por su pie ;

además el cateto SD es común y los lados AD y DC (catetos) también son iguales por ser

lados del cuadrado base; entonces los triángulos tienen iguales dos lados y el ángulo

comprendido entre ellos. Concluimos, además, que estos triángulos tienen igual área, por lo

que el área de uno cualquiera de ellos es 9,0 cm2.

Podemos escribir la fórmula para el cálculo del área de un triángulo;

42

)..5,0(

2

.9

2

DCDCDCDCSDA SDC y ahora podemos despejar

cmDC 6364.9 y decidir que hP = SD = 3 cm.

Podemos probar que el triángulo ABS es rectángulo en A , pues AD es la proyección de AS

sobre el plano base y ABAD entonces ABSA (teorema de las tres perpendiculares en el

punto A). De igual manera con el triángulo BCS.

Se puede probar también que ABS = BCS. (no vale la pena)

Calculamos la longitud de SCSA aplicando el teorema de Pitágoras

cmSDADSA 53)12.(333.236 222222222

Calculamos la superficie total de la pirámide,

29432,9432,405424,2.1854

2

53.6.21836 cmAAA LBT

y el volumen 3363

3.36

3

.cm

hAV PB .

Page 72: PRUEBA 1 - math.cubava.cumath.cubava.cu/files/2015/05/solucionario-libro-jacinto.pdf · g 9 2x 3 x 2 y ahora podemos ... llegamos a la conclusión parcial y = 8 ó y = 1 para los

72

PRUEBA 37

1.- Intentaremos convertir toda la ecuación, aplicando propiedades de las potencias y de los

radicales, a la misma base. Podemos escribir la ecuación dada en la forma:

2

22

1

33

3

3

x

x

; y en el miembro izquierdo

2

2

2

1

33

3

xx

que conduce a

2

2

2 33

3 xx

; y ya podemos igualar los exponentes 2

2

2

3

xx

eliminar el denominador y obtener x3 = – x – 2 para llegar a la ecuación cúbica

x3 + x + 2 = 0. Con la ayuda de la regla de Ruffini podemos hallar la descomposición en

factores ( x + 1 )( x2 – x + 2) = 0 donde no podemos continuar descomponiendo pues en el

trinomio ocurre que D = b2 – 4ac = (– 1)2 – 4.1.2 = 1 – 8 = –7 < 0 .

Única solución x = – 1 . Puede comprobarse en la ecuación original.

2.- Podemos denotar, en la figura, que el segmento EB = x , y entonces el lado del cuadrado

podrá escribirse en todo momento como CB = x + 4 .

Se cumple que los triángulos DEC y AFD son rectángulos y además podemos probar que

EDC = AFD (alternos entre las paralelas) por lo que los triángulos tienen dos ángulos

respectivamente iguales y entonces DEC AFD.

Planteamos la proporción entre lados homólogos (se oponen a ángulos iguales)

DC

AF

CE

AD (no fue necesario trabajar con las hipotenusas) . Al expresar la proporción con

variables obtenemos la ecuación fraccionaria siguiente 4

9

4

4

x

x y eliminando

denominadores nos queda x2 + 8x + 16 = 36 que ordenada adecuadamente e igualada a cero

llegamos a x2 + 8x – 20 = 0 que se puede descomponer en factores mediante

( x + 10 )( x – 2 ) = 0 de la que tomaremos solamente x = 2 para poder responder en

nuestro ejercicio. Conocemos EB = x = 2 cm.

El lado del cuadrado mide CB = 2 + 4 = 6 cm. La longitud del segmento BF se puede

calcular mediante la diferencia ABAFBF = 9 – 6 = 3 cm.

El área de toda la figura es 2393362

3.236 cmAAA EBFCF

La amplitud del EFB se puede calcular mediante la función tangente en el triángulo

rectángulo EBF.

666,03

2

BF

EBEFBtan donde EFB = 33,70 .

Page 73: PRUEBA 1 - math.cubava.cumath.cubava.cu/files/2015/05/solucionario-libro-jacinto.pdf · g 9 2x 3 x 2 y ahora podemos ... llegamos a la conclusión parcial y = 8 ó y = 1 para los

73

3.- Partiremos del primer miembro y trabajaremos primeramente su denominador,

xx

xxxx

x

xx

x

xx

x

x

x

x

cossen

sensencoscossen2

cos

sencos

sen

cossen2

cos

2cos

sen

2sen 32222

xxx

x

xx

xxx

xx

xxx

cos

1

cossen

sen

cossen

)sen(cossen

cossen

sencossen 2232

.

Y calculamos el recíproco mediante x

x

cos

cos

1

1 . Queda probada la identidad.

4.- Hacemos la conversión del área 0,18hm2 = 1800m2. Tomaremos las variables L y a

para denotar el largo y ancho del huerto respectivamente.

Las ampliaciones se representarán entonces por L + a (sobre el largo) y a + a = 2a (sobre el

ancho). Podemos plantear la ecuación L.a = 2a.(L + a) – 1800.

Del perímetro podemos plantear que L + a = 70, por lo que L = 70 – a.

Entonces en la primera ecuación nos queda, después de sustituir L,

(70 – a).a = 2a.70 – 1800, con la que podemos llegar a la ecuación cuadrática

a2 + 70.a – 1800 = 0, que se descompone en (a + 90).(a – 20) = 0, donde solamente

tomaremos a = 20. Calculamos que L = 50.

El huerto ampliado tendrá las dimensiones ancho 40m y largo 70m.

5.- En la expresión para el volumen total podemos hacer sustituciones,

3

.314

3

.5.14,3.4

3

...4

3

....314

2222 hhhrhr

hrVVV COCIT

y ahora podemos calcular dmh 3314

314.3 .

Calculamos el área lateral del cilindro, 2

. 942,943.5.14,3.2...2 dmhrA CIL

PRUEBA 38

1.- En la ecuación original el miembro izquierdo puede expresarse más simplificado debido a

la identidad fundamental logarítmica 16 = 4cos 2x . 64sen x y ahora podemos convertir toda la

ecuación a la base 4 ; 42 = 4cos 2x. 43sen x . El miembro derecho está listo para efectuar el

producto de potencias de igual base, 42 = 4cos 2x + 3 sen x .

Podemos ahora igualar los exponentes 2 = cos 2x + 3 sen x . Se impone a continuación

hacer la sustitución cos 2x = 1 – 2 sen2x y entonces la ecuación quedará en una misma

función trigonométrica 2 = 1 – 2 sen2x + 3 sen x que la ordenaremos para luego intentar

descomponer en factores la ecuación cuadrática

2 sen2 – 3 sen x + 1 = 0 ; ( 2 sen x – 1 )( sen x – 1 ) = 0 . Se obtienen los valores

senx = 1 ó sen x = ½ .Calculamos los valores para x ; x = 300 ó x = 1500 ó x = 900.

Page 74: PRUEBA 1 - math.cubava.cumath.cubava.cu/files/2015/05/solucionario-libro-jacinto.pdf · g 9 2x 3 x 2 y ahora podemos ... llegamos a la conclusión parcial y = 8 ó y = 1 para los

74

2.- En la fórmula para la longitud de la circunferencia podemos calcular el diámetro,

L = 62,8 = . d = 3,14.d donde podemos despejar cmABd 2014,3

8,62 .

El triángulo ABE es rectángulo en E ( E está inscrito sobre el diámetro, Tales).

El triángulo DOF también es rectángulo pero en O (diámetros perpendiculares).

Podemos probar que EAB = CDF , la fundamentación es un poco extensa pero

convincente ( ambos ángulos están inscritos sobre arcos iguales, debido a que los arcos DE y

GB son iguales y además los arcos DB y CB también son iguales, entonces por suma de

arcos iguales los arcos EB y CG también son iguales).

También ocurre que el segmento AE = r ( es una cuerda que sustenta un ángulo inscrito de

300) y además ODrAE . Con todos estos datos podemos concluir que se cumple que

AEB = ODF por tener respectivamente iguales dos ángulos y el lado comprendido. Por

elementos correspondientes podemos decidir que EAB = OFD ;

que son ángulos correspondientes iguales, entonces podemos probar que DF EB .

Podemos calcular que la longitud del segmento OF = cm3,1773,1.103.10 (porque se

opone al ángulo de 600 en el triángulo rectángulo ODF y el lado que se opone al ángulo de

300 mide 10cm).

Otra vía :

O = 900 (diámetros perpendiculares). Arco AE = 600. El ángulo inscrito ABE = 300. El

arco AD = 900 por ser O = 900 un ángulo central. Luego arco ED = arco AD – arco AE = 300

y como arco DE = arco GB por dato, entonces arco GB = 300. Por ser AFD un ángulo

exterior que abarca los arcos AD y GB se tiene que

000

302

3090

2

arcoBGarcoADAFD .

Otra vía muy rápida: Se puede decidir que con los puntos D;E;B y G se puede formar un

trapecio isósceles.

Por tanto DF EB , por formar ángulos correspondientes iguales.

Como OD es un radio y el perímetro de la circunferencia es 68,2 entonces cmOD 10 .

Luego en el triángulo ODF se cumple que cmOF 3,173.10 por oponerse al ángulo de

600 .

Page 75: PRUEBA 1 - math.cubava.cumath.cubava.cu/files/2015/05/solucionario-libro-jacinto.pdf · g 9 2x 3 x 2 y ahora podemos ... llegamos a la conclusión parcial y = 8 ó y = 1 para los

75

3.- Con la base del logaritmo, x2 + 1 , únicamente hay que establecer que x 0 .

En principio 7 x2 – 3 > 0 que solo ocurre cuando 7

32 x , o sea, 7

21

7

3x

lo que es equivalente con 655,07

21x ó 655,0

7

21x .

¡Ahora, a trabajar! De la desigualdad con logaritmos podemos inferir, como la base es mayor

estrictamente que 1, que 7x2 – 3 < ( x2 +1 )2 que efectuando, reduciendo términos

semejante y ordenando adecuadamente podemos expresar por,

x4 – 5x2 + 4 > 0 y se descompone en 4 factores lineales para el análisis de los intervalos

( x + 2 )( x – 2 )( x + 1 )( x – 1 ) > 0

+ –2 – –1 + 1 – 2 +

//////////////////OO///////////////////////////OO/////////////////////

//////////////////////////////////////////////OO////////////////////////////////////////////////

–7

21

7

21

Podemos ahora responder, realizando la intersección de los intervalos señalados.

Solución :

x reales con x < – 2 ó –1 < x < –7

21 ó

7

21 < x < 1 ó x > 2.

4.- Tomaremos como edad del hombre : x . Ecuación

Vivió en Puerto Rico : 3

x 48

53 x

xxx

Vivió en R. Dominicana : 5

%.20x

x 15x = 5x +3x +15x – 720

Vivió en Venezuela : x – 48 8x = 720

x = 90

Vivió en Venezuela 90 – 48 = 42 años. Llegó a Venezuela cuando tenía 48 años.

5.- Para comenzar el análisis debemos trazar la construcción auxiliar, segmento FP

perpendicular a BG en P. Se forma el triángulo rectángulo FPG con PG = 5 cm (por resta

de segmentos sobre BG ).

Aplicando el teorema de Pitágoras en el triángulo rectángulo FPG podemos decidir o calcular

FG = 13 cm (trío de números pitagóricos, en este caso son 13 ; 12 y 5 ).

La figura dada es un prisma que tiene por base un trapecio rectángulo pero la misma no está

apoyada sobre el trapecio, esto es esencial para poder calcular el volumen.

33 8,118006.12.2512.12.

2

1015..

2. dmcmBCFP

AFBGhAV PBP

.

Page 76: PRUEBA 1 - math.cubava.cumath.cubava.cu/files/2015/05/solucionario-libro-jacinto.pdf · g 9 2x 3 x 2 y ahora podemos ... llegamos a la conclusión parcial y = 8 ó y = 1 para los

76

Calculamos el área total del cuerpo,

AT = AABCD + ABCHG + 2.AABGF + AFGHE + AADEF

AT = 144 + 15.12 + 2.25.6 + 13.12 + 12.10 = 144 + 180 + 300 + 156 + 120 = 900cm2

AT = 9,0 dm2 .

PRUEBA 39

1.- Al sustituir en la diferencia planteada queda la ecuación 09632 3 xx , y

utilizaremos la posibilidad de realizar un cambio de variable z = 2 x + 3 .

Nos queda entonces 3 3zz y debemos elevar ambos miembros a la potencia 6 para

poder eliminar las raíces de una sola vez. 636

3zz por lo que ahora todo es más

sencillo con z3 = ( 3z )2 , ecuación que se puede escribir z3 – 9z2 = 0 .

Extraemos factor común z2.( z – 9 ) = 0 , y aparecen las posibilidades

z = 0 ó z = 9 . Hacemos reversible el cambio de variable igualando,

2 x + 3 = 0 , por lo que 2

3x ; y también 2 x + 3 = 9 , donde x = 3 . Se deben hacer

las dos comprobaciones para verificar que los dos valores de x son soluciones.

2.- Es un bonito ejemplo de aplicación. Consideramos que el punto O es el pie de la antena

con el suelo y podemos plantear que 20

2,58sensen 0 MO

MT

MOMTO , y entonces

despejamos MO = 20.sen 58,20 = 20.0,85 = 17 m .

De ahora en adelante se pueden utilizar dos vías ; calcular TO y luego aplicar el teorema de

Pitágoras en el triángulo rectángulo TOS ó aplicar la ley de los cosenos en el triángulo TMS,

pues podemos calcular la amplitud del TMS .

Usaremos la segunda vía . Tenemos que TMO = 31,80 ( pues es complementario con el

MTO y ahora calculamos que TMS = 148,20 (adyacente con el TMO, son

suplementarios, y entonces suman 1800 ). Calculamos la longitud de,

0222

2,148cos...2 MTMSMTMSTS y despejamos

mTS 6,35126785,0.680400289)85,0.(20.17.22017 22

Los tensores superiores miden 36 m.

Page 77: PRUEBA 1 - math.cubava.cumath.cubava.cu/files/2015/05/solucionario-libro-jacinto.pdf · g 9 2x 3 x 2 y ahora podemos ... llegamos a la conclusión parcial y = 8 ó y = 1 para los

77

3.-Vamos a partir del miembro izquierdo de la igualdad e intentaremos llegar al segundo

miembro realizando transformaciones y sustituciones.

1cos

2

coscos.sen2sen1cos

2

cossen 222

2

2

xxxxx

xxx

2

1cos22sen

2

2cos2coscos.sen2sen 2222 xxxxxxx

2

2cos2sen xx . Queda demostrada la identidad trigonométrica.

4.-Declaramos las variables. Estudiantes en ómnibus : x

Estudiantes en tren : 76 – x

Podemos formar la ecuación 1,20 . x + 0,80 . ( 76 – x ) = 74,80

120 x + 6080 – 80 x = 7480

donde llegamos a 40 x = 1400

x = 35

a) Viajaron en tren 76 – 35 = 41 estudiantes .

b) Este segundo inciso se puede calcular sin considerar los resultados del inciso anterior.

$1,20 – $0,80 = $0,40 (diferencia entre los precios de los pasajes).

Es lo que se ahorra por cada estudiante que no viajara en ómnibus y lo hiciera en tren.

Por los 5 alumnos se ahorra en total 5. $0,40 = $2,00.

Esta cantidad se resta del total $74,80 – $2,00 = $72,80

Toda la transportación hubiese costado $72,80.

5.- En este ejercicio se hace necesario trazar la altura SO de la pirámide, que corta al prisma

en un punto O’ de su base superior. Llamaremos C’ al punto de intersección de la arista

lateral SC de la pirámide con el prisma.

Solo necesitamos calcular la altura del prisma para poder calcular su volumen, pues con la

longitud de la diagonal del cuadrado, podemos calcular el área de su base.

Podemos probar que SOC SO’C’ ya que ambos son triángulos rectángulos y se cumple

que OSC es común para los dos triángulos, entonces son semejantes porque tienen dos

ángulos respectivamente iguales.

La altura h del prisma es el segmento 'OO . Podemos plantear la razón entre los segmentos

proporcionales ''' CO

OC

SO

SO y haremos las siguientes sustituciones con números y la

variable h : 2

3

6

6

h .

Page 78: PRUEBA 1 - math.cubava.cumath.cubava.cu/files/2015/05/solucionario-libro-jacinto.pdf · g 9 2x 3 x 2 y ahora podemos ... llegamos a la conclusión parcial y = 8 ó y = 1 para los

78

Resolvemos esta sencilla ecuación fraccionaria en la variable h eliminando, como de

costumbre, los denominadores y obtenemos que h = 2 dm.

Calculamos el volumen del prisma,

322

162.2

4.

2. dmh

dhAV BPRISMA .

PRUEBA 40

1.- En la segunda ecuación ya podemos tener despejada la variable x = 5y .

En la primera ecuación podemos hacer transformaciones con el objetivo de simplificarla

tanto como sea posible. log2 (x + 3y) + log2 (x – y2) – log2 (x – y) = 3

Podemos ahora llevar a un solo logaritmo el miembro izquierdo de esta ecuación,

3))(3(

log2

2

yx

yxyx y expresarla sin logaritmo 3

2

2))(3(

yx

yxyx.

Sustituimos la variable x = 5y ; 84

)5.(8 2

y

yyy ahora podemos simplificar

considerando que y 0 .

5y – y2 = 4 lo que se puede escribir como y2 – 5y + 4 = 0 que se descompone en

factores (y – 4)(y – 1) = 0 donde y = 4 ó y = 1 . Con estos valores se pueden encontrar

x = 20 ó x = 5 respectivamente. Los pares ordenados que satisfacen el sistema de

ecuaciones son : ( 20 ; 4 ) ó ( 5 ; 1 ) .

2.- Si tenemos que A(–2 ; 0 ) entonces B( 2 ; 0 ) pues son puntos simétricos con respecto al

eje Y . Trabajaremos en el AOC , rectángulo en O, donde podemos plantear que

24,63 0 OC

OA

OCtan y ahora calculamos la longitud del segmento

OC = 2 . tan 63,40 = 2 . 2 = 4 u . Entonces ya tenemos las coordenadas del punto superior

C( 0 ; 4 ) . Calculamos la pendiente 22

4

)2(0

04

AC

AC

ACxx

yym ;

Entonces, 2

1BMm , pues BMAC .

a) Una ecuación de la recta que contiene al segmento BM es : )2.(2

10 xy

que se puede escribir como x + 2y –2 = 0 .

b) Se puede asegurar la longitud del segmento AB = 4 u . Podemos calcular ahora la

longitud de uno de los lados del triángulo ABC y lo haremos aplicando el teorema de

Pitágoras en el triángulo rectángulo AOC ,

uOAOCAC 522024 2222

.

Page 79: PRUEBA 1 - math.cubava.cumath.cubava.cu/files/2015/05/solucionario-libro-jacinto.pdf · g 9 2x 3 x 2 y ahora podemos ... llegamos a la conclusión parcial y = 8 ó y = 1 para los

79

Calculamos el perímetro del triángulo ABC

uACABP 96,1296,8424,2.4452.24.2

3.- Hay que comenzar el ejercicio planteando la condición que nos impone la expresión

subradical 09

13

3

932 2

x

xx que podemos escribir, luego de haber transferido la

fracción al segundo miembro de la desigualdad, por 62

932

3

13

2

x

xx y ahora realizar

la comparación 62932 33

2 xxx que nos conduce a la comparación de los exponentes

2x2 – 3x – 9 – 2x – 6 ,o sea, 2x2 – x – 3 0 , que se puede descomponer en factores para

analizar los intervalos ( 2x – 3 )( x + 1 ) 0 .

+ –1 – 3/2 +

///////////////////////////////////////////////////

Pero nos piden únicamente los valores negativos, por lo que nuestra respuesta es el

intervalo ]1;( .

4.- Estamos frente a un problema en el que solo se necesita pensar un poco y calcular.

Cada caja de tizas trae 12 . 12 = 144 tizas. Cada caja trae 144 : 4 = 36 tizas verdes.

En una gruesa de cajas de tizas de colores hay 144 . 36 = 5184 tizas verdes.

5.- a) Los triángulos SAC y SBC son rectángulos en C ( SC es perpendicular al plano,

entonces es perpendicular a toda recta del plano que pase por su pie).

En los dos triángulos aplicaremos el teorema de Pitágoras para calcular los catetos que están

en el plano base.

cmSCSAAC 9811442251215 2222

cmSCSBCB 1451442891217 2222

Ahora podemos investigar qué tipo de triángulo es el ABC en la base.

CB 2 = 145 ; AC 2 = 81 y AB 2 = 82 = 64 . Podemos sumar estos dos últimos valores

y apreciamos que nos da el resultado del primero , por lo que el triángulo ABC es rectángulo

en A (recíproco del teorema de Pitágoras).

Page 80: PRUEBA 1 - math.cubava.cumath.cubava.cu/files/2015/05/solucionario-libro-jacinto.pdf · g 9 2x 3 x 2 y ahora podemos ... llegamos a la conclusión parcial y = 8 ó y = 1 para los

80

b)Conociendo los tres lados podemos aplicar la fórmula de Herón y rápidamente calcular el

área del mismo.

Pero también podemos demostrar que el triángulo ABC es rectángulo en A (se cumple el

teorema de las tres perpendiculares en A pues ABAC

y AC es la proyección de SA sobre el plano base ).

Calculamos su área

2602

15.8

2

.cm

SAABA ABS

PRUEBA 41

1.-La ecuación dada se puede escribir, intentando convertirla en el miembro izquierdo a un

solo logaritmo, como 2

182log1log1log 22 xxx y al aplicar las

propiedades de los logaritmos nos queda 2

1

82.1

1log

2

2

xx

x donde podemos obtener

2

1

210

82

1

x

x que es equivalente con 10

82

12

x

x. Igualamos las

expresiones subradicales y eliminamos el denominador, x + 1 = 10 ( 82 – x2 ) ; ecuación

que puede convertirse en 10 x2 + x – 819 = 0 en la que podemos calcular su discriminante

D= b2–4ac = (1)2–4.10.(–819) = 1 + 32760 = 32761 > 0 (dos soluciones reales distintas)

[ 181D ] ; 920

180

10.2

1811

.22;1

a

Dbx .

La otra solución posible x = – 9,1 no pertenece al dominio de la ecuación original.

Hacemos la comprobación y concluimos que S = { 9 } .

2.- Calculamos el valor numérico de la pendiente para luego hallar una ecuación de la

recta r1 . 25,1

3

5,10

031

BA

BA

rxx

yym ; r1 : y + 3 = 2 ( x – 0 ) que

escribiremos como 2 x – y = 3 . Como r2 r1 entonces r2 = – 2

1 pues mr1 . mr2 = –1 .

Hallaremos ahora una ecuación de la recta r2 : y – 0 = – 2

1 ( x – 5 ) que podemos

convertir en x + 2y = 5 .

Page 81: PRUEBA 1 - math.cubava.cumath.cubava.cu/files/2015/05/solucionario-libro-jacinto.pdf · g 9 2x 3 x 2 y ahora podemos ... llegamos a la conclusión parcial y = 8 ó y = 1 para los

81

Para calcular la altura del triángulo que se forma es necesario calcular antes la ordenada del

punto de intersección P de ambas rectas, por lo que resolveremos el sistema de ecuaciones

2x – y = 3 ; x + 2y = 5 .(–2) , o sea, –2x – 4y = –10 . Sumando ordenadamente las dos

ecuaciones nos queda – 5y = – 7 , o sea , y = 7/5 = 1,4 ( altura del triángulo). La longitud

de BC puede calcularse sobre el eje X ; 5 – 1,5 = 3,5 u.

Calculamos el área del triángulo,

245,22

9,4

2

4,1.5,3

2

.u

hBCA .

3.- Resolveremos la ecuación considerando la incógnita x ; para ello eliminamos el

denominador multiplicando en ambos miembros por 4ax .

4ax2 – 8x = 4a2 x – 9a , y ordenamos la ecuación atendiendo a la variable x,

4ax2 + (–8 – 4a2 ) x + 9a = 0.

Analizamos el discriminante de esta ecuación cuadrática en variable x ;

D = (–8 – 4a2 )2 – 4.(4a).(9a) = 64 + 64a2 + 16a4 –144a2 = 16a4 –80a2 + 64 .

Para que la ecuación tenga dos soluciones reales distintas tiene que ocurrir que D > 0 , o sea ,

16a4 – 80a2 + 64 > 0 que podemos descomponer en factores completamente mediante

16 ( a + 2 )( a – 2 )( a + 1 )( a – 1 ) > 0 para analizar los signos por los diferentes intervalos,

+ –2 – –1 + 1 – 2 +

//////////////////////////OO/////////////////////////OO//////////////////////////

Entonces el parámetro a tiene que tomar los valores reales comprendidos en los intervalos

( – ; – 2 ) ( – 1 ; 1 ) ( 2 ; + ) con a 0 .

4.- Número mayor : x Podemos plantear las ecuaciones x + y = 63

Número menor : y yy

x 92

La primera ecuación se puede escribir como x = 63 – y

la segunda ecuación como x = 2y + 9 pues y 0 .

Ahora igualamos las ecuaciones 63 – y = 2y + 9 en la que podemos calcular el valor para la

variable : y = 18 para luego hallar x = 63 – 18 = 45.

Los números son 45 y 18 . Probar en el texto del problema.

5.-En el triángulo CDB, rectángulo en D, podemos plantear que CB

CDDBC sen para

poder calcular la altura del cilindro,

h = CD = CB . senDBC = 10 . sen 64,50 = 10 . 0,903 = 9,03 cm.

Aplicamos el teorema de Pitágoras en el triángulo CDB y podemos calcular la longitud del

segmento cmCDCBDB 3,45,185,8110003,910 2222

Page 82: PRUEBA 1 - math.cubava.cumath.cubava.cu/files/2015/05/solucionario-libro-jacinto.pdf · g 9 2x 3 x 2 y ahora podemos ... llegamos a la conclusión parcial y = 8 ó y = 1 para los

82

Trazamos el segmento auxiliar AC (dentro de la figura ) y se forma el triángulo ACB,

rectángulo en C ( C inscrito sobre el diámetro, teorema de Tales ) para, a continuación,

poder calcular el diámetro AB aplicando el teorema de los catetos,

3,4..1022

ABDBABCB donde cmAB 3,233,4

100

3,4

102

.

Calculamos el diámetro del cilindro d = DBAB .2 = 23,3 – 2.4,3 = 14,7cm

Calculamos entonces el volumen del cilindro,

.5,17,15269.4

7,14.14,3.

4

.. 33

22

dmcmhd

hAV B

PRUEBA 42

1.- Eliminamos el denominador y obtenemos 62.352.2 xx donde, elevando al

cuadrado en ambos miembros, nos queda la nueva ecuación,

362.36)2(9)52(4 xxx que después de operar, reducir y aislar la raíz para

volver a elevar al cuadrado se convierte en 2.362 xx ;

4 – 4x + x2 = 1296 ( x – 2 ) . Esta última ecuación se puede escribir como

x2 – 1300x + 2596 = 0 que se puede descomponer en factores ( x – 2 )( x – 1298 )=0; si no

encuentra los números puede utilizar la fórmula para la resolución de la ecuación de segundo

grado , en este caso D = 1679616 > 0 ; para mayor ayuda le brindamos .1296D Al

comprobar las posibles soluciones se verifica que x = 1298 es una raíz extraña , por lo que la

única solución es x = 2 , después de realizar su correspondiente comprobación.

2.- En la figura, debido a los puntos de tangencia, podemos razonar que DEAD y por otra

parte CDEC (los segmentos de tangentes trazadas desde un punto exterior a una

circunferencia hasta los puntos de tangencia son iguales). Trazamos la construcción auxiliar

ADCP , donde P es un punto sobre el segmento AD ; se forma el triángulo rectángulo

DPC y se puede calcular la longitud de PC aplicando el teorema de Pitágoras 22

DPDCPC , pero antes hay que calcular ECDEDC = 10 + 6,4 = 16,4 cm y

también ,

APADDP = 10 – 6,4 = 3,6cm.

ABcmPC 16256132696,34,16 22 pues podemos probar que el

cuadrilátero ABCP es un rectángulo. El perímetro de toda la figura está dado por:

cmAB

ECADP 5892,5712,258,12202

16.14,34,6.210.2

2

..2.2

.

Page 83: PRUEBA 1 - math.cubava.cumath.cubava.cu/files/2015/05/solucionario-libro-jacinto.pdf · g 9 2x 3 x 2 y ahora podemos ... llegamos a la conclusión parcial y = 8 ó y = 1 para los

83

3.- La expresión no admite ningún valor de ángulo axial porque indefinen indistintamente a

las funciones tan y cot . Tampoco puede tomar /4 + k porque anulan los denominadores

o indefinen la cosecante, por tanto los valores inadmisibles son k/2 ; /4 + k con k

entero.

Partimos del primer miembro de la igualdad haciendo ya las primeras sustituciones de las

funciones tan y cot .

x

xxx

x

x

xxx

x

x

xx

x

x

xx

x

cos

sencossen

cos

sen

cossencos

sen

cos

sen1

sen

cos

sen

cos1

cos

sen

)cos(sensen

cos

)cos(sencos

sen

)sen(cossen

cos

)cos(sencos

sen 2222

xxx

x

xxx

x

xxx

x

xxx

x

)cos(sencos.sen

)coscos.sen).(sencos(sen

)cos(sencos.sen

cossen 2233

xxxx

xxxxxx

xxxx

xx

12sen

21

cos.sen2

2

cos.sen

cos.sen

cos.sen

1

.cos.sen

cos.sen1

xxxxx

xx

xxx

xx

12csc.2 x . Queda probada la identidad para los valores admisibles de la variable.

4.- Podemos modelar la producción de piezas por cada día de la siguiente manera:

Primer día : x

Segundo día : xxxx .5

6%.120%.20

Tercer día : xxxxxxxx .25

66.

5

6.

5

11.

5

11.

5

1.

5

11).

5

6%(20).

5

6(

Podemos expresar toda la suma en los tres días por una ecuación que ahora será bien sencilla :

242.25

66.

5

6 xxx y podemos eliminar denominadores para llegar a la ecuación

25 x + 30 x + 66 x = 6050 y entonces calcular x = 50 .

El primer día se produjeron 50 piezas, el segundo, 60 piezas y el tercer día , 132.

5.-En la fórmula para el cálculo del área de un triángulo podemos proceder para el triángulo

SOB, rectángulo en O (la pirámide es regular, entonces SO es la altura, por lo que DBSO ) .

2

.20

SOOBA SOB , o sea, 40. SOOB (*).

Page 84: PRUEBA 1 - math.cubava.cumath.cubava.cu/files/2015/05/solucionario-libro-jacinto.pdf · g 9 2x 3 x 2 y ahora podemos ... llegamos a la conclusión parcial y = 8 ó y = 1 para los

84

Por otra parte OB

SOtanDBStan 02,68 pero tan 68,20 = 2,5 por lo que podemos

escribir SOOB .5,2 . Sustituyendo en la igualdad (*) nos queda una ecuación con una sola

variable, 40).5,2.( OBOB , o sea , 165,2

402

OB y entonces podemos calcular

OB = 4 m. Calculamos h = SO = 10 m y DB = 8 m = d.

El volumen de la pirámide es,

3

22

10766,1066

10.64

3

10.2

8

3

.2

3

.m

hd

hAV B .

PRUEBA 43

1.-Resolvemos la ecuación en función del parámetro k . Eliminamos el denominador y nos

queda x2 + 2k2 = kx + 7 para llegar a la ecuación cuadrática ordenada,

x2 – kx + 2k2 – 7 = 0 . Para que tenga una sola solución doble tiene que cumplirse que

D = 0 , por lo que D = b2– 4ac = (– k)2 – 4.1.( 2k2 – 7 ) = 0 , y de aquí obtenemos la

ecuación cuadrática k2 – 8k2 + 28 = 0 , o sea, – 7k2 = – 28 por lo que podemos decidir que

k2 = 4 y entonces k = 2 ó k = – 2 .

2.-Llamaremos F al punto de intersección de los segmentos AD y EC (suponemos, para más

rigor, que E pertenece al segmento AB ) .

Tenemos que DFC=600 (por alternos entre paralelas con FDC= 600 )

En el triángulo CFD podemos calcular FC con la función tangente mediante,

tan DFC = tan 600 = FCFC

CD 8 donde m

tanFC

3

38

3

8

60

80

El AFC = 1200 (es adyacente con el CDF = 600, son suplementarios, suman 1800),

entonces FAC=200 (por suma de ángulos interiores de un triángulo).

En el triángulo AFC aplicamos la ley de los senos,

00 120sen20sen

ACFC , o sea , m

FCAC 7,11

342,0

4

342,0

2

3.

3

38

20sen

120sen.0

0

Podemos ahora calcular la longitud de AE en el triángulo rectángulo AEC pues podemos

plantear AC

AEAEC sen donde nos queda ,

.52,7643,0.7,1140sen.7,11sen. 0 mACEACAE

Page 85: PRUEBA 1 - math.cubava.cumath.cubava.cu/files/2015/05/solucionario-libro-jacinto.pdf · g 9 2x 3 x 2 y ahora podemos ... llegamos a la conclusión parcial y = 8 ó y = 1 para los

85

Entonces la altura de la torre es 7,52 + 8 = 15,5 m.

3.- Comenzamos calculando 23,03,210

35,66 0 tan .

Al sustituir obtenemos la ecuación sen2x + sen 2x – cos 2x = 2 que transformamos,

sustituyendo la identidad cos 2x = 2 cos2x – 1 en la ecuación:

sen2x + 2sen x.cos x – 2 cos2x + 1 – 2 = 0 , ahora transformamos la ecuación sustituyendo

con la identidad sen2x = 1 – cos2x y llegamos a la ecuación,

1 – cos2x + 2sen x. cos x – 2cos2x – 1 = 0 para poder llegar a la ecuación más sencilla

2 sen x.cos x – 3 cos2x = 0 que se puede factorizar cos x (2sen x – 3cos x ) = 0 de donde

podemos decidir que cos x = 0 ó 2 sen x – 3 cos x = 0 .

x = /2 ó 2 sen x = 3 cos x

2

3

cos

sen

x

x , o sea, tan x = 1,5

x = 56,310

Pero, como en el intervalo 0 < x < el ángulo está expresado en radianes, no podemos

responder en grados sino en radianes, entonces )(982,0180

14,3.31,56

180

.31,560

0

0

0

radianesx

4.- Denotemos cada asignatura por su letra de comienzo. Podemos plantear el siguiente

sistema de ecuaciones

2

63

2

23

2

...333,943

BQBQ

BQM

QBQM

BQM

[ 3 . 3,94 = 283 ]

que podemos modificarlo para resolverlo con facilidad (I): M + Q + B = 283

(II): M – B = 6

(III): 2M – Q – B = 12

Sumando ordenadamente las ecuaciones I y III podemos directamente calcular el valor de

M = 98,33 para luego calcular en II el valor de B = M – 6 = 98,33 – 6 = 92,33 .

Calculamos el promedio que nos piden,

33,952

66,190

2

33,9233,98

2

BM . Como hicimos aproximaciones para obtener las

notas, el promedio que hemos calculado también sale con aproximación; el promedio exacto

es 95,333…

Page 86: PRUEBA 1 - math.cubava.cumath.cubava.cu/files/2015/05/solucionario-libro-jacinto.pdf · g 9 2x 3 x 2 y ahora podemos ... llegamos a la conclusión parcial y = 8 ó y = 1 para los

86

5.- En el interior del cono se puede formar un triángulo rectángulo con la altura (h) , el

radio (r) y la generatriz (g). Aplicamos el teorema de Pitágoras y podemos plantear la

ecuación h2 + r2 =g2 , o sea, h2 + r2 = 102 .

Por otra parte, el área lateral de cualquiera de las caras del ortoedro se puede escribir como

( 2r ) . h = 96 pues la base del cono está inscrita en el cuadrado, entonces podemos

simplificarla y escribir r . h = 48 para despejar r

h48

.

Sustituimos h en la ecuación de Pitágoras y nos queda, 22

2

1048

r

r , o sea ,

22

2

2

1048

rr

. Eliminamos el denominador y podemos llegar a la ecuación bicuadrática

ordenada r4 – 102.r2 + 82.62 = 0 que de manera “pitagórica” se puede descomponer en

factores ( r2 – 82 )( r2 – 62 ) = 0 y nos aporta directamente las posibles soluciones

r = 8 ó r = 6 donde h = 6 ó h = 8 respectivamente.

Trabajando con r = 8 y h = 6 :

El área de la base del ortoedro es ABO= ( 2r )2 = (2.8)2 = 162 = 256 cm2.

El área lateral del cono ALC= .r.g = 3,14 . 8 . 10 = 251,2 cm2.

No podemos considerar estos valores para r y h porque comparamos y obtenemos que

ABO > ALC lo que está en contradicción con el enunciado del ejercicio.

Trabajando con r = 6 y h = 8 :

El área de la base del ortoedro es ABO= ( 2r )2 = (2.6)2 = 122= 144 cm2.

El área lateral del cono ALC= .r.g = 3,14 . 6 .10 = 188,4 cm2.

Hay que trabajar con estos últimos valores pues con ellos se cumple ABO < ALC .

Calculamos el área de la base del cono ABC= . r2 = 3,14 . 62 = 113 cm2.

Calculamos el área total del cuerpo, de abajo hacia arriba:

AT = ABO + ALO + ( ABO – ABC ) + ALC = 144 + 4.96 + ( 144 – 113 ) + 188

AT = 144 + 384 + 31 + 188 = 747 cm2 = 7,5 dm2 .

PRUEBA 44

1.- Para agilizar el trabajo haremos el cambio de variable y = log2 x por lo que la ecuación

se puede escribir como .917.17 22 yyyy

Si aislamos las raíces podemos extraer factor común radical en el primer miembro

yyy 9)1.(17 2 que podemos despejar

y

yy

1

917 2 .

Page 87: PRUEBA 1 - math.cubava.cumath.cubava.cu/files/2015/05/solucionario-libro-jacinto.pdf · g 9 2x 3 x 2 y ahora podemos ... llegamos a la conclusión parcial y = 8 ó y = 1 para los

87

Podemos escribir directamente que 2

2

2

22

2

21

1881

)1(

)9(

1

917

yy

yy

y

y

y

yy

,

y eliminamos el denominador 81 – 18y + y2 = –y4 – 2y3 + 16 y2 + 34y + 17 , que luego de

reducir y ordenar obtenemos la ecuación y4 + 2y3 – 15y2 – 52y + 64 = 0 en la que al aplicar la

regla de Ruffini nos permite la siguiente descomposición en factores,

( y – 1 )( y – 4 )( y2 + 7y + 16 ) = 0 .

El trinomio no tiene descomposición en los reales pues

D=b2– 4ac = 72 – 4.1.16 = 49 – 64 = – 15 < 0 . Las únicas posibilidades para la variable ¨y¨

son y = 1 ó y = 4 por lo que podemos calcular los valores para la variable x en

1 = log2 x donde x = 2 y en 4 = log2 x donde x = 16 .

Comprobamos en la ecuación original y podemos apreciar que ambas son soluciones.

2.- En principio podemos calcular las longitudes de los lados del rectángulo pues podemos

designar variables y resolver un sistema de ecuaciones.

Largo del rectángulo : L = DC . L + a = 44 donde a = 44 – L

Ancho del rectángulo : a = BC . L . a = 480

Entonces podemos hacer la sustitución L.( 44 – L ) = 480 y llegar a la ecuación cuadrática

ordenada L2 – 44L + 480 = 0, que se puede descomponer en factores mediante

( L – 24 )( L – 20 ) = 0 donde L = 20 ó L = 24 para obtener los valores para la variable a,

a = 24 ó a = 20 respectivamente; pero en nuestro caso L > a , por lo que tomaremos

únicamente L = 24 cm y a = 20 cm .

¡Ahora, a trabajar!

Realizaremos una construcción auxiliar. Prolongamos el segmento MF fuera del rectángulo

y de igual manera prolongamos el ancho AD hasta cortar en P a la prolongación de MF .

Podemos probar que DHC DPM pues los dos son triángulos rectángulos y además

DPM = HDC (ambos son ángulos agudos y tienen sus lados respectivamente

perpendiculares). Los dos triángulos tienen dos ángulos respectivamente iguales .

Podemos plantear PM

DC

DM

HC

PD

DH lados proporcionales que se oponen a los ángulos

iguales en los triángulos semejantes. Sustituimos donde sea posible en la expresión anterior

PMPD

24

13

1026 y calculamos cmPD 8,33

10

26.13 ; cmPM 2,31

10

24.13

Mediante una demostración similar a la anterior podemos probar que también se cumple

APF DHC .

a) Calculamos ADPDAP = 33,8 – 20 = 13,8 cm y como DC

AP

HC

AF por lados

proporcionales; sustituimos 24

8,13

10

AF donde cmAF 75,5

24

8,13.10

Page 88: PRUEBA 1 - math.cubava.cumath.cubava.cu/files/2015/05/solucionario-libro-jacinto.pdf · g 9 2x 3 x 2 y ahora podemos ... llegamos a la conclusión parcial y = 8 ó y = 1 para los

88

b) Calculamos las áreas de los triángulos APF y DPM .

27,392

8,13.75,5

2

.cm

APAFA APF ; 28,202

2

13.2,31

2

.cm

DMPMA DPM

Calculamos el área del cuadrilátero DMFA .

ADMFA = ADPM – AAPF = 202,8 – 39,7 = 163,1 cm2

AFMHB = AABCD – ( ADHC – ADMFA ) = 480 –

1,163

2

24.10

AFMHB = 480 – ( 120 + 163,1 ) = 480 – 283,1 = 196,9 = 197 cm2.

Otra vía :

El inicio es igual. Construcción auxiliar. Por el punto F trazamos DCFP donde P pertenece

al segmento DC . Se puede probar que DHC DME y DHC FPE por ser triángulos

rectángulos que tienen un ángulo agudo común en el primer caso e igual en el segundo.

(HDC = EFP , ángulos agudos que tienen sus lados respectivamente perpendiculares)

Se pueden plantear las proporciones MEDE

1026

13

24 y

PEFE

10

20

2426 , las cuales nos

permiten calcular 08,14DE ; 33,8PE y 41,5ME .

a) Se tiene que cmPEDEDPAF 75,533,808,14 .

b) Se observa que el área del cuadrilátero FMHB es igual al área de ABCD menos la suma

del área del trapecio ADEF y el área del cuadrilátero MECH; y que el área de este último

cuadrilátero es la diferencia de las áreas del DCH y del DME. Entonces

219787,19617,351203,198480

2

13.41,5

2

24.1020.

2

08,1475,5480

)]([

cm

AAAAA DMEDCHADEFABCDFMHB

Page 89: PRUEBA 1 - math.cubava.cumath.cubava.cu/files/2015/05/solucionario-libro-jacinto.pdf · g 9 2x 3 x 2 y ahora podemos ... llegamos a la conclusión parcial y = 8 ó y = 1 para los

89

3.- La desigualdad 332

2

x

x es equivalente con – 3

32

23

x

x por lo que

tenemos que resolver por separado dos desigualdades para después hallar la intersección de

sus respectivas respuestas.

– 32

23

x

x 3

32

2

x

x

032

)1(7

32

770

32

9620

332

20

x

x

x

x

x

xx

x

x

032

115

)1.(032

115

032

962

0332

2

x

x

x

x

x

xx

x

x

El análisis de los signos por cada intervalo en las dos desigualdades es:

+ 1 – 3/2 +

///////////////////////O///////////////////////////////////////////

////////////////////////////////////////////////////O///////////////

+ 3/2 – 11/5 +

La intersección entre los dos gráficos nos da las soluciones del ejercicio,

x reales con x 1 ó x 2,2 .

4.- Recordemos que s = t . v

Velocidad del ciclista A : v Velocidad del ciclista B : w

Analizaremos los espacios recorridos respectivamente.

Recorrió en las 3 primeras horas : 3.v Recorrió en las 3 primeras horas : 3.w

Recorrió en las 5 primeras horas : 5.v Recorrió en las 5 primeras horas : 5.w

Como las carreteras son perpendiculares, podemos aplicar el teorema de Pitágoras con los

espacios que les faltan por recorrer en cada caso, tomando como hipotenusas las distancias que

separan a los dos ciclistas entre sí.

(I) : (80 – 3w)2+(60 – 3v)2=702 (II) : (80 – 5w)2+(60 – 5v)2=502

que se pueden transformar respectivamente en,

9w2+9v2–480w–360v= –5100 .(–25)

25w2+25v2–800w–600v= –7500 .( 9 ) preparamos las ecuaciones

–225w2–225v2+12000w+9000v=127500

225w2+225v2–7200w–5400v = –67500 sumamos ordenadamente

Page 90: PRUEBA 1 - math.cubava.cumath.cubava.cu/files/2015/05/solucionario-libro-jacinto.pdf · g 9 2x 3 x 2 y ahora podemos ... llegamos a la conclusión parcial y = 8 ó y = 1 para los

90

y llegamos a la ecuación lineal:

4800w + 3600v = 60000

4w + 3v = 50 donde despejamos 4

350 vw

y sustituimos en la

ecuación (I), 4900936036004

350.380 2

2

vv

v para calcular v .

entonces 0130093604

9170 2

2

vv

v de donde podemos obtener la ecuación

01300936016

81306028900 2

2

vvvv

y cuando eliminamos el denominador y reducimos los términos semejantes llegamos a la

ecuación cuadrática ordenada 225v2 – 2700v + 8100 = 0 , que aún más simplificada es

v2 – 12 v + 36 = 0 por lo que ( v – 6 )2 = 0, donde v = 6 .

84

32

4

1850

4

6.350

w

Velocidad del ciclista A : 6 km/h

Velocidad del ciclista B : 8 km/h

5.- Si el cubo tiene 512 cm3 de volumen entonces su arista lateral mide 8 cm ya que en la

fórmula de su volumen se verifica 83 = 512 .

En el cuadrado base se forman, a los lados del triángulo equilátero inscrito, dos triángulos

rectángulos iguales que tienen ángulos de 150 y 750 , con un cateto de 8 cm.

Podemos calcular la longitud del lado del triángulo equilátero mediante la función coseno

CJCJ

CBJCB

815coscos 0 por lo que despejamos cuidadosamente

cmCJ 28,8966,0

8

15cos

80

.

El prisma CIJKLM tiene sus tres caras laterales iguales, por lo que el área de cualquiera de

ellas es CLCLCJcmAJCLK .28,8.8,823

4,248 2 y podemos calcular la altura del

prisma de base triangular cmCL 1028,8

8,82 .

La altura del prisma HRSKLM está dada por HCCLHL = 10 – 8 = 2 cm.

Calculamos el volumen del prisma,

.34,592

73,1.6,682.

4

73,1.)28,8(.

4

3.. 3

22

cmhL

hAV B

Page 91: PRUEBA 1 - math.cubava.cumath.cubava.cu/files/2015/05/solucionario-libro-jacinto.pdf · g 9 2x 3 x 2 y ahora podemos ... llegamos a la conclusión parcial y = 8 ó y = 1 para los

91

PRUEBA 45

1.- Sustituimos en la expresión, eliminamos los denominadores multiplicando todos los

términos por el denominador )2(2 xx y nos queda ahora la siguiente ecuación,

)2(.48 xxx . Realizando las operaciones indicadas, transponiendo y

ordenando, llegamos a la ecuación 08.6 xx que se puede descomponer en

factores mediante 0)2)(4( xx donde obtenemos 4x ó 2x .

Las posibles soluciones son x = 16 ó x = 4 . Hay que despreciar la última por no

pertenecer al dominio de las funciones. Solución única x = 16. Recuerde comprobar.

2.- a) Como Q y P son puntos medio de los lados iguales en el triángulo isósceles ABC

podemos afirmar que QACP , que son a la vez catetos de los triángulos rectángulos AQE

y CPD . Se cumple que PCD = ABC (alternos entre paralelas) pero CAB = ABC

(ángulos base del triángulo isósceles, son iguales) y entonces se cumple que CAB = PCD

(por la propiedad transitiva de la igualdad). Podemos concluir afirmando que CPD = AQE

(por tener dos ángulos respectivamente iguales y el lado comprendido entre ellos).

b) Se cumple que PDQE (elementos correspondientes en triángulos iguales).

Aplicando el teorema de Pitágoras en el triángulo rectángulo CPD podemos calcular que

CP = 12 cm por lo que el perímetro del triángulo ABC se puede calcular fácilmente :

P = 4.12 + 14,4 = 62,4 cm.

c) En el triángulo CQN (rectángulo en Q ) podemos calcular que QNC = 520 (ángulos

complementarios ) y como BNE = QNC (opuestos por el vértice), entonces BNE = 520.

3.- Como la fracción es propia el numerador tiene que ser estrictamente menor que el

denominador ; entonces la fracción se puede representar por 2x

x . Según lo planteado en el

problema podemos llegar a escribir la ecuación fraccionaria,

6

1

23)2(

2

x

x

x

x. Procedemos a eliminar denominadores considerando el mínimo

denominador común 6 ( x + 5 )( x + 2 ) y llegamos a la ecuación,

6 ( x + 2 )2 = 6x ( x + 5 ) + ( x + 2 )( x + 5 ) que reducida y ordenada queda como

x2 + 13x – 14 = 0 , que se descompone en ( x + 14 )( x – 1 ) = 0 y entonces hallamos las

posibles soluciones x = – 14 ó x = 1 . Despreciamos la primera porque nos conduce a una

fracción impropia. La fracción pedida es 3

1 .

Page 92: PRUEBA 1 - math.cubava.cumath.cubava.cu/files/2015/05/solucionario-libro-jacinto.pdf · g 9 2x 3 x 2 y ahora podemos ... llegamos a la conclusión parcial y = 8 ó y = 1 para los

92

4.- Conocemos que el conjunto imagen de la función y = sen x está en el rango

–1 y 1 , entonces podemos escribir que – 116

61

2

p

p para que puedan existir

soluciones en dicha ecuación. Resolveremos por separado cada desigualdad para luego hallar

los elementos comunes entre ambas soluciones.

Comenzamos por 116

62

p

p donde comparando con cero y operando en el miembro

izquierdo llegamos a la desigualdad 016

1662

2

p

pp que admite la descomposición en

factores 0)4)(4(

)2)(8(

pp

pp (obsérvese que uno de los factores del denominador cambió de

signo, por lo que se invirtió el sentido de la desigualdad).

Del mismo modo en la segunda desigualdad 116

62

p

p si comparamos con cero,

después operamos y hacemos un cambio de signo en la misma fracción (no afecta el sentido

de la desigualdad), llegamos a la expresión más cómoda para trabajar, ,0)4)(4(

1662

pp

pp

que se descompone completamente .0)4)(4(

)2)(8(

pp

pp

El respectivo análisis de los signos por intervalo nos da las soluciones de cada una.

+ –8 – –4 + 2 – 4 +

////////////////////O/////////////////////////O/////////////////////////////////

////////////////////////////////////O////////////////////////////O/////////////////

+ –4 – –2 + 4 – 8 +

Respuesta: p real en los intervalos (– ;- 8] [–2 ; 2] [8 ; +).

5.- La pirámide es regular y las caras laterales son triángulos isósceles iguales. Calculamos la

altura de uno de estos triángulos (caras) laterales en los que la misma divide a la base (arista

de la base de la pirámide) en dos partes iguales. La mitad entonces mide 3,0 cm y aplicando

el teorema de Pitágoras calculamos la altura del triángulo (cara) lateral : cmha 4 .

Calculamos el área de uno de estos triángulos mediante la fórmula siguiente,

2

. a

C

haA y obtenemos 212cmAC . Dividimos el área lateral por el área de una de

estas caras llegamos a que n = 4 , que es el número de lados en la base.

La base es un cuadrado. Tomamos ahora un triángulo de apoyo de una cara lateral, en el que

la hipotenusa es la altura de esa cara y el cateto en la base es la mitad de la arista de la base

(la pirámide es recta).

Page 93: PRUEBA 1 - math.cubava.cumath.cubava.cu/files/2015/05/solucionario-libro-jacinto.pdf · g 9 2x 3 x 2 y ahora podemos ... llegamos a la conclusión parcial y = 8 ó y = 1 para los

93

Aplicamos de nuevo el teorema de Pitágoras para calcular la altura de la pirámide (cateto)

y entonces cma

hh aP 7916342

22

2

2

Calculamos el volumen

.7,3168,313

64,2.36

3

7.6

3

. 32

cmhA

V PB

PRUEBA 46

1.- Considerando que sen 4x = 2.sen 2x . cos 2x podemos escribir la ecuación como

2sen 2x .cos 2x – 3cos2x + sen2x + sen2x + cos2x = 0 donde podemos llegar a

2sen 2x .cos 2x – 2cos2x + 2sen2x = 0 y extraer –2 como factor común

2sen 2x .cos 2x – 2( cos2x – sen2x ) = 0 y sustituir

2sen 2x .cos 2x – cos 2x = 0 donde factorizando queda 2cos 2x ( sen 2x – 1 ) = 0

y llegamos a que cos 2x =0 ó sen 2x = 1 y ahora resultará fácil obtener las soluciones

2x = /2 + k. ó 2x = /2 + 2k.

x = /4 + k./2 ó x = /4 + k.

donde prevalece que x = /4 + k./2 ; k entero.

2.- Utilizaremos la vía del radio de la circunferencia; todas las distancias del centro

O ( x0 ; y0 ) a cada punto son iguales. 2

0

2

0

2

0

2

0 )4()5()()3( yxyx

(distancias entre los puntos del plano) 2

0

2

0

2

0

2

0 )5()2()()3( yxyx .

Eliminando las raíces, resolviendo los binomios y reduciendo términos semejantes llegamos a

un sistema de ecuaciones muy sencillo 4x0 – 8y0 = 32 ; –2x0 – 10y0 = 20

que al resolverlo obtenemos x0 = 20/7 ; y0 = –18/7 (coordenadas del centro).

Se pueden usar otras vías: intersectando las ecuaciones de dos mediatrices, por ejemplo, de los

segmentos AB y BC ; camino interesante, pero con un poco más de labor.

3.- La desigualdad puede quedar escrita 152

22.2

22

x

x y realizando el cambio de variable

W = 2x nos queda que 154

4 W

W . Si comparamos con cero y realizamos las operaciones

en el miembro izquierdo llegamos a la expresión 04154 2

W

WW

que conduce a 0)14)(4(

W

WW y que al resolverla obtenemos los intervalos

– –1/4 + 0 – 4 +

O///////////////////OO/////////////////////////

–1/4 < W < 0 ó W > 4 ; pero la expresión exponencial no acepta el primero

pues 2x > 0 . Entonces 2x > 4 donde necesariamente x reales con x > 2 son las soluciones.

Page 94: PRUEBA 1 - math.cubava.cumath.cubava.cu/files/2015/05/solucionario-libro-jacinto.pdf · g 9 2x 3 x 2 y ahora podemos ... llegamos a la conclusión parcial y = 8 ó y = 1 para los

94

4.- Consideramos las variables Álbum : A ; Beatriz : B y Carlos : C

La primera ecuación nos quedará (I): A + B + C = 575

La segunda ecuación es (II): B – 15 = C + 15 (los 15 sellos de Beatriz pasan a Carlos)

Una tercera ecuación puede ser (III): B – 15 = A ó también C + 15 = A (nosotros

usaremos, de estas dos últimas, la primera) B2 – 30B + 225 = A que podemos ir a sustituir en

(I). Si en la ecuación (II) despejamos C y también sustituimos en (I) obtenemos:

B2 – 30B + 225 + B + B – 30 = 575 donde nos queda B2 – 28B – 380 = 0 que admite la

descomposición (B – 38 )( B + 10 ) = 0 donde B = 38 ó B = –10 (imposible esta última).

Sustituimos en la ecuación (II) y hallamos C = 8.

Beatriz tiene 38 sellos y Carlos tiene 8.

5.- Partiremos de la igualdad entre las áreas laterales de cada cuerpo.

.r.g = 2..r.h . Si simplificamos la expresión llegamos a que g = 2h pero en el cono circular

recto se puede formar un triángulo rectángulo característico en su interior y mediante el

teorema de Pitágoras podemos llegar a g2 = r2 + h2. Como r = 5,0 dm sustituimos todo en esta

última ecuación y nos queda que (2h)2 = 52 + h2 que al ser resuelta nos conduce a que

3

3.5h . El volumen del cilindro será 34,226

3

73,1.125.14,3

3

3.5.5.14,3.. 22 hrV .

El volumen es de 226 dm3.

PRUEBA 47

1.- La ecuación, mejor preparada, nos puede quedar 15 33.3 xxx

y que al utilizar de

nuevo las propiedades de las potencias obtenemos 15 33 xxx

donde podemos igualar

los exponentes y resultará la ecuación con radicales 15 xxx . Al elevar al

cuadrado ambos miembros y aislando el radical que aún nos queda obtenemos

45.2 22 xxx . Elevando de nuevo al cuadrado, reduciendo términos y ordenando

llegamos a la ecuación x4 – 12x2 – 20x + 16 = 0 que admite la solución entera x = 4 que es

la única, que nos afirman, que de este tipo tiene.

2.- a) Mediante el perímetro de la circunferencia podemos obtener el radio de la misma en la

fórmula P = 2..r , donde 50,24 = 2.3,14.r y 828,6

24,50r ; ahora aparecerán todas las

longitudes de los lados del DOB : OB = 8 ; DB = 17 y OD = 15. Vemos ahora que

172 = 82 + 152 es una expresión verdadera y entonces, por el recíproco del teorema de

Pitágoras, el DOB es rectángulo en el punto O.

Page 95: PRUEBA 1 - math.cubava.cumath.cubava.cu/files/2015/05/solucionario-libro-jacinto.pdf · g 9 2x 3 x 2 y ahora podemos ... llegamos a la conclusión parcial y = 8 ó y = 1 para los

95

b) Resulta evidente que debemos calcular el área del ACB que es rectángulo en C por

estar inscrito sobre el diámetro AB (teorema de Tales). El ABC es común a los dos

triángulos rectángulos por lo que podemos afirmar que ACB ~ DOB (tienen dos ángulos

respectivamente iguales). Podemos establecer que CB

OB

AC

OD

AB

DB (lados proporcionales);

sustituyendo CBAC

815

16

17 . Despejamos entonces y nos queda que

17

16.15AC y

17

16.8CB donde AC = 14,1cm y CB = 7,53cm.

El área pedida es

222

4,471,535,1002

53,7.1,14

2

8.14,3

2

.

2

.cm

CBACrAS

.

El área sombreada es de 47cm2.

3.- a) En la ecuación general de la recta, B

Am pero en este caso queda

2

2

9 k

km

donde

92

2

k

km .

b) Debemos entonces proponer que la pendiente sea positiva o sea

0)3)(3(

2

kk

k donde debemos proceder con cautela en el numerador (cero doble).

+ –3 – 0 – 3 +

//////////////////////OOO/////////////////////////

Obtenemos entonces que k < –3 ó k > 3.

4.- Designemos las variables Monedas de 1 peso : x ; Monedas de 20 centavos : y

Como cada persona toma una moneda x + y = 44

Como cada moneda aporta su valor 1.x + 0,20y = 24

Si restamos ordenadamente las dos ecuaciones nos queda 0,80y = 20 donde y = 25

y entonces el valor de x será x = 44 – 25 = 19 (monedas de peso) mujeres.

En el grupo hay 19 mujeres y 25 hombres.

5.- Es importante realizar un buen dibujo de la situación y percatarnos de que la única

posibilidad de modelación se basa en que el triángulo lateral de menor área es rectángulo

isósceles de catetos 9,0cm (uno de ellos es la altura de la pirámide).

La cara lateral de menor área es el SBC (se puede discutir, vale la pena).

El ACS es rectángulo en C (teorema de las tres perpendiculares en C).

Page 96: PRUEBA 1 - math.cubava.cumath.cubava.cu/files/2015/05/solucionario-libro-jacinto.pdf · g 9 2x 3 x 2 y ahora podemos ... llegamos a la conclusión parcial y = 8 ó y = 1 para los

96

El SBC es rectángulo e isósceles por lo que cmBSACCB 0,9 y

ABSC = 29 cm (hipotenusa de un triángulo rectángulo isósceles).

Se puede, incluso, probar la igualdad de los triángulos rectángulos ACS y ABC (hipotenusa

común y un cateto que mide 9,0cm).

22 6,115582,3.5,40)82,21.(2

81

2

29.9.2

2

9.9dmcmAL

33 12,05,1215,40.33

9.5,40dmcmV .

PRUEBA 48

1.- La segunda ecuación se puede poner como log2( x.y ) = 2 donde x.y = 4. Despejando la

variable x en esta última y sustituyendo en la primera queda 64

..4

y

yyy

, y

racionalizando el segundo término resulta la ecuación 6.2.4

yy

y que procedemos a

elevar al cuadrado en ambos miembros 364.2..4

.216

2 yy

y

y

y

y. Simplificando y

efectuando llegamos a 3641616

yy

que al eliminar el denominador y ordenarla queda

una ecuación mucho más sencilla y2 – 5y + 4 = 0 con soluciones y = 1 ó y = 4 . Se calculan

los correspondientes valores para la x ; x = 4 ó x = 1. Los posibles pares son ( 4 ; 1) ó

( 1 ; 4 ) ; pero el único que cumple la condición inicial planteada es el primero.

2.- a) Basta probar que los tres puntos están alineados, que el área del triángulo es cero o que

no se cumple la desigualdad triangular. Trabajaremos con las pendientes de los segmentos

AB y BC (B punto común). 3

1

6

2

51

24

ABm ;

3

1

14

45

BCm . Los tres puntos están

alineados y no se puede formar un triángulo.

b) Calculamos las coordenadas del punto medio de AB :

2

24;

2

51M donde obtenemos

M( –2 ; 3 ). La mediatriz es perpendicular y entonces su pendiente será

m’ = –3. Formamos la ecuación con el punto M y la pendiente m’ , donde resulta

3x + y = –3 que para x = 0 tenemos que y = –3 ; para y = 0 tenemos x = –1.

Los puntos son ( 0 ; –3 ) y ( –1; 0 ).

Page 97: PRUEBA 1 - math.cubava.cumath.cubava.cu/files/2015/05/solucionario-libro-jacinto.pdf · g 9 2x 3 x 2 y ahora podemos ... llegamos a la conclusión parcial y = 8 ó y = 1 para los

97

3.- Partiremos del primer miembro para llegar al segundo. Sustituimos y efectuamos,

cos 2(2x) + 4.cos2x + 2.(2.sen x . cos x)2 + 3 =

= 2.cos22x – 1 + 4.(2.cos2x – 1) + 2.(4.sen2x . cos2x) + 3 =

= 2.(2.cos2x – 1)2 – 1 + 4.(2.cos2x – 1) + 8.sen2x.cos2x + 3 =

= 2.(4.cos4x – 4.cos2x +1) – 1 + 8.cos2x – 4 + 8.cos2x.(1 – cos2x) + 3 =

= 8.cos4x – 8.cos2x + 2 – 1 + 8.cos2x – 4 + 8.cos2x – 8.cos4x + 3 =

= 8.cos2x queda demostrada la identidad.

4.- Podemos advertir que la suma de los tiempos consumidos en cada etapa nos da las

5 h (horas) exactas por lo que t1 + t2 = 5 pero v

st donde podemos llegar a la ecuación

52

24

%.80

24

vv que puede simplificarse 5

2

2430

vv y si eliminamos los

denominadores llegamos a la ecuación 30.( v + 2 ) + 24v = 5v.( v + 2 ) que se puede

transformar en 5v2 – 44v – 60 = 0 que admite la descomposición en factores lineales

( v – 10 ).( 5v + 6 ) = 0 con única solución v = 10 pues despreciamos la posible solución

negativa. La velocidad prevista era de 10 km/h.

5.- Trabajaremos inicialmente con la condición planteada, considerando que la arista de la

base es a y la altura de los triángulos (caras) laterales sea ah .

Comenzamos con 222

..4 a

ha a

entonces 22.2 aha a lo que nos conduce obviamente a

que aha . En un triángulo rectángulo de apoyo de cualquier cara lateral tenemos que la

hipotenusa está dada por aha y el cateto en la base necesariamente es 2

a (la base es un

cuadrado) y la altura es, por dato, 3 . Podemos descubrir que el ángulo entre el cateto en la

base y la hipotenusa es de 600. En ese triángulo podemos aplicar el teorema de Pitágoras o

aplicar trigonometría y llegar a que a = 2,0dm pues

la altura h = 1,73 = 3 .Calculamos el volumen 322

3,23

73,1.4

3

73,1.2

3

.dm

haV .

El ángulo de inclinación de la cara lateral es de 600.

PRUEBA 49

1.- Podemos eliminar los denominadores directamente considerando el mínimo común

denominador 22 2.2 xxxx .

Entonces )22.(2.22.2 222 xxxxxx , que nos conduce a la ecuación sin

radicales 2x3 – 6x = 0 que tiene posible soluciones x = 0 ó 3x .

Comprobamos que la única que satisface es x = 0 pues las otras no pertenecen al dominio de

la ecuación original.

Page 98: PRUEBA 1 - math.cubava.cumath.cubava.cu/files/2015/05/solucionario-libro-jacinto.pdf · g 9 2x 3 x 2 y ahora podemos ... llegamos a la conclusión parcial y = 8 ó y = 1 para los

98

2.- a) Trazamos el radio OC e intentaremos probar la semejanza de los triángulos

rectángulos COB y AO’T’ (rectángulos en los puntos de tangencia C y T’ con los radios

respectivos). Tenemos que COB = O’AT’ (ángulos agudos con sus lados respectivamente

perpendiculares). Se cumple entonces que COB AO’T’ (tienen dos ángulos

respectivamente iguales).

La igualdad ''' AT

CO

TO

CB se obtiene por los lados homólogos y podemos calcular

2,28,6

5.3''.'

CB

TOCOAT . El área del triángulo AO’T’ será 23,3

2

2,2.3cmA .

b) Podemos hallar TTATAT '' = 2,2 + 7,8 = 10cm y se cumple además

que ACAT pues son segmentos tangentes a la circunferencia mayor desde el punto A,

entonces AC = 10cm.

La distancia de A hasta O se puede calcular aplicando el teorema de Pitágoras en el triángulo

AOT rectángulo en T (punto de tangencia y radio).

cmOTATAO 2,1124,2.55.5125510 2222

.

3.- Se debe probar que h(x) = sen 2x cuando W = 4.

El denominador, después de sumarlo, nos queda simplemente xsen

2 por lo que ahora

probaremos el resto xxxxx

x

x2sencos.sen.2

2

sen.cos.4

sen

2

cos.4 . Queda demostrada.

4.- Denotemos la cantidad fija por F y el porciento por P.

Para los $80,00 y los $100,00 nos quedan las respectivas ecuaciones

80 + F + P%.80 = 83,90 y 100 + F + P%.100 = 104,50 que más simplificadas

pueden ser 9,3100

.80

PF y 5,4

100

.100

PF .

El sistema queda 10.F + 8.P = 39 que al resolverlo nos brinda

10.F + 10.P = 45

los valores P = 3 y F = 1,5.

La cantidad fija es de $1,50 y el tanto por ciento por la cantidad que se envía es el 3%.

(Este problema se puede resolver también por una simple vía aritmética).

Page 99: PRUEBA 1 - math.cubava.cumath.cubava.cu/files/2015/05/solucionario-libro-jacinto.pdf · g 9 2x 3 x 2 y ahora podemos ... llegamos a la conclusión parcial y = 8 ó y = 1 para los

99

5.- a) Se cumple que 2

1

g

h y también tenemos la igualdad gr

hr..

3

.. 2

que al

simplificarla nos conduce a r.h = 3.g donde rg

h 3 entonces

2

13

r y calculamos r = 6,0u.

En el triángulo generatriz del cono podemos asegurar que el ángulo entre la generatriz y el

radio es de 300 (por la proporción que nos dicen que están h y g) . El ángulo entre la

generatriz y la altura es de 600 (ángulos complementarios).

b) Podemos calcular g pues tenemos que g

630cos 0 por lo que

92,673,1.43.4

2

3

6

30cos

60

g

22 4,24392,12.84,18)692,6.(6.14,3).(.... urgrrgrAAA BLT

el área total del cono es 243u2.

PRUEBA 50

1.- La ecuación puede quedar

4

3sen.2log

2

32cossen1log 3

22 xxx donde

igualando los argumentos y efectuando el producto queda

4

3sen.2sen.

2

3sen.2cos

2

32cos 3 xxxxx que la llevaremos toda a sen x ;

4

3sen.2sen.

2

3sen.sen21

2

3sen.21 322 xxxxx y nos puede quedar

4

3sen.

2

5sen.2

2

5 2 xx que nos conduce a la ecuación cuadrática

8.sen2x + 10.sen x – 7 = 0 y se descompone así : (4.sen x + 7).(2sen x – 1) = 0 donde

4

7sen x ó

2

1sen x . La primera expresión no tiene sentido pero la segunda nos entrega

las soluciones 6

ó

6

5 de los cuadrantes I y II respectivamente.

2.- Partiremos probando la semejanza entre los triángulos rectángulos FCB y EFB.

EBF = BFC (alternos entre las paralelas en el rectángulo).

Se cumple entonces que FCB ~ EFB (tienen dos ángulos respectivamente iguales).

Aplicando el teorema de Pitágoras en el FCB podemos calcular FC .

cmBCFBFC 0,864610 2222

.

Page 100: PRUEBA 1 - math.cubava.cumath.cubava.cu/files/2015/05/solucionario-libro-jacinto.pdf · g 9 2x 3 x 2 y ahora podemos ... llegamos a la conclusión parcial y = 8 ó y = 1 para los

100

Con los segmentos proporcionales planteamos EB

FB

FB

FC donde podemos

calcular cmFC

FBEB 5,12

8

10 22

y restando segmentos obtenemos AB = 10cm quedando

probado que el ABF es isósceles de base AF .

El cálculo del área del ABF se puede hacer mediante ABFA sen.10.2

1 2 .

23010

6.50sen.50 cmBFCA . El área es de 30cm2.

3.- Consideramos las variables por las letras de comienzo de cada nombre, entonces

A + B + D = 200. La segunda ecuación se obtiene al entregar Berta los $20 a Damián,

B – 20 = D + 20 y la última puede ser 2.(B – 20) = A ó 2.(D + 20) = A.

Nosotros utilizaremos 2D + 40 = A.

Sustituyendo A y B en la primera ecuación nos quedará 2D + 40 + D + 40 + D = 200

donde D = 30 y luego podemos hallar A = 100 y B = 70.

Arnaldo aportará $100 , Berta $70 y Damián $30.

4.- Hay que comenzar planteando que 02

42

2

42 22

x

xx

x

xx necesariamente.

Sumando las fracciones con el uso del común denominador (x + 2).(x – 2) nos queda

02.2

42.242.2 22

xx

xxxxxx donde operando y reduciendo los términos

semejantes llegamos a que

02.2

2 3

xx

x. Resolviendo la desigualdad obtenemos

los intervalos :

– –2 + 0 – 2 +

O///////////////////O///////////////////////

que nos da (–2 ; 0 ] (2 ; +) como dominio de h.

Page 101: PRUEBA 1 - math.cubava.cumath.cubava.cu/files/2015/05/solucionario-libro-jacinto.pdf · g 9 2x 3 x 2 y ahora podemos ... llegamos a la conclusión parcial y = 8 ó y = 1 para los

101

5.- Percatarnos que 628 = 200. es bastante fácil; por lo que podemos escribir

.r2 + .r.g = 200. que se puede llevar a una más sencilla simplificando

r2 + r.g = 200. En el triángulo generatriz, rectángulo, podemos aplicar el teorema de

Pitágoras y nos queda 2256 rg que se puede sustituir en la ecuación anterior

y obtener 200256. 22 rrr . Aislamos el radical y al elevar al cuadrado nos queda la

ecuación sin radicales r2.(256 + r2) = 40000 – 400.r2 + r4 que nos aporta la ecuación

656.r2 = 40000 donde r2 = 61.

Calculamos el volumen

32

10223

16.61.14,3

3

..dm

hrV

.

PRUEBA 51

1.- La ecuación puede tomar la forma siguiente 3

2cos.3

cos.2

2sen

33

x

x

x

donde se pueden igualar

los exponentes y a la vez hacemos sustituciones 3

)sen21(3

cos2

cossen2 2 x

x

xx

simplificando llegamos a sen x = 1 – 2.sen2x ; ecuación cuadrática que se puede escribir de

la manera 2.sen2x + sen x – 1 = 0 que admite la descomposición en

factores (sen x + 1).(2.sen x – 1) = 0 y nos aporta que 1sen x ó 2

1sen x .

Las posibles soluciones son x = 2700 ó x = 300 ó x = 1500 pero la primera no pertenece al

dominio de la ecuación original. Solución 300 + k.3600 ; 1500 + k.3600 ;

con k entero.

2.- a) El lado del cuadrado mide necesariamente 24cm. Basándonos en el área del triángulo

rectángulo ADH podemos calcular DH = 14cm y con ello, por resta de segmentos, calcular

EH = 6,0cm y HC = 10cm.

Los triángulos EHF y BHC son rectángulos en E y C respectivamente; se cumple además que

EFH = HBC (alternos entre paralelas). Se cumple ahora que

EHF BHC (tienen dos ángulos respectivamente iguales). Podemos trabajar con los

segmentos proporcionales HB

HF

HC

EH

BC

EF y sustituyendo nos queda

HB

HFEF

10

6

24.

Page 102: PRUEBA 1 - math.cubava.cumath.cubava.cu/files/2015/05/solucionario-libro-jacinto.pdf · g 9 2x 3 x 2 y ahora podemos ... llegamos a la conclusión parcial y = 8 ó y = 1 para los

102

Calculamos entonces cmEF 4,1410

24.6 con lo podemos calcular el área del

rectángulo EFGC . A = 14,4.16 = 230,4. El área es de 230cm2.

b) Aplicando el teorema de Pitágoras en el BHC podemos calcular

cmHCBCHB 266761024 2222

y entonces, en la proporción, es fácil

decidir que cmHF 6,1510

26.6 . La longitud de BF se tiene como

HFHBBF = 26 + 15,6 = 41,6cm.

3.- El número de dos cifras puede escribirse como n = 10a + b.

Podemos plantear, por la propia operación de división que 10a + b = 6.(a + b) + 3 donde

aparece una primera ecuación que simplificada nos queda 4a – 5b = 3.

El número con las cifras intercambiadas será n’ = 10b + a ; ahora podemos hallar la

desigualdad 10b + a > 4.(a + b) que para poder obtener la igualdad (ecuación) tenemos que

sumar 9 al miembro derecho: 10b + a = 4.(a + b) + 9. El sistema de dos ecuaciones puede

quedar entonces 4a – 5b = 3 ; 3a – 6b = –9 que al resolverlo nos entrega las soluciones b = 5

y a = 7. Entonces el número buscado es el 75.

4.- Primeramente calculamos 1444

.5

tantantan (fórmula de reducción en el

tercer cuadrante) la desigualdad quedará entonces 12

45log 5,0

x

x.

Como la base del logaritmo es 0,5 < 1 entonces para convertir a potencia debemos invertir el

sentido de la desigualdad.

2

1

2

45

x

x y, comparando con cero y efectuando en el miembro izquierdo, nos queda

0)2.(2

2810

x

xx, podemos llegar a la desigualdad 0

)2.(2

109

x

x que al resolverla nos

conduce a 29

10 x .

Pero no podemos olvidar que debe cumplirse 02

45

x

x para que el logaritmo tenga sentido y

esto nos sugiere que 5

4x ó 2x . Por lo que al intersectar los intervalos

+ –10/9 – 2 +

O/////////////////////////////O

+ –4/5 – 2 +

////////////////////////////////////OO///////////////////////

Concluimos que la respuesta final es x reales con –5

4

9

10 x .

Page 103: PRUEBA 1 - math.cubava.cumath.cubava.cu/files/2015/05/solucionario-libro-jacinto.pdf · g 9 2x 3 x 2 y ahora podemos ... llegamos a la conclusión parcial y = 8 ó y = 1 para los

103

5.- Debemos comenzar calculando, en el plano base, la arista “a” del prisma. Para ello

haremos una descomposición del área del rectángulo AQPD de la siguiente forma:

a2 + 5a = 234 lo que nos lleva a una ecuación cuadrática con soluciones a = 13 ó

a = –18. En el BQG, que es rectángulo en Q pues GQ es altura, podemos aplicar el teorema

de Pitágoras y calcular cmBQBGGQ 1214425169513 2222

.

Entonces el volumen del prisma es 3322 03,2202812.13. dmcmhaV .

El área del rombo ABGF es fácil de calcular: A = a.h = 13.12 = 156cm2.

Entonces el área total del cuerpo será AT = 4.169 + 2.156 = 988cm2.

PRUEBA 52

1.- La ecuación se puede expresar como 2

1log.

2

1log xx . Considerando el

dominio para las expresiones vemos que log x 0 necesariamente para que pueda estar

definida la raíz y entonces en el miembro izquierdo tendremos la suma de dos expresiones que

son no negativas , por lo que la ecuación no tiene soluciones ya que el miembro derecho es

una expresión negativa.

Si intenta resolver la ecuación sin tener presente este análisis inicial, de encontrar posibles

soluciones, seguramente ninguna satisface la ecuación original. Pruebe!

2.- Trazamos los radios AO y 'BO . Trazamos además el segmento 'OO , compuesto por la

suma de los dos radios, por lo que 'OO = 16cm. Si trazamos por O un segmento perpendicular

a 'BO se forma un triángulo rectángulo, donde su cateto menor, por resta de los radios, mide

8,0cm. Descubrimos que el ángulo que se opone al cateto menor es de 300 (es la mitad de la

hipotenusa) por lo que el cateto mayor mide cm3.8 . Este cateto mayor es la altura del

trapecio rectángulo AOO’B. El problema ya se convirtió en una simple resta de áreas.

Calculamos el área del trapecio

27,11073,1.643.8.2

412.

2

'cmh

AOBOAT

.

Calculamos el área de los sectores circulares

2

0

02

7,163

16.14,3

360

120.4.cmAS

.

2

0

02' 4,75

6

144.14,3

360

60.12.cmAS

. Restando las áreas obtenemos

2' 6,18)4,757,16(7,110)( cmAAAA SSTR .

Page 104: PRUEBA 1 - math.cubava.cumath.cubava.cu/files/2015/05/solucionario-libro-jacinto.pdf · g 9 2x 3 x 2 y ahora podemos ... llegamos a la conclusión parcial y = 8 ó y = 1 para los

104

3.- La expresión no admite los ángulos axiales, por lo que x k.(/2) ; k entero.

En el miembro izquierdo tenemos:

tanxx

x

xx

x

xx

xx

cos

sen

cos.sen.6

sen.6

cos.sen.2.3

1)sen.21(sen.4 222

.Queda probada la identidad.

4.- Denotemos los ejercicios por cada día de la siguiente manera:

LUNES: x

MARTES: x + 2

MIÉRCOLES: (x + 2) + 2 = x + 4

El producto de todos los ejercicios es x.(x + 2).(x + 4) = x3 + 6x2 + 8x

La suma de todos los ejercicios es x + (x + 2) + (x + 4) = 3x + 6

Podemos ahora plantearnos resolver la ecuación 63263

86 23

x

x

xxx. Donde

eliminando el denominador, efectuando y ordenando llegamos a la ecuación más sencilla

x3 – 3x2 – 34x – 48 = 0 que admite la descomposición en factores

(x + 2).(x + 3).(x – 8) = 0. Solamente podemos tomar la solución x = 8 pues no deben existir

cantidades negativas de ejercicios. El total de ejercicios que resolvió el alumno será de

8 + 10 + 12 = 30. Resolvió 30 ejercicios en los tres días.

5.- En la pirámide podemos calcular el área de la base y luego calcular su altura. Conocemos

la longitud de los catetos del triángulo base. 2962

16.12cmAB .

En la pirámide se cumple que 3

.hAV B donde despejamos cm

A

Vh

B

1096

320.3.3

El lado mayor del triángulo base es la hipotenusa del triángulo rectángulo y la podemos

calcular mediante el teorema de Pitágoras cmD 204001442561216 22

Entonces el radio medirá 10cm. 3322 14,3314010.10.14,3.. dmcmhrV .

PRUEBA 53

1.- La ecuación se puede expresar como xx

x

x

xsen

2cos

2sen

cos

sen que sustituyendo por las

respectivas fórmulas del seno y el coseno del ángulo duplo queda

xx

xx

x

xsen

1cos.2

cos.sen.2

cos

sen2

.

Se aprecia que sen x = 0 es una posible solución y entonces la ecuación queda

11cos.2

cos.2

cos

12

x

x

x.

Page 105: PRUEBA 1 - math.cubava.cumath.cubava.cu/files/2015/05/solucionario-libro-jacinto.pdf · g 9 2x 3 x 2 y ahora podemos ... llegamos a la conclusión parcial y = 8 ó y = 1 para los

105

Eliminando denominadores, efectuando y ordenando llegamos a la ecuación

2.cos3x – cos x + 1 = 0 que aplicando el método de Ruffini la descomponemos en

(cos x + 1).(2.cos2x – 2.cos x + 1) = 0 donde la ecuación cuadrática no tiene soluciones reales,

pues D=b2 – 4ac = (–2)2 – 4.2.1= 4 – 8 = – 4 <0.

sen x = 0 nos implica x = 0 ; x = ó x = 2.

cos x = –1 nos implica x = . (repetida).

2.- a) Los triángulos rectángulos DCE y BCF tienen ambos un ángulo de 300 y un cateto de

igual longitud (lado del cuadrado); podemos afirmar que DCE = BCF (tienen

respectivamente iguales dos ángulos y el lado comprendido).

b) Podemos decir entonces que CFEC = 20cm (miden el doble de DE = 10cm que se

opone al ángulo de 300) y entonces cmDC 3.10 (se opone al ángulo de 600). Calculamos

el perímetro del cuadrado, cmP 2,6973,1.403.10.4

c) Se puede calcular cmDEADEA 3,73,773,0.10)173,1.(10103.10

65,262

3,53

2

)3,7( 2

A . El área del EAF es 27cm2.

3.- Es indispensable que 3x – 13 > 0 ; por lo que para todos los casos 3

13x .

Ahora establecemos una diferenciación de casos.

Cuando x – 4 > 1 ; o sea x > 5 nos queda 3x – 13 x – 4 que nos conduce a la desigualdad

5,42

9x ; lo que es imposible según la premisa x > 5.

Cuando x – 4 < 1 ; o sea x < 5 nos queda 3x – 13 x – 4 que nos lleva a la desigualdad

5,42

9x que sí es posible considerarlo pues x < 5.

La respuesta es x reales con 4,5 x < 5.

4.- Consideramos el peso del vaso y del agua (V y A)

Podemos hallar una primera ecuación V + A = 325

La otra ecuación estará dada por 1802

AV

Al resolver este sencillo sistema de ecuaciones podemos obtener A = 290 por lo que

V = 35. El vaso vacío pesa 35g.

(Este problema se puede resolver por la vía aritmética)

Page 106: PRUEBA 1 - math.cubava.cumath.cubava.cu/files/2015/05/solucionario-libro-jacinto.pdf · g 9 2x 3 x 2 y ahora podemos ... llegamos a la conclusión parcial y = 8 ó y = 1 para los

106

5.- El problema nos facilita como dato el área del rectángulo ADFC que es la cara lateral

mayor porque AC es la hipotenusa (lado mayor) de la base del prisma y podemos calcularla

aplicando el teorema de Pitágoras en el ABC

3512257844412821 2222

BCABAC . En el rectángulo ADFC

FCACA . lo que implica cmAC

AFC 10

35

350

Ahora solo nos falta hallar el radio del cilindro.

En la base podemos trazar el radio desde el punto O hasta el cateto tangente BC y se forma

un nuevo triángulo rectángulo que es semejante al triángulo rectángulo ABC

(el BCA es común). Se puede hacer el mismo trazado del radio hacia el cateto tangente

AB . Podemos apreciar que se forma un cuadrado en la base y que su lado es el radio, por lo

que podemos plantear la siguiente igualdad entre segmentos proporcionales

28

28

21

rr donde r = 12cm

Calculemos el volumen 6,452110.144.14,310.12.14,3.. 22 hrV

El volumen es 4,5dm3.

PRUEBA 54

1.- Podemos trabajar en el segundo miembro y escribir la ecuación con las bases iguales

xxx

112

2

1

2

136

por lo que podemos igualar los exponentes

xxx 112 36 donde elevamos al cuadrado, realizamos las operaciones indicadas y

llegamos a la ecuación x6 – 2x3 – x2 + 2x = 0 que puede factorizarse

x.(x5 – 2x2 – x + 2) = 0 y que se puede llegar también a

x.(x + 1).(x – 1)2.(x2 + x + 2) = 0 aplicando división sintética.

Las soluciones de la ecuación son x = 0 ó x = –1 ó x = 1 (se pueden comprobar).

2.- Hay que considerar, ante todo, que ABDB = 6,0cm (segmentos de tangente desde un

punto exterior a una circunferencia hasta los puntos de tangencia) por lo que BC = 10cm

(suma de segmentos).

Trazamos el radio OD (perpendicular en el punto de tangencia al igual que OA ).

El ABC es rectángulo en A y el ODC lo es en D, ambos tienen un ángulo común,

el OCD, y por tanto ABC ODC (tienen dos ángulos respectivamente iguales).

Page 107: PRUEBA 1 - math.cubava.cumath.cubava.cu/files/2015/05/solucionario-libro-jacinto.pdf · g 9 2x 3 x 2 y ahora podemos ... llegamos a la conclusión parcial y = 8 ó y = 1 para los

107

Podemos plantear la proporcionalidad entre los lados homólogos AC

CD

AB

OD

CB

CO

y sustituir los valores AC

ODCO 4

610 . Apreciamos que no es posible trabajar si no

calculamos cmAC 8 en el triángulo rectángulo ABC (trío de números pitagóricos)

Por lo que cmOD 0,38

4.6 . Entonces

22

2

9,91,14242

3.14,3

2

8.6..

2

1

2

..

2

1cmOD

ACABAAA CTR .

3.- a) Sustituimos la ecuación de la recta en la ecuación de la curva.

(x – 1)2 + (k.x – 2)2 = 4 . Realizamos las operaciones indicadas y podemos llegar a la

ecuación cuadrática (1 + k2).x2 + (–4k – 2).x + 1 = 0 donde tenemos que trabajar con su

discriminante D = (–4k – 2)2 – 4.(1 + k2) = 16k2 + 16k + 4 – 4 – 4k2 = 12k2 + 16k ;

para que la recta corte a la curva 12k2 + 16k > 0.

Si factorizamos obtenemos 4k.(3k + 4) > 0 que al resolverla nos aporta los valores que puede

tomar k: 3

4k ó k > 0.

b) La recta y = k.x – 2 será tangente a la curva cuando 3

4k ó k = 0.

Las rectas serán 23

4 xy ó y = –2.

4.- El problema requiere de un simple razonamiento de cálculo.

El equipo cuesta $860. A los 6 meses ya tenían reunido entre los dos $660.

Faltarán $860 – $660 = $200. En 4 meses el hermano menor completó esos $200 , por lo que

siempre aportó $50 mensuales. En los 10 meses el hermano menor aportó $500 para comprar

el equipo.

5.- Calculemos primero la altura que tiene el agua en el recipiente antes de ser introducido el

cilindro. Sabemos que 1L = 1dm3 = 1000cm3.

40000 = 50.40.h donde cmh 202000

40000 .

El volumen del cilindro se puede expresar de dos formas. Si expresamos por x la altura que

se desplaza (hacia arriba) el agua cuando se introduce el cilindro tenemos

400.(20 + x) = 50.40.x . Si resolvemos esta ecuación nos queda que x = 5cm.

La altura del cilindro es H = h + x = 20 + 5 = 25cm.

V = AB.h = 400.25 = 10000cm3 = 10dm3.

Necesitamos el radio de la base del cilindro: 400 = .r2 donde 4,12714,3

4002 r

y nos queda que r = 11,3cm.

AL = 2..r.H = 2.3,14.11,3.25 = 1774,1cm2 = 17,7dm2.

Page 108: PRUEBA 1 - math.cubava.cumath.cubava.cu/files/2015/05/solucionario-libro-jacinto.pdf · g 9 2x 3 x 2 y ahora podemos ... llegamos a la conclusión parcial y = 8 ó y = 1 para los

108

PRUEBA 55

1.- La ecuación original puede tomar la forma 2log2

2log32log 222

x

x .

Aplicando las leyes de los logaritmos podemos expresarla como

1

22 2log2

2.32log

x

x para simplificar un tanto la ecuación haremos el cambio

v = 2x quedará entonces

2

13.2

v

v. Eliminando denominadores, operando y despejando

adecuadamente llegamos a que v = 4 . Entonces 2x = 4 lo que nos conduce a que x = 2.

2.- a) El triángulo ACD es rectángulo en C (trapecio rectángulo) y se cumple que

DAC = EDA (alternos entre las paralelas del trapecio).

Falta ahora probar que DEI es rectángulo. En el hexágono regular el ABI es isósceles de

base AI y podemos obtener que AIB = 300 (suma de los ángulos interiores de un triángulo

isósceles con un ángulo interior de 1200).

Entonces EIA = 900 (por resta de 1200 – 300), luego, EID = 900. El EID es rectángulo

en I ; por tanto EID ACD (tienen dos ángulos respectivamente iguales).

b) Tenemos que DAC = ADG = AIB = 300. El triángulo GHF es equilátero de lado

HF = 4,0m por lo que AG = 8,0m y entonces GD = 16m ( AG se opone al ángulo de 300

en el DAG rectángulo en A). Si trazamos AF podemos calcular .12mACFD (por resta

de segmentos). La diagonal menor del hexágono regular mide siempre 3a donde a es el lado

del hexágono. Entonces AF = m3.4 .

3.4.2

1216

4

3..

2

2

...

HFAF

ACGDAAA EQUILATTTRAP

296,8973,1.523.523.43.56 m . El área es 90m2.

3.- Un rápido análisis del denominador, donde están las funciones tan y cot , nos conduce a

los valores inadmisibles 2

.

k ; con k entero. Partimos del miembro derecho, en el que

primeramente trabajaremos su denominador.

xxx

xx

x

x

x

x

2sen

1

cos.sen.2

sen.21sen.2

2sen

2cos

cos

sen 22

y el recíproco de esta última expresión se

convierte en sen 2x , por lo que queda demostrada la identidad.

4.- Los números son “n” y “n + 1”.

La condición se debe plantear así: (n + 1)3 – n3 = (2n)2 – 107 que realizando las operaciones

indicadas llegamos a la ecuación más simple n2 – 3n – 108 = 0 que nos entrega las soluciones

n = 12 ó n = –9. (No tomaremos la negativa para evitar complicaciones al formar la

diferencia cuando se restan los cubos).

Los números que tomaremos son entonces 12 y 13. El promedio es 12,5.

Page 109: PRUEBA 1 - math.cubava.cumath.cubava.cu/files/2015/05/solucionario-libro-jacinto.pdf · g 9 2x 3 x 2 y ahora podemos ... llegamos a la conclusión parcial y = 8 ó y = 1 para los

109

5.- a)Como CH es la altura relativa a la hipotenusa en el ABC podemos aplicar el teorema

de la altura.

HBAHCH .2

= 8.18 = 144 por lo que CH = 12cm.

Podemos calcular la altura del prisma que es CD = cm3.12 . También podemos calcular la

longitud de DH = 24cm pues es la hipotenusa en un triángulo rectángulo que tiene un ángulo

de 300, por lo que es el doble del cateto CH .

Si trazamos DB podemos probar que el HDB es rectángulo en H pues en este punto se

cumple el teorema de las tres perpendiculares; DH es oblicua y CH es la proyección. Por el

teorema de Pitágoras podemos calcular,

cmHBDHDB 309003245761824 2222

.

b)33 24,356,323873,1.1872312.

2

12.26.

2

.. dmcmCD

CHABhAV B .

PRUEBA 56

1.- Eliminamos los denominadores y obtenemos la ecuación

cos x.(sen2x + 3) = (cos x + 2).(1 – 2.sen2x + 3.sen2x)

donde cos x.(sen2x + 3) = (cos x + 2).(1 + sen2x)

cos x.sen2x + 3.cos x = cos x + cos x.sen2x + 2 + 2.sen2x

y al finalizar podemos llegar a la ecuación cos2x + cos x – 2 = 0

(cos x + 2).(cos x – 1) = 0 que implica cos x = –2 ó cos x = 1.

La primera es imposible y la segunda aporta x = 2.k. ; k entero.

2.- a) ud AB 525916342526 2222

udBC 5251694362522222

Un paralelogramo con un par de lados consecutivos iguales es un rombo.

b) 3

4

25

26

BCm ; 1

1

1

65

56

ACm

La ecuación de la recta AD nos queda 6.3

45 xy , o sea 4x – 3y = 9.

Como las diagonales son perpendiculares entre sí 1BDm .

La ecuación de BD es sencilla : y – 2 = x – 2 ; o sea y = x.

c) Intersectando las dos ecuaciones obtenemos 4x – 3x = 9; donde x = 9;

entonces también y = 9. Punto D(9 ; 9).

Page 110: PRUEBA 1 - math.cubava.cumath.cubava.cu/files/2015/05/solucionario-libro-jacinto.pdf · g 9 2x 3 x 2 y ahora podemos ... llegamos a la conclusión parcial y = 8 ó y = 1 para los

110

3.- En principio k 0. La ecuación dada puede llevarse a la forma

k.x2 + (k + 1).x + k = 0 y aplicando el discriminante llegamos a:

D = (k + 1)2 – 4k2 = k2 + 2k + 1 – 4k2 = –3k2 + 2k + 1 0

donde 3k2 – 2k – 1 0 que se descompone en (k – 1).(3k + 1) 0 y se resuelve mediante el

análisis de intervalos

+ –1/3 – 1 +

///////////////////////

encontramos que – 13

1 k .

4.- En este problema hay dos velocidades como incógnitas.

Velocidad del bote: VB Velocidad de la corriente del río : VC

10

5,1

15

CB

CB

VV

VV

6

2

12

CB

CB

VV

VV

Resolvemos el sistema sumando las ecuaciones y obtenemos VB = 8km/h ; VC = 2km/h.

5.- El radio de la esfera es 10cm. Podemos apreciar que se forma un triángulo rectángulo

isósceles con el radio de la esfera, la distancia entre el centro de la esfera y el centro de la base

del cono, y el radio de la base del cono.

Por lo que el radio de la base del cono mide cm2.5 (Teorema de Pitágoras). La altura del

cono es cm2.510 (suma de segmentos).

Se puede calcular el volumen del cono

9,894

3

1,17.50.14,3

3

2.510.2.5.14,3

3

..2

2

hr

V

.

El volumen del cono es 895cm3.

Para calcular el área lateral del cono necesitamos calcular su generatriz.

22

2.5102.5 g (Teorema de Pitágoras en el triángulo rectángulo que genera al

cono).

5,18342292501,1750 2 g .

5,4095,18.41,1.5.14,35,18.2.5.14,3.. grAL .

El área lateral del cono es 410cm2.

Page 111: PRUEBA 1 - math.cubava.cumath.cubava.cu/files/2015/05/solucionario-libro-jacinto.pdf · g 9 2x 3 x 2 y ahora podemos ... llegamos a la conclusión parcial y = 8 ó y = 1 para los

111

PRUEBA 57

1.- La vía más eficiente: Despejar yx en la segunda ecuación y sustituir en la primera, lo

que nos conduce a que 3

yx . Con este último resultado podemos sustituir en cualquiera de

las dos ecuaciones iniciales y se obtiene la ecuación y2 = 48y que nos da las soluciones y = 0

ó y = 48; estas nos aportan respectivamente x = 0 ó x = 16.

Pares ordenados (0 ; 0) ó (16 ; 48).

2.- a) En la figura se puede localizar el punto medio M del segmento ED y trazar el segmento

MA . En el EAD, rectángulo en A, el punto M está en la mitad de la hipotenusa por lo que es

el centro de la circunferencia circunscrita al triángulo (uno de los recíprocos del teorema de

Tales) por lo que EMMDAM (radios).

Entonces AMAC y el ACM es isósceles de base CM . Como el BCE es rectángulo en

B podemos calcular fácilmente que AEM = 720 y luego, como que AME es isósceles de

base AC entonces AME = 360 y también ACE = 360.

b) En el ABC, rectángulo en B, ACB = 540 y aplicando AC

ABACB sen podemos llegar

a que AB = 10.sen 540 = 10.0,809 = 8,09 = 8,1 m.

Otra vía :

De no utilizarse esta construcción auxiliar.

a) 018sen.20AE . En el ACE utilizamos la ley de los senos y podemos plantear

0108sen

10

sen

ACE

AE, o sea,

00000000

36sen18cos.18sen.2)1890sen(.18sen.210

108sen.18sen.20sen ACE

y como el ángulo ACE es agudo entonces su amplitud es de 360.

b) Queda igual.

3.- En principio tiene que cumplirse que x + 27 > 0; o sea x > –27 y también que

16 – 2x > 0; o sea 2x < 16 lo que implica que x < 8. Pero como x > 0 entonces el rango

inicial es 0 < x < 8.

Convertimos a un solo logaritmo en el miembro izquierdo y podemos arribar a la nueva

desigualdad xx

x

216

27 (la base es > 1).

Page 112: PRUEBA 1 - math.cubava.cumath.cubava.cu/files/2015/05/solucionario-libro-jacinto.pdf · g 9 2x 3 x 2 y ahora podemos ... llegamos a la conclusión parcial y = 8 ó y = 1 para los

112

Comparamos con cero y después de restar, considerando el denominador común 16 – 2x

nos queda 0)8.(2

27152 2

x

xx que factorizando y multiplicando en el denominador por –1

para facilitar el trabajo obtenemos 0)8.(2

)3).(92(

x

xx y que al resolverla obtenemos los

intervalos siguientes:

– 3 + 9/2 – 8 +

O//////////////////////OO////////////////////////

O/////////////////////////////////////////////////////////////O

0 8

que nos lleva a la respuesta 3 < x < 4,5 ó x > 8 de los que únicamente podemos considerar,

debido a la intersección con el rango inicial, que 3 < x < 4,5.

4.- Si ganaron 20% de lo que le costaron los cerdos tres meses atrás entonces con los $9090

obtuvieron lo que les costaron y la ganancia de 20%. Consideremos el costo como “c”.

Podemos plantear la ecuación 9090 = c + 20%.c lo que es igual a

9090 = 120%.c. Calculamos 757512

10.9090c .

Restando ahora 9090 – 7575 = 1515 obtenemos lo que ganaron en la venta pero a esta cifra

hay que restar el monto invertido en el proceso de ceba.

Obtenemos que en este negocio ganaron $1115,00.

5.- a) Se puede apreciar que la parte cortada fue una pirámide regular. La sección rayada es un

triángulo equilátero del cual podemos calcular el lado, arista de la base de la pirámide, que es

2.32. aL (hipotenusa de un triángulo isósceles). Aplicando la fórmula para el área del

triángulo equilátero nos queda 2

22

79,74

73,1.18

4

3.2.3

4

3.cm

LAB .

b) El trabajo serio consiste ahora en calcular la altura de la pirámide cortada. La altura de la

pirámide cae en la base en el punto de intersección de las medianas que divide a cualquiera de

ellas en la razón 1 : 2 ; en este caso la mediana y la altura (en el triángulo base) son iguales, a

esta altura la llamaremos “h” y la altura de la pirámide la denotaremos por “H”.

En un triángulo equilátero se cumple la relación 3.2 Lh o sea

2

6.3

2

3.2.3

2

3.

Lh .

En el triángulo de apoyo de una arista lateral a = 3cm de la pirámide, la proyección de la

arista sobre la base es 62

6.3.

3

2.

3

2h ; entonces en este triángulo podemos calcular

369632

2 H (teorema de Pitágoras).

3335,2115,4260

3

73,1.79,76

3

.2 cm

HAaVVV B

PCR .

Page 113: PRUEBA 1 - math.cubava.cumath.cubava.cu/files/2015/05/solucionario-libro-jacinto.pdf · g 9 2x 3 x 2 y ahora podemos ... llegamos a la conclusión parcial y = 8 ó y = 1 para los

113

PRUEBA 58

1.- Igualamos las funciones y elevamos ambos miembros al cuadrado; nos quedará

121.44 xxx . Aislamos el radical para de nuevo elevar al cuadrado xx .2 ;

llegamos a la ecuación x2 = 4x que aporta las posibles soluciones x = 0 ó x = 4.

Como x = 0 es una raíz extraña entonces las coordenadas del punto de intersección

son (4 ; 3).

2.- Trabajaremos directamente en la expresión, operando y sustituyendo

12cos

2cos

1cos.2

sencos

3cos4cos.3

sensen

cos.cos.sen.2.5,0

2

22

22

2

x

x

x

xx

xx

xx

xxx

.

Entonces h(x) = 1 para todos los valores x de su dominio. Es constante.

3.- Expresemos por “n” el número de personas y por “a” el aporte de cada una.

Podemos plantear la igualdad n.a = 1200.

La otra ecuación puede obtenerse de a > n por lo que a – 194 = n.

Sustituyendo directamente la ecuación segunda en la primera nos queda

(a – 194).a = 1200 que nos conduce a la ecuación a2 – 194a – 1200 = 0 que al resolverla

obtenemos a = 200 ó a = –6 (esta última imposible).

Llegamos entonces a que son 6 personas y si quedaron 2 hombres sin pareja entonces había 2

mujeres y 4 hombres en el grupo.

4.- a) ¡Tenemos que probar que ACO = DCB !

Como AOB = DOB (opuestos por el vértice) entonces ACO ODB (tienen los ángulos

base respectivamente iguales). El ángulo CDB es común para los ODB y CDB por lo que

ODB CDB (tienen los ángulos base respectivamente iguales).

Entonces ACO CDB (propiedad transitiva); lo que nos conduce a que sus ángulos

opuestos a la base son iguales. Por lo tanto el punto O está en la bisectriz del C.

b) En todos estos triángulos semejantes la razón entre el lado y la base es k = 2 pues en el

CDB puede asegurarse. Por tanto en el ODB tenemos que OD = 2,0m.

Podemos calcular ODCDCO = 8 – 2 = 6,0m y entonces AO = 3,0m.

Finalmente OBAOAB 3 + 4 = 7,0m.

5.- a) Si trazamos DB podemos apreciar que QPDB (B es punto medio y DQP isósceles).

Como DB es la proyección de EB sobre el plano base entonces en el punto B se cumple el

teorema de las tres perpendiculares por lo que EBP es rectángulo en B.

Como BP = 16cm entonces podemos calcular EB en el triángulo rectángulo aplicando el

teorema de Pitágoras cmBPEPEB 3090025611561634 2222

.

Page 114: PRUEBA 1 - math.cubava.cumath.cubava.cu/files/2015/05/solucionario-libro-jacinto.pdf · g 9 2x 3 x 2 y ahora podemos ... llegamos a la conclusión parcial y = 8 ó y = 1 para los

114

b) Consideremos que “a” sea la arista lateral del cubo. Como conocemos la medida de la

diagonal interior podemos plantear 3.aEB donde 3.103

30a

33

3 519073,1.30003.3.10003.10 cmaV

V = 5,2dm3.

PRUEBA 59

1.- Podemos comenzar transformando las bases xx 2cos.2sen.54 22.2 donde

xx 2cos.2sen.54 22 . Igualando los exponentes y sustituyendo cos2x podemos obtener la

ecuación 2.sen2x + 5.sen x + 2 = 0 que se descompone en

(sen x + 2).(2.sen x + 1) = 0 que conduce a sen x = –2 (imposible) ó 2

1sen x (que tiene

soluciones en el tercero y en el cuarto cuadrante) x = 2100 + k.3600 ;

x = 3300 + k.3600 ; k entero.

2.- El lado del cuadrado mide 6,0cm y por resta de segmentos

DCCEED = 14 – 6 = 8,0cm.

Podemos aplicar el teorema de Pitágoras en el ADE, rectángulo en D,

cmADDEAE 1010068 2222

.

El DFC también es rectángulo en C y DCF = AED (alternos entre las paralelas) entonces

DFC ADE (tienen dos ángulos respectivamente iguales). Planteamos la proporción entre

los lados homólogos ED

DC

AD

DF

AE

CF , o sea,

8

6

610

DFCF donde podemos calcular

5,78

6.10CF ; 5,4

8

6.6DF

P = 12 + 10 + 8 + 4,5 + 7,5 = 42cm.

25,735,1324362

5,4.6

2

6.836 cmAT .

3.- Primeramente debe cumplirse que x2 – 5x + 6 > 0 o sea (x – 3).(x – 2) > 0 y que al

resolver la desigualdad tenemos x < 2 ó x > 3.

Para que la raíz cuadrada esté definida debe cumplirse que 0)65(log1 2

2 xx , o sea,

1)65(log 2

2 xx ; que ocurre solo si x2 – 5x + 6 2. Comparando con cero y

factorizando nos queda (x – 4).(x – 1) 0 y que al resolverla nos aporta 1 x 4.

Page 115: PRUEBA 1 - math.cubava.cumath.cubava.cu/files/2015/05/solucionario-libro-jacinto.pdf · g 9 2x 3 x 2 y ahora podemos ... llegamos a la conclusión parcial y = 8 ó y = 1 para los

115

Al intersectarla con la primera condición nos quedan los siguientes intervalos

+ 2 – 3 +

/////////////////////////////////////OO///////////////////////////////////////////

///////////////////////////////////////////////////////////////////////

+ 1 – 4 +

y nos conduce a las soluciones 1 x < 2 ó 3 < x 4.

4.- Denotemos las distancias por AB = x ; BC = y ; CA = z .

Se forma el sistema con tres variables a partir del triángulo.

(I) z + y = 18

(II) x + z = 16

(III) x + y = 14

Se pueden utilizar varias vías de resolución, una con mucha eficiencia es:

Despejar z en (I) y sustituir en (II). Luego resolver el sistema de dos ecuación con dos

variables que aparece entre la nueva ecuación y la ecuación (III).

Las distancias son 10km; 8km y 6km.

5.- Necesitamos el radio del cilindro y lo podemos calcular por el área de la base 2.rAB

entonces 1256 = 3,14.r2 donde 40014,3

12562 r y obtenemos r = 20dm.

El radio de la esfera es R = 29dm.

Necesitamos la altura del cilindro y podemos calcular su mitad en el triángulo rectángulo que

se forma entre el centro de la esfera, el centro de la base del cilindro y un radio de la esfera

aplicando el teorema de Pitágoras.

dmh

2144140084120292

22 ; 212

h.

Por tanto h = 42dm. 2222 6,10510563841.14,3.429.14,3.4..4 mdmRAE .

.5,1051055042.20.14,3.4)...2.(2.2 22 mdmhrAL

El área de la esfera es mayor.

PRUEBA 60

1.- En el miembro derecho de la ecuación podemos escribir 25.25 = (2.5)5 = 105 y la x puede

ser sustituida por xlog10 . La ecuación queda

5log1log.41010

xx. Igualando los

exponentes, ya que las bases son iguales, y aislando la raíz obtenemos xx log51log.4

donde elevando al cuadrado y ordenando llegamos a la ecuación

log2x – 14.log x + 24 = 0 que admite la descomposición en factores

(log x – 12).(log x – 2) = 0 donde log x = 12 ó log x = 2.

La primera igualdad aporta x = 1012 (raíz extraña) y la segunda x = 102 = 100.

Page 116: PRUEBA 1 - math.cubava.cumath.cubava.cu/files/2015/05/solucionario-libro-jacinto.pdf · g 9 2x 3 x 2 y ahora podemos ... llegamos a la conclusión parcial y = 8 ó y = 1 para los

116

2.- Esta claro que ABC y DEF son rectángulos. También se cumple que

MDB = MBD (ambos son adyacentes a ángulos iguales). Ocurre ahora que MDB es

isósceles (tiene dos ángulos iguales) y entonces MBMD (se oponen a ángulos iguales en un

mismo triángulo). Como BEAD entonces DBBEDBAD (monotonía en la suma de

segmentos) luego, DEAB . Concluimos que ABC = DEF (tienen respectivamente

iguales dos ángulos y el lado comprendido). Se cumple entonces que,

CBFD (lados correspondientes en triángulos iguales)

entonces MBCBMDFD (pues MBMD ) y nos queda que .CMFM ( por resta

de segmentos). ¡ Queda probado!

3.- Denotemos las cajas de Roberto por R y las de Francisco por F.

Si Francisco dona la cuarta parte de sus cajas se queda con 3/4 de sus cajas.

Si Roberto dona 12,5% de sus cajas se queda con 87,5% de sus cajas.

Como solo puede donar uno de los dos entonces obtenemos las ecuaciones,

47.4

3 RF y 48%.5,87 RF

Que se puede llevar a un sistema más sencillo 3F + 4R = 88

8F + 7R = 384

y que al resolverlo nos aporta las soluciones R = 32 y F = 20.

Entre los dos recogieron 52 cajas de naranjas.

4.- a) Hallaremos las tres distancias

17116)1()4()54()62();(

17161)4()1()51()65();(

1899)3(3)41()25();(

2222

2222

2222

CAd

CBd

BAd

El triángulo ABC es isósceles de base AB .

b) 13

3

52

14

ABm y

2

14;

2

25M o sea

2

5;

2

7M .

La pendiente de la recta bisectriz es m’ = 1 (rectas perpendiculares) y como ya tenemos las

coordenadas del punto medio de AB podemos formar una ecuación de la recta bisectriz:

2

7.1

2

5xy ,o sea, x – y = 1. Cuando x = 0 entonces y = –1.

Cuando y = 0 entonces x = 1. Los puntos son (1 ; 0) y (0 ; –1).

5.- Si el ABC es equilátero y nos dan su área podemos calcular su lado, que es en este caso

la arista de la base de la pirámide. Llamaremos “a” a la arista. 4

3.2aA y ahora

.4

3.173

2a ; despejando “a” nos queda: .400

3

3.3.100.4

3

3.

3

173.42 a

Donde a = 20cm.

Page 117: PRUEBA 1 - math.cubava.cumath.cubava.cu/files/2015/05/solucionario-libro-jacinto.pdf · g 9 2x 3 x 2 y ahora podemos ... llegamos a la conclusión parcial y = 8 ó y = 1 para los

117

a) Se cumple que .SBSA (son oblicuas que tienen proyecciones iguales sobre ).

Entonces el ABS es isósceles de base .AB

Tenemos que hallar la altura h del ABC (cara de la pirámide). Esta altura se toma a partir

del punto de AB hasta S. Como SAC = 450 entonces SC = 20cm y

.2.20 cmSA (hipotenusa de un triángulo rectángulo isósceles).

Consideremos por M el punto medio de AB . El AMS es rectángulo en M ( SM es altura de

esa cara) y podemos aplicar el teorema de Pitágoras para calcular SM .

cmAMASSM 5,2665,2.107.10700100800102.20 2222

.2652

5,26.20

2

. 2cmSMAB

A

b) En el punto M se cumple el teorema de las tres perpendiculares, entonces en dicho punto se

encuentra el ángulo de inclinación de esa cara pues SM es una línea de máxima pendiente. En

el SMC, rectángulo en C ( SC es altura, perpendicular a todas las rectas del plano que pasen

por C) podemos aplicar la función seno.

SM

SCSMC sen y entonces 755,0

5,26

20sen SMC .

Podemos calcular que SMC = 49,10 utilizando la tabla o calculadora.

PRUEBA 61

1.- La primera ecuación puede escribirse como 2510

10)log(

2

yx y la segunda como

4log10log.log yxyx . Ahora el sistema queda reducido a las ecuaciones

14

yx

; 40log.log yxyx .

Donde nos queda; que x – y = 4 ; 4.(x + y) = 40.

Ahora el sistema es mucho más sencillo x – y = 4 ; x + y = 10 y que al resolverlo, sumando

ordenadamente ambas ecuaciones nos queda x = 7 ; y = 3.

2.- Trazamos el segmento AO ; debemos probar que CAO = OAB.

En los triángulos AEB y ACD tenemos: A común; ADAE y ABAC (datos).

Entonces AEB = ACD (tienen respectivamente iguales dos lados y el ángulo comprendido)

y entonces OEC = ODB (elementos correspondientes en triángulos iguales).

En los triángulos EOC y DOB tenemos: DBEC (resta de segmentos iguales) y

DOB = EOC (opuestos por el vértice). Entonces ECO = DBO (terceros ángulos)

Implica que EOC = DOB (tienen respectivamente iguales dos ángulos y el lado

comprendido) y entonces ODEO (elementos correspondientes en triángulos iguales).

Page 118: PRUEBA 1 - math.cubava.cumath.cubava.cu/files/2015/05/solucionario-libro-jacinto.pdf · g 9 2x 3 x 2 y ahora podemos ... llegamos a la conclusión parcial y = 8 ó y = 1 para los

118

En los triángulos AOE y AOD tenemos: lado AO común, ADAE (dato)

y ODEO .

Entonces AOE = AOD (tienen los tres lados iguales), luego CAO = OAB (elementos

correspondientes en triángulos iguales).

El punto O está en la bisectriz del A.

3.- La cantidad de pinos la denotaremos por “P” y al total de árboles “A”.

La primera ecuación es muy fácil P = 40%.A.

En la segunda igualdad hay que tener cuidado P + 22 = 44%.(A + 30).

El sistema nos puede quedar más fácil de resolver si lo llevamos a:

2201125

025

AP

AP

Al resolver el sistema obtenemos que P = 88. Pero estos eran los pinos en la parcela al inicio,

después se sembraron 22. Entonces actualmente hay 110 pinos.

4.- Hallando los valores inadmisibles:

.2

kx para la tangente

sen 2x + 2.sen x = 0 implica 2.sen x.(cos x + 1) = 0 donde x = k.

cos 2x + 2.cos x + 1 = 0 implica 2.cos x.(cos x + 1) = 0 ya están considerados.

Los valores inadmisibles son 2

.

kx ; con k entero.

Partimos del miembro izquierdo

)1.(coscos.2

1

)1.(coscos.sen.2

sen

)1.(cossen.2

cos

sen

xxxxx

x

xx

x

x

quedando probada.

5.- Podemos plantear considerando la arista de la base por “a” y la altura de los triángulos

isósceles laterales por “h”:

482

..42

haa donde queda a2 + 2a.h = 48. El triángulo de apoyo de las aristas laterales

es rectángulo isósceles, es por ello que la altura de la pirámide es igual a la mitad de la

diagonal del cuadrado base o sea 2

2.aH . En el triángulo de apoyo de la cara lateral

podemos hallar una relación entre a y h. Por el teorema de Pitágoras

2

3.

22

2.22

aaah

.

Page 119: PRUEBA 1 - math.cubava.cumath.cubava.cu/files/2015/05/solucionario-libro-jacinto.pdf · g 9 2x 3 x 2 y ahora podemos ... llegamos a la conclusión parcial y = 8 ó y = 1 para los

119

Retomamos la primera igualdad y sustituimos la 2

3.ah

482

3..22 a

aa nos queda 4831.2 a donde podemos calcular a = 4,2dm.

32 4,1741,1.2,4.6,17.6

1

2

2.2,4.)2,4.(

3

1

3

.dm

HAV B .

PRUEBA 62

1.- Comenzaremos racionalizando el miembro izquierdo multiplicando por la conjugada

xxx

xxxxsen

sen1sen

sen)1.(sensen.21sen

. De donde podemos obtener la ecuación más

simple xxx sensen.2sen1 2 . Después de elevar al cuadrado ambos miembros y

ordenar la ecuación llegamos a (sen x + 1).(3.sen x – 1) = 0 donde

sen x = –1 ó sen x = 3

1. La primera no pertenece al dominio de la ecuación original y de la

segunda obtenemos por la tabla x = 19,740 ó x = 160,530.

2.- a) Los triángulos CEB y ABD son rectángulos (teorema de Tales y radio en punto de

tangencia respectivamente). Podemos afirmar que ABC es isósceles de base AB (la altura

cae en el punto medio de AB ) y entonces DAB = CBE, por lo que

ABD ~ CEB (tienen dos ángulos respectivamente iguales).

b) Se cumple que AB

CB

DB

EC

AD

EB (elementos proporcionales). El lado EC se puede

calcular aplicando el teorema de Pitágoras en el ECB.

cmCBEBEC 183245769002430 2222

. Se puede calcular la longitud

de la hipotenusa AD porque C es su punto medio ( CM es paralela media pues M es punto

medio y CM DB ) por lo que CDCBAC .

Entonces AD = 48cm.

Nos queda entonces que ABDB

2418

48

30

Despejando llegamos a .8,2830

18.48cmDB ;

30

24.48AB 38,4cm

25532

8,28.4,38

2

.cm

BDABA .

Page 120: PRUEBA 1 - math.cubava.cumath.cubava.cu/files/2015/05/solucionario-libro-jacinto.pdf · g 9 2x 3 x 2 y ahora podemos ... llegamos a la conclusión parcial y = 8 ó y = 1 para los

120

c) La distancia de A a la circunferencia es ,AE pues A está en la prolongación del

diámetro . Restando segmentos EBABAE = 38,4 – 30 = 8,4cm.

3.- Las horas de Miguel: M. Las horas de Julio: J.

Aparecen las ecuaciones M – 9 = 2J ; M + 20%.M + J + 11 = 100

Se pueden expresar más sencillas por

M – 2J = 9

6M + 5J = 445

Al resolver el sistema obtenemos las soluciones M = 55 y J = 23.

Julio tiene 23 horas y Miguel tiene 55 horas.

4.- Para que todos los puntos del gráfico de f estén por debajo del eje de las abscisas tiene que

cumplirse f(x) < 0, o sea 022

2

23

xx

xxx.

La expresión puede quedar totalmente factorizada

0)1.(

)2).(1).(1(

xx

xxx ; o sea 0

)2).(1(

x

xx ; con x 1.

Para resolver la igualdad analizamos los intervalos con los signos

– –1 + 0 – 2 +

//////////////////////////OO//////////////////////////O

Y se obtienen los intervalos x < –1 ó 0 < x < 2 y debemos hacer referencia

a que x 1.

5.- El pentágono regular está formado por 5 triángulos isósceles iguales. En cada uno de ellos

el ángulo que se opone al lado base (arista de la base) tiene una amplitud .725

360 00

Podemos calcular que los ángulos base tienen una amplitud de 540 (suma de ángulos interiores

de un triángulo isósceles).

Ahora podemos hallar la altura relativa a la base. 5

54 0 htan donde

cmtanh 88,6376,1.554.5 0 . El área de la base del prisma es

21722

88,6.10.5 cmAB

;

33 72,1172010.172. dmcmhAV B .

Page 121: PRUEBA 1 - math.cubava.cumath.cubava.cu/files/2015/05/solucionario-libro-jacinto.pdf · g 9 2x 3 x 2 y ahora podemos ... llegamos a la conclusión parcial y = 8 ó y = 1 para los

121

PRUEBA 63

1.- Podemos llevar todos los términos a la misma base para luego operar con los exponentes

)4.(6

28

2

1

)2.(3

12

2

22:2

x

x

x

x

x

x

. Podemos restar los exponentes en el

miembro izquierdo y luego plantear que )4.(6

28

2

1

)2.(3

12

2

x

x

x

x

x

x.

Considerando que el mínimo común denominador es 6.(x + 2).(x – 2) podemos eliminar los

denominadores 2.(x + 1).(x + 2) – 6.(x – 1).(x – 2) = 28 – x2 y llegar entonces a la ecuación

cuadrática siguiente: x2 – 8x + 12 = 0 que admite las soluciones

x = 6 ó x = 2 (esta última no pertenece al dominio).

2.- Es necesario dibujar un paralelogramo cualquiera y trazar las bisectrices de sus ángulos

interiores. Los ángulos consecutivos de un paralelogramo son suplementarios (suman 1800)

por tanto sus mitades (que aparecen con las bisectrices) son ángulos complementarios (suman

900). Entonces el tercer ángulo del triángulo que se forma es de 900. De igual modo ocurre con

todos los demás triángulos que se forman con estas condiciones. Por tanto siempre se forma un

rectángulo al trazar las cuatro bisectrices de los ángulos interiores de un paralelogramo.

3.- Denotaremos ángulo mayor: Z ; ángulo mediano: Y ; ángulo menor: X.

en todo triángulo se cumple X + Y + Z = 1800 (suma de ángulos interiores).

Pero Z + Y > X por lo que escribimos Z + Y – 900 = X para poder tener la ecuación.

También X + Z < 2Y por lo que tendremos X + Z + 150 = 2Y.

El sistema nos queda X + Y + Z = 1800

X – Y – Z = –900

X – 2Y + Z = –150

Que es muy fácil resolverlo si observamos las dos primeras ecuaciones.

Los valores que se obtienen son X = 450 ; Y = 650 ; Z = 700.

Nos piden calcular tan (650 + 700) = tan 1350 = –tan (1800 – 450) = –tan 450 = –1 (fórmula de

reducción de la tangente para el segundo cuadrante).

4.- No hay valores inadmisibles. Partiremos del primer miembro.

2

2cos

2

sencos

2

cossen

2

cos

2

sen

2

2.cos

2

2.sen.

2

2.cos

2

2.sen

45sen.cos45cos.sen.45sen.cos45cos.sen

222222

0000

xxxxxxx

xxxx

xxxx

Queda entonces demostrada la identidad.

Page 122: PRUEBA 1 - math.cubava.cumath.cubava.cu/files/2015/05/solucionario-libro-jacinto.pdf · g 9 2x 3 x 2 y ahora podemos ... llegamos a la conclusión parcial y = 8 ó y = 1 para los

122

5.- Vamos a tomar el camino de la semejanza de triángulos.

Completaremos el cono y trazaremos su generatriz. Se forman tres triángulos semejantes. En

particular, entre los triángulos “pequeño” y “mediano” se cumple la siguiente relación entre

los lados proporcionales (consideraremos que h es el cateto desconocido, altura, del triángulo

pequeño) h

10

3

6 donde h = 5cm.

Lo que nos conduce a que la altura del cono “grande” es H = 10 + 5 = 15cm.

Como al perforar se pierde el cono superior “pequeño”.

473

5.3.14,3

3

..

12723

15.9.14,3

3

..

22

22

hrV

HRV

CONITO

CONO

3

22

9423301272)28347(1272

28310.3.14,310..

cmV

rV

R

CILIND

PRUEBA 64

1.- Se puede realizar la transformación inicial

xxx 2cossen.21sen.3 7.77 .

Sumamos algebraicamente los exponentes en el miembro derecho y luego como las bases son

iguales se pueden igualar los resultados.

3.sen x + 1 = 2.sen x – cos2x . Si sustituimos el cos2x = 1 – sen2x y transformamos nos queda

la ecuación sen2x – sen x – 2 = 0 que se puede descomponer

(sen x – 2).(sen x + 1) = 0 donde obtenemos sen x = 2 (imposible) ó sen x = –1 que nos

conduce a 2

3x .

2.- Dibujemos un triángulo isósceles de base .AB y vértice opuesto C . Hay que tener mucho

cuidado pues las alturas no pueden quedar, de ningún modo, dentro del triángulo ya que miden

la mitad de lo que mide el lado base del triángulo isósceles. Esto nos lleva rápidamente a que

ABC = BAC = 300 (se oponen a un cateto que es la mitad de la hipotenusa) por lo que

ACB = 1200 (suma de ángulos interiores de un triángulo, en este caso, isósceles ).

a)El triángulo es obtusángulo.

b) Consideremos los pies de las alturas en los puntos D y E .Un razonamiento trigonométrico

en el ADC (que está fuera del triángulo ABC ) nos conduce a calcular la longitud de AC .

Page 123: PRUEBA 1 - math.cubava.cumath.cubava.cu/files/2015/05/solucionario-libro-jacinto.pdf · g 9 2x 3 x 2 y ahora podemos ... llegamos a la conclusión parcial y = 8 ó y = 1 para los

123

Es fácil verificar que DAC = 300 y podemos proponer la vía del coseno.

AC

ADDAC cos lo que implica que .

3

3.8

30cos

40AC

20

2

2

23,973,1.333,52

3.3.

18

64120sen.

3

3.8.

2

1sen..

2

1cmACBACA

.

3.- Los números se denotarán por n ; n + 1 ; n + 2.

Obtenemos la ecuación ( n + 2 )2.( n + 1 ) = 24n que después de realizadas las operaciones

indicadas, reducir y ordenar nos queda n3 + 5n2 – 16n + 4 = 0

que puede ser factorizada y obtener la expresión (n – 2).(n2 + 7n – 2) = 0.

El primer factor nos aporta n = 2 y el segundo no aporta soluciones enteras pues

D = b2 – 4ac = 49 + 8 = 57

Los números son 2 ; 3 y 4. Su suma es 9.

4.- Este ejercicio nos enlaza desigualdad y trigonometría.

Para que la función esté definida tiene que tomarse necesariamente

cos4 x + cos2 x + 2.sen x – 2 0.

Expresión que convertiremos completamente a sen x.

( 1 – sen2x )2 + 1 – sen2x + 2sen x – 2 0 .

Realizando operaciones y transformaciones obtenemos sen4x – 3.sen2x + 2.sen x 0 donde

cambiaremos la variable W = sen x , y llegaremos a que W.(W3 – 3W + 2) 0 que puede ser

factorizada completamente W.(W – 1)2.(W + 2) 0 y que al resolverla nos conduce al

análisis de los signos por los intervalos

+ –2 – 0 + 1 +

///////////////////////////////////////////////////////////////////

W –2 ó W 0 pero como es lógico para sen x debemos analizar solamente que

0 W 1 o sea 0 sen x 1 y esto se cumple solo si 0 x . (primer y segundo

cuadrantes).

5.- Si nos dan el área del cuadrado podemos calcular de manera rápida su diagonal o su lado.

Calculemos la diagonal en la fórmula para el área 2

2

ACAC lo que implica

AC 2 = 2.114,5 = 289 donde AC = 17cm.

a) Si trazamos el segmento ED podemos verificar que AED = 900 (teorema de Tales en el

círculo base) y por tanto ED es la proyección de la oblicua EC sobre el plano base. Se

cumple en el punto E el teorema de las tres perpendiculares ya que AEED . Entonces

ECAE y se cumple que AEC es rectángulo en E.

Page 124: PRUEBA 1 - math.cubava.cumath.cubava.cu/files/2015/05/solucionario-libro-jacinto.pdf · g 9 2x 3 x 2 y ahora podemos ... llegamos a la conclusión parcial y = 8 ó y = 1 para los

124

b) Si EC = 15cm podemos calcular AE por el teorema de Pitágoras

.602

15.8

2

.

8642252891517

2

2222

cmECAE

A

ECACAE

PRUEBA 65

1.- Podemos hacer la transformación del miembro izquierdo convirtiendo a un solo logaritmo

xx

x

22

44log 4 y llevar ahora a la forma de potencia x

x

x

422

44

donde haremos el cambio

2x = a para obtener entonces una ecuación más sencilla

a2 – 4 = a2.(a – 2) que nos conduce a la expresión a3 – 3a2 + 4 = 0 que admite la

descomposición (a – 2)2.(a – 1) = 0 donde a = 2 ó a = 1. Podemos calcular x = 1 ó x = 0

pero ninguno pertenece al dominio de la ecuación original. En la primera ecuación con la

variable ¨a¨ se pudo realizar una simplificación con cuidado de no perder posibles soluciones.

2.- a) El ABC es rectángulo en C (ACB inscrito sobre el diámetro AB ). Ocurre que

ABAE (A es punto de tangencia y AB diámetro) y como AB DC (paralelogramo)

entonces también se cumple DCAE por lo que el AED es rectángulo en E. También

tenemos que ADE = ABC (ángulos opuestos del paralelogramo). Entonces

ABC AED (tienen dos ángulos respectivamente iguales).

b) En el ABC podemos calcular, aplicando el teorema de Pitágoras.

.101005766762426 2222

cmACABCB

De los triángulos semejantes podemos deducir por sus lados proporcionales

AE

AC

DE

CB

DA

AB y como CBDA = 10cm (lados opuestos del paralelogramo) podemos

sustituir los valores y llegar a AEDE

2410

10

26 donde cmDE 85,3

26

10.10 ;

.23,926

24.10cmAE

.8,172

23,9.85,3

2

. 2cmAEDE

A

Page 125: PRUEBA 1 - math.cubava.cumath.cubava.cu/files/2015/05/solucionario-libro-jacinto.pdf · g 9 2x 3 x 2 y ahora podemos ... llegamos a la conclusión parcial y = 8 ó y = 1 para los

125

3.- Si consideramos que en el recipiente A hay “a” litros y en el recipiente B hay “b” litros

tenemos a + b = 90.

Después de la primera operación en el recipiente A quedan .4

3La

y en el B hay

.4

1Lab

. Después de la segunda operación en A hay .15

4

3La

y en el B quedan

.154

1Lab

pero estas cantidades son iguales, o sea 15

4

115

4

3 aba que se puede

llevar al sistema de ecuaciones a + b = 90

2a – 4b = –120

que al resolverlo nos brinda las soluciones a = 40 y b = 50.

Al principio en A había 40L y en B, 50L.

(Este problema puede realizarse por otra vía de forma aritmética).

4.- Es lógico que con una buena calculadora electrónica se puede obtener el resultado. Veamos

como hacerlo con el uso de propiedades, leyes matemáticas, fórmulas de reducción, etc.

2

1

6

7sen

; 3001,0log ; 9

149 3

1log7

;

3

3570 0 tan

102 5

1log1 2

; 236log9log4log 666 .

La expresión nos queda entonces

0856,0

264

3,11.2

36300

)63,17.(2

)63.10).(63.10(

63.10.3

2.3

3

63.10

3

11

23

3.10

9

1).3(1

5.- La base es un triángulo equilátero y en el mismo se sabe la relación que hay entre la altura

y el lado 3.2 lh , o sea, 2

3.lh . En el triángulo rectángulo de apoyo de la arista lateral

vemos que la proyección de la arista es h3

2 (propiedad de la mediana en el triángulo base) ;

podemos sustituir y nos queda 3

3.

2

3..

3

2 ll .

Por otra parte tanl

H .3

3.

12

..

3

3..

4

3..

3

1..

3

1 32

tanltan

llHAV B . La alumna tenía razón.

Page 126: PRUEBA 1 - math.cubava.cumath.cubava.cu/files/2015/05/solucionario-libro-jacinto.pdf · g 9 2x 3 x 2 y ahora podemos ... llegamos a la conclusión parcial y = 8 ó y = 1 para los

126

PRUEBA 66

1.- Primeramente necesitamos calcular 2

13log 9 por lo que nos queda 15 .

La ecuación entonces puede quedar, después de trasponer y elevar al cuadrado, de la siguiente

forma log2 x .( log2 x – log2 4 ) = 15 donde realizaremos el producto indicado y llegamos a

la ecuación 015log.2log 2

2

2 xx .

Entonces descomponemos en factores ( log2 x – 5 )( log2 x + 3 ) = 0 y obtenemos las

siguientes expresiones log2 x = 5 ó log2 x = – 3 las que conducen a las soluciones

x = 25 = 32 ó x = 2–3 = 8

1

2.- a) Vemos que DBA = BAT (alternos entre paralelas). El BAT es semiinscrito sobre

el arco AB por lo que la amplitud de este arco es de 760 . Podemos calcular entonces la

amplitud del arco BC mediante 3600 – ( 1440 + 760 ) = 1400 .

Calculamos ahora que CAB = 700 ( inscrito sobre el arco BC = 1400 ) y obtenemos que

CDB = 700 + 380 = 1080 (por ser ángulo exterior del DAB).

b)Tenemos que COB = 1400 (ángulo central que se opone al arco de 1400). Trazamos la

construcción auxiliar (radios) CO y OB . Restaremos el área del COB del área del sector

circular.

22

0

02

6,1918

352

18

7.4.14,3

360

140..cm

rAS

2002 14,5642,0.840sen.16.2

1140sen..

2

1cmrAT

El área sombreada es A = AS – AT = 19,6 – 5,14 = 14,46 = 15 cm2.

3.- El denominador de la expresión puede escribirse como,

xxx

xxx

x

xxxcossen

sen2

)cos(sensen2

sen2

cos.sen2sen2 2

El numerador se puede transformar así,

xxxxxx

x

xx

x

xx sen.coscos.sencossen

cos

sen1.cos

sen

cos1.sen 223333

xxxxxxxxxx

xxxxxx

cossen)cos.sencoscos.sen)(sencos(sen

)cos(sencossencossen

22

33

Por lo que 1cossen

cossen

xx

xxA

Page 127: PRUEBA 1 - math.cubava.cumath.cubava.cu/files/2015/05/solucionario-libro-jacinto.pdf · g 9 2x 3 x 2 y ahora podemos ... llegamos a la conclusión parcial y = 8 ó y = 1 para los

127

4.- Cantidad de alcohol inicial : h Cantidad de agua inicial : a

Siempre se maneja concentración de alcohol en la disolución .

h + 8 = 90% ( a + h + 8 ) ; h = 75 % ( a + h + 8 )

Nos quedan entonces las ecuaciones h – 9a = – 8

h – 3a = 24

Al resolver el sistema llegamos a los siguientes valores h = 40 y a = 3

15

Hay inicialmente 40 L de alcohol y 3

15 L de agua en la disolución.

5.- Podemos calcular la altura del ortoedro aplicando trigonometría,

100

3,41sen 0 h donde h = 100 . sen 41,30 = 100 . 0,66 = 66

AL = P . h = 210 . 66 = 13860 cm2 = 138,6 dm2 .

PRUEBA 67

1.- Esta ecuación requiere de muchos pasos algebraicos, veamos,

xxx

x 43.33

34 y entonces

3

33.344

xxxx

que conduce a

3

4.34.2 xx y podemos transformar en

3

2

3

4

x

x

Elevamos al cuadrado en ambos miembros de esta última proporción

3

4

3

42

2

x

x

que es equivalente con 3

4

3

42

x

que se cumple solo si 2x = 1.

Por tanto x = 2

1 .

2.- Hay que hacer algunos razonamientos muy interesantes en esa figura.

Como el ABC es isósceles y ABC=1200 entonces CAB = ACB = 300 (ángulos base

de un triángulo isósceles que tiene un ángulo interior de 1200) .

Como CAB = 300 está inscrito sobre BC entonces el arco BC tiene una amplitud de 600

y .20cmrBC

Igual ocurre con .20cmrAB Apreciamos también que ADB=CAB (están inscritos

sobre arcos iguales) por lo que ADB=300.

En los triángulos ADB y AEB tenemos: EBA común y ADB = CAB, entonces

ADB AEB (tienen dos ángulos respectivamente iguales).

¡ Aclaramos que cmAE 14 y no 15cm !

Page 128: PRUEBA 1 - math.cubava.cumath.cubava.cu/files/2015/05/solucionario-libro-jacinto.pdf · g 9 2x 3 x 2 y ahora podemos ... llegamos a la conclusión parcial y = 8 ó y = 1 para los

128

Tenemos entonces las razones DB

AB

AD

AE (lados proporcionales, se oponen a los ángulos

iguales en los triángulos semejantes) y sustituimos 38

2014

AD donde podemos calcular

cmAD 6,2620

38.14

.5,27,2524

38.6,2630sen.38.6,26.

2

1sen...

2

1 20 dmADBDBADA

Otra vía:

Se puede utilizar la vía de resolver el triángulo DAB (ley de los senos) y entonces no es

necesario utilizar la semejanza ni la longitud del segmento .14cmAE

3.- El miembro derecho se puede escribir como 12

10

y entonces nos quedarán

expresiones exponenciales con la base 12

1 (hay que invertir el sentido de la desigualdad al

igualar los exponentes)

0154log

2

1

2

12

3

xx

por lo que nos conduce a

log3 (4x2–5x+1) 0 y entonces 4x2–5x+1 1 donde simplificamos y podemos

descomponer en factores el miembro izquierdo x ( 4x – 5 ) 0 .

Pero tenemos que analizar también que, inicialmente, tiene que cumplirse una desigualdad

muy importante para que esté definida la función logarítmica, por lo que hay que plantear

4x2–5x+1 > 0 y que al descomponerla en factores nos queda

( x – 1 )( 4x – 1 ) > 0. Hacemos los análisis de los signos por intervalos en cada caso,

+ 0 – 5/4 +

////////////////////////////////////////////

/////////////////////////////////////////////OO////////////////////////////////////////////

+ 1/4 – 1 +

Al intersectar los resultados obtenidos llegamos a que la solución es:

x reales en los intervalos [ 0 ; 1/4 ) ( 1 ; 5/4 ] .

Page 129: PRUEBA 1 - math.cubava.cumath.cubava.cu/files/2015/05/solucionario-libro-jacinto.pdf · g 9 2x 3 x 2 y ahora podemos ... llegamos a la conclusión parcial y = 8 ó y = 1 para los

129

4.-Los números se pueden expresar por : n ; n + 2 y n + 4 .

Se cumple que n2 + ( n + 2 )2 > ( n + 4 )2 lo que nos lleva a escribir la ecuación

n2 + ( n + 2 )2 – 48 = ( n + 4 )2 . Efectuando y ordenando convenientemente nos queda

una ecuación mucho más sencilla n2 – 4n – 60 = 0 que se puede descomponer en factores

mediante ( n – 10 )( n + 6 ) = 0 y nos da las soluciones iniciales n = 10 ó n = – 6 .

Los números son 10 ; 12 y 14 . Su semiproducto es 8402

14.12.10 .

También se puede analizar el caso –6 ; – 4 y –2 .

5.- En la figura hay dos triángulos de apoyo interiores que debemos resolver (triángulos

rectángulos) para poder hallar las medidas de la base y las alturas de las caras laterales (que

son triángulos isósceles iguales dos a dos).

En el triángulo de apoyo que tiene la mayor proyección sobre la base:

h1 = 8 cm (altura de la cara lateral por el costado, es el doble del cateto que se opone al ángulo

de 300) y el otro cateto es 3.42

L (se opone al ángulo de 600 y hemos considerado que

L es el largo de la base de la pirámide en la figura) por lo que podemos escribir cmL 38 .

En el triángulo de apoyo que tiene la menor proyección sobre la base:

h2 = cm2.4 (altura de la cara lateral del frente, el triángulo de apoyo es un triángulo

rectángulo isósceles ya que el ángulo de inclinación es de 450) y podemos escribir además que

42

a (porque los catetos son iguales y consideramos que el ancho de la base está dado

por a ) por lo que a = 8 cm.

Calculamos el área lateral,

212 14206,1426441.1.73,1.328.824.382

..2

2

..2 cm

hahLAL

Calculamos el volumen, .14863,1473

73,1.256

3

4.8.38

3

..

3

. 3cmhaLhA

V B

PRUEBA 68

1.- La ecuación se puede escribir, igualando las bases, de manera más simplificada,

5

1

5 22

2

2.22 xx

x

en el miembro derecho, mantenemos la base y sumamos los

exponentes y entonces podemos igualar los exponentes de ambos miembros porque las bases

serán iguales 5

11

5 22

2

xx

x .

Vamos a eliminar los denominadores multiplicando en los dos miembros por el denominador

común y nos queda 1522 xx .

Page 130: PRUEBA 1 - math.cubava.cumath.cubava.cu/files/2015/05/solucionario-libro-jacinto.pdf · g 9 2x 3 x 2 y ahora podemos ... llegamos a la conclusión parcial y = 8 ó y = 1 para los

130

Aislamos la raíz , elevamos los dos miembros al cuadrado y llegamos a la ecuación

bicuadrática x4 – 3x2 – 4 = 0

que podemos descomponer en factores ( x2 – 4 )( x2 + 1 ) = 0 que solo nos aporta los

valores x = 2 ó x = –2 . Se pueden comprobar en la ecuación original.

2.- El área del AOB = 20u2 , entonces x . y = 40 (I).

Por otra parte tenemos que x

y

x

ymtan r

0

06,1122 0 , por lo que podemos extraer

una relación muy importante y = 1,6 . x (II) .

Resolvemos el sistema de las ecuaciones I y II para poder llegar al resultado que nos piden

x ( 1,6 x ) = 40 ; donde 1,6 . x2 = 40 por lo que 5256,1

40x

y solo podemos tomar x = 5 porque nos indican que las coordenadas son positivas, por lo

que podemos calcular que y = 8 .

Los puntos son A( 5 ; 0 ) y B( 0 ; 8 ) . Ahora podemos hallar una ecuación de la recta

y – 0 = –1,6 ( x – 5 ) , o sea, 1,6x + y = 8 que es igual a 8x + 5y = 40 .

3.- Necesitamos hacer en este ejercicio una diferenciación de casos debido a la expresión

x.

Si x 0 entonces x= x , por lo que 3 – 2x – x2 > 0 , o sea, x2 + 2x – 3 < 0 que al

descomponerla en factores nos queda ( x + 3 )( x – 1 ) < 0 y al resolverla nos conduce a que

– 3 < x < 1 pero la condición inicial del caso nos indica que 0 x < 1 .

Si x < 0 entonces x= –x , por lo que 3 + 2x – x2 > 0 , o sea, x2 – 2x – 3 < 0 que al

descomponerla en factores nos queda ( x – 3 )( x + 1 ) < 0 y al resolverla nos conduce a que

–1 < x < 3 pero según la condición del caso nos queda –1 < x < 0 .

Ahora, al unir las dos soluciones, llegamos a que – 1 < x < 1. Solo nos falta ahora, haciendo

la misma diferenciación de casos, hallar los valores reales dentro del intervalo

–1 < x < 1 que anulan el denominador, o sea, los que hacen que 3 – 2x –x2 = 1 .

Obtenemos cuatro valores para la x pero únicamente hay dos dentro del intervalo que hemos

obtenido como respuesta, por lo que x 1 – 3 y x –1 + 3 .

4.- Juan obtuvo notas P1 y P2 . Tenemos que 5,942

21 PP

Planteamos la diferencia P2 – P1 = 10%.P1 .

Nos queda entonces un sistema a resolver P1 + P2 = 189 .10

11P1 – 10P2 = 0

Sumando ordenadamente las dos ecuaciones después de multiplicar la primera por el número

indicado obtenemos el valor para la variable P1 = 90 y P2 = 99.

Obtuvo 90 puntos en el primer examen y 99 en el segundo .

Page 131: PRUEBA 1 - math.cubava.cumath.cubava.cu/files/2015/05/solucionario-libro-jacinto.pdf · g 9 2x 3 x 2 y ahora podemos ... llegamos a la conclusión parcial y = 8 ó y = 1 para los

131

5.- Si DBC = 600 entonces BDC = 300 (ángulos complementarios, suman 900).

Ocurre que cmBC 10 (se opone al ángulo de 300 en el triángulo rectángulo, es la mitad de

la longitud de la hipotenusa) y cmDC 310 (se opone al ángulo de 600).

a) .6,5473,2.2073,1.2020)310.(2)10.(2 cmP

b) DC es la proyección de la oblicua SC sobre el plano y CBDC debido al rectángulo

base; entonces CBSC (teorema de las tres perpendiculares en el punto C). El triángulo

BCS es rectángulo.

En la fórmula para el volumen de la pirámide hallaremos la altura SD ,

2503

.2

3.10.10

3

.

hhA

V B entonces nos queda 750.3.50 h donde calculamos

cmh 353.50

750

Aplicamos el teorema de Pitágoras en el triángulo rectángulo SDC (rectángulo en D pues SD

es la altura, perpendicular al plano base, perpendicular a todas las rectas del plano que pasen

por el punto D)

cmDCSDSC 38,1973,1.24,2.53.5.5)310()35( 2222

2979,962

10.38,19cmA

.

PRUEBA 69

1.- Se despeja el logaritmo y se convierte a potencia 9x –2.3x – 2 = 3x–1 .

Podemos hacer la sustitución y = 3x ; entonces nos queda la ecuación cuadrática

3

222 yyy que se puede expresar como 3y2 –7y –6 = 0 y nos aporta las

soluciones y = 3 ó y 3

2 . Hacemos reversible el cambio de variable y obtenemos

3x = 3 donde x = 1 ; 3x 3

2 es imposible para la función exponencial.

Única solución x = 1.

2.- El ejercicio puede tener varias vías de solución. Trazamos CD y BE .

Se forma el DBE rectángulo en B (ángulo inscrito sobre el diámetro, teorema de Tales).

Como C es punto medio entonces DEAB . Tenemos que cmCB 9 es la altura relativa

a la hipotenusa, entonces CEDCCB .2

(teorema de la altura) y aquí podemos calcular

cmDC

CBCE 27

3

922

, lo que nos conduce al cálculo del diámetro cmDE 30 .

Page 132: PRUEBA 1 - math.cubava.cumath.cubava.cu/files/2015/05/solucionario-libro-jacinto.pdf · g 9 2x 3 x 2 y ahora podemos ... llegamos a la conclusión parcial y = 8 ó y = 1 para los

132

En el triángulo AOC , rectángulo en C, podemos calcular la amplitud del AOC , ya que

6,015

9sen

AO

ACAOC y entonces podemos hallar que

AOC = 36,90 para calcular AOE = 1800–36,90 = 143,10 (ángulos adyacentes, son

suplementarios, suman 1800).

Calculamos el área del sector circular,

2

0

02

0

2

8,283,280360

1,143.15.14,3

360

..dm

rAS

3.- En el sistema calcularemos las incógnitas en función de k . Podemos multiplicar la

segunda ecuación por (–k) y sumarla con la primera para encontrar que 4

122

k

ky ;

entonces 4

82

k

kx . En cada caso se tiene que cumplir que

x >0 ; y > 0 a la vez, por lo que hay que establecer las dos desigualdades

0)2)(2(

12

4

122

kk

k

k

k que nos conduce al análisis de signos por intervalos

– –2 + 1/2 – 2 +

O///////////////////OO/////////////////////////////////////

y también

0)2)(2(

8

4

82

kk

k

k

k que nos lleva a los signos por intervalos

– –2 + 2 – 8 +

O///////////////////////////////////////OO///////////////////

Al intersectar los gráficos hallamos el conjunto de valores de k que cumplen la condición

k reales con – 2 < k < 1/2 ó k > 8 .

4.- El pintor más eficiente pinta el ómnibus en 6 h ; entonces en 1 h pintará 6

1del ómnibus.

El otro pintor, en 1 h pintará 8

1 del ómnibus . Entre los dos, en 1 h pintarán

24

7

8

1

6

1

del ómnibus. Aplicamos una sencilla proporción que nos conduce a una ecuación fraccionaria

T

1

24

7 donde podemos calcular el tiempo que, trabajando juntos, emplearán en pintar un

ómnibus T = 3,43 h que significan 3 h y 0,43 h , o sea,

3 h y 0,43.60 min = 26 min .

Entre los dos pintan un ómnibus en 3h y 26 min . Para pintar dos ómnibus y almorzar

consumirán, aproximadamente 7 h con 52 min .

Page 133: PRUEBA 1 - math.cubava.cumath.cubava.cu/files/2015/05/solucionario-libro-jacinto.pdf · g 9 2x 3 x 2 y ahora podemos ... llegamos a la conclusión parcial y = 8 ó y = 1 para los

133

5.- Como la altura es perpendicular a la base, es perpendicular a toda recta de esa base que

pase por B ; por lo que BTOB . Afirmamos ahora que BT es la proyección de la oblicua

OT sobre la base y ATBT (radio en el punto de tangencia) entonces ATOT (teorema de

las tres perpendiculares en el punto T).

El OAT es rectángulo en T y entonces cmAT 10 (se opone al ángulo de 300 en el

triángulo rectángulo, es la mitad de la longitud de la hipotenusa .20cmAO )

En el OBT , rectángulo, podemos aplicar el teorema de Pitágoras y

calcular cmOBOTOB 86436100610 2222

.

Entonces h = 8 cm y podemos calcular el área lateral,

AL= 2..r.h = 2.3,14.6.8 = 301,4 cm2

PRUEBA 70

1.- La ecuación dada puede escribirse de la siguiente forma para poder luego igualar los

exponentes, xxtanxtanx

sensen21 2 entonces podemos igualar los exponentes

1 – 2 sen2x = –sen x ; lo que nos lleva directamente a la ecuación

2 sen2x – sen x – 1 = 0 que la podemos descomponer en factores mediante

( sen x – 1 )( 2sen x + 1 ) = 0 lo que nos implica que senx = 1 ó sen x 2

1 .

De la primera obtenemos x = /2 (no pertenece al dominio de la ecuación original), de la

segunda obtenemos dos posibles valores (la función sen x tiene imángenes negativas en los

cuadrantes 3ro. y 4to.) por lo que x = 7/6 ó x = 11/6 (esta tampoco pertenece al

dominio de la ecuación original, análisis de mucho interés).

Ya tenemos la abscisa del punto, ahora calcularemos la ordenada,

76,058,03

73,1

3

3

6

7

6

7 2

1

3

7cos

tanf entonces el punto es

P( 7/6 ; 0,76 )

Page 134: PRUEBA 1 - math.cubava.cumath.cubava.cu/files/2015/05/solucionario-libro-jacinto.pdf · g 9 2x 3 x 2 y ahora podemos ... llegamos a la conclusión parcial y = 8 ó y = 1 para los

134

2.- Trazamos la cuerda BE y tenemos que FBE = 150 (inscrito sobre el arco EF que tiene

una amplitud de 300) . Un sencillo análisis en el ABE nos conduce a que

AEB = 300 (suma de ángulos interiores de un triángulo). Entonces el arco AB tiene

una amplitud de 600 ( AEB = 300 inscrito sobre el arco AB ) . Podemos decidir que

rAB (cuerda que sustenta un arco de 600).

En el BCE aplicamos la ley de los senos 00 30sen15sen

94,12 BC y podemos calcular la longitud

de cmBC 252588,0

5,0.94,12 , o sea, r = 25 cm.

AR = ACIR – ACUA = .r2 – r2 = r2 ( – 1 ) = 252 ( 3,14 – 1 ) =625 . 2,14 = 1337,5

AR = 13,38 dm2 .

3.- En principio k > 0 ; k 1 por lo que x2 + 2x – 8 > 0, o sea, (x + 4).(x – 2) > 0. Pero se

tiene que cumplir también que 6 – x – x2 > 0 para que está definido el logaritmo, donde

x2 + x – 6 < 0 lo que nos conduce a (x + 3).(x – 2) < 0. Al resolver respectivamente las

desigualdades obtenemos los intervalos

+ –4 – 2 +

////////////////////////OO/////////////////////

+ –3 – 2 +

O//////////////////O

Al intersectar los dos gráficos vemos que no hay elementos comunes por lo que la función h

no existe.

4.- Son tres los implementos deportivos P , B y G .

En principio P + B + G = 100 y luego podemos obtener la otra ecuación con la frase de

Eduardo B = 2P + G + 7 . Las mujeres, como siempre buenas calculadoras, a través de la

alumna María, expresan la opinión más eficiente 121 = 4G + P . El sistema puede

escribirse ordenado

P + B + G = 100

2P – B + G = – 7

P + 4G = 121

Page 135: PRUEBA 1 - math.cubava.cumath.cubava.cu/files/2015/05/solucionario-libro-jacinto.pdf · g 9 2x 3 x 2 y ahora podemos ... llegamos a la conclusión parcial y = 8 ó y = 1 para los

135

Si sumamos las dos primeras ecuaciones obtenemos una nueva ecuación únicamente en

las variables P y G que podemos trabajarla con la última 3P + 2G = 93 .(–2)

P + 4G = 121

Después de multiplicar la primera por (–2) se suma a la segunda y nos conduce a la expresión

–5P = – 65, lo que nos permite calcular P = 13 , G = 27 y B = 60.

El bate cuesta $60 , el guante, $27 y la pelota, $13.

5.- Con los datos que nos da el ejercicio podemos calcular rápidamente la altura de la

pirámide. [Importante: 6,4 dm3 = 6400 cm3 ] 3

.hAV B

P

cmA

Vh

B

32600

19200

40.30.2

1

6400.3.3

En el triángulo ABC , rectángulo en A, trazamos la altura relativa a la hipotenusa que

llamaremos AD y también trazaremos la oblicua SD .

Podemos verificar que AD es la proyección de la oblicua SD pues SA es la altura y como

BCAD entonces se cumple que BCSD (teorema de las tres perpendiculares en el

punto D).

Como conocemos el área del triángulo ABC podemos calcular AD , pero antes hay que

calcular la hipotenusa del ABC . Si cmAC 40 y cmAB 30 entonces podemos decidir

que cmBC 50 (trío de números pitagóricos).

2

.ADBCA lo que implica que cm

BC

AAD 24

50

600.2.2 .

En el ADS , rectángulo en A , podemos ahora calcular SD (que es la altura relativa al

segmento BC en el BSC.

Como cmSA 32 y cmAD 24 entonces podemos obtener cmSD 40 (trío de números

pitagóricos).

Calculamos el área del BSC,

.2101000

2

40.50

2

.dm

SDBCA .